2005工程硕士GCT考试真题(英语、语文、逻辑)

时间:2019-05-14 11:57:59下载本文作者:会员上传
简介:写写帮文库小编为你整理了多篇相关的《2005工程硕士GCT考试真题(英语、语文、逻辑)》,但愿对你工作学习有帮助,当然你在写写帮文库还可以找到更多《2005工程硕士GCT考试真题(英语、语文、逻辑)》。

第一篇:2005工程硕士GCT考试真题(英语、语文、逻辑)

第一部分 语言表达能力测试(50题,每题2分,满分100分)

一、选择题

1.下面各组词语中,没有错别字的一组是:

A.乐此不彼 密云不雨 墨守成规 循规蹈矩 B.靡靡之音 诲人不倦 漫不经心 循循善诱 C.坚守自盗 杀鸡儆猴 出人头地 力挽狂澜 D.投机取巧 骇人听闻 目不瑕接 龙潭虎穴 2.下列加点字的释义全都正确的是

A.失宠(偏爱)韬(显示)光养晦 老骥伏枥(马槽)B.爽(失)约 马革(皮)裹尸 作茧自缚(捆绑)C.舞蹈(顿足)既往不咎(过错)悲天悯(怜惜)人 D.龌龊(肮脏)不知端倪(头绪)臭(难闻的)味相投 3.下没有语病的一句是

A.“费改税”能否遏制住愈演愈烈的乱收费现象,这对农村工作是个考验。B.亚健康状态,往往受到无规律的生活和沉重的学习工作压力而引起。C.一听说有盛夏的免费音乐节,许多的附近居民早早就赶来,等待观看。D.就反腐败这样一个国际性的顽疾来说,“公开”是最有效的一剂良药。4.下面各句中,语意明确、没有歧义的一句是 A.据外电报道,昨日新加坡一油轮起火爆炸。B.小李见老王把他的书撕了,便揪住老王让他赔。C.几个公司的领导对这个调整方案意见不一,争执不下。D.工作组来我县调查的前三天,整个县城就己沸沸扬扬。5.对下面这段话使用的修辞手法分析不当的一项是

风是调皮的小男孩,抓把土抛到空中,趁机扯乱女孩子的长发;风是年老的画家,一味选灰色调,造出黄昏的画面;风是不高明的小偷,溜进屋时弄响了门,逃走时还在窗上留下了脚印。

A.这段话使用了拟人的修辞手法。

B.这段话运用了三个长度、句式风格相近的句子,这是排比的修辞手法。C.这段话使用了明喻,分别把风比作“小男孩”、“年老的画家”、“小偷”。D.这段话中几种修辞方法结合得不露痕迹,恰到好处。

6.下面这首古诗描述了我国民间一个传统节令的景象,这个节令是 中庭地白树栖鸦,冷露无声湿桂花。今夜月明人尽望,不知秋思落谁家?

A.重阳 B.七夕 C.中秋 D.元宵 7.下面表述不正确的一项是

A.13世纪的《马可•波罗行记》是最早较系统地向欧洲介绍中国的一部游记。B.圆明园于1860年被英法联军抢劫焚烧之后,1900年又被八国联军彻底破坏。C.中国第一所现代大学是1898年建立的京师大学堂,1912年改名为北京大学校。D.明清殿试合格进士分为三甲,一甲第一名称状元,第二名称探花,第三名称榜眼。8.以下表述不正确的是

A.世界三大宗教是佛教、基督教、伊斯兰教。B.佛教的创始人是释迦牟尼。C.道教的创始人是老子。D.儒学的创始人是孔子。

9.“一把钥匙开一把锁”的说法在教育学体现了人的发展的 A.阶段性 B.不平衡性 C.顺序性 D.个别差异性

10.《书愤》一诗借诸葛亮抒怀:“《出师》一表真名世,千载谁堪伯仲间。”这首诗的作者是

A.杜甫 B.杜牧 C.辛弃疾 D.陆游 11.经济学家认为,经济学的产生源于资源的 A.稀缺性 B.可分配性 C.可分割性 D.可使用性

12.根据法律规定,县人民代表大会代表不得直接提出罢免的是 A.县长 B.县公安局长 C.县检察长 D.县法院院长

13.行政机关或者行业组织依法实施公民特定资格考试时,下列说法正确的是 A.应当事先组织考前培训 B.应当事先公布考试大纲 C.应当事先制定培训教材 D.应当事先指定助考材料 14.以下人类活动对生物多样性造成威胁的是 A.管制狩猎和物种贸易 B.引进外来新物种 C.动物圈养 D.保护栖息地 15.排泄系统在体内收集由细胞产生的废弃物,并将它们排出体外。通过排泄人体中的有害物质,可以保持人体内环境的稳定。大多数废弃物通过肾排出体外,其余少部分则通过其他器官排出体外。以下选项中不是排泄器官的是 A.肺 B.皮肤 C.脾 D.肝脏

二、填空题

16.在下列各句横线处,依次填入最恰当的词语。

①外表大大咧咧的她,其实内心里深藏着不为人知的丰富________的感情。

②经过________深入的调查研究,这家公司最终选择上海作为进入中国市场的切入点。③国大名绣之一的苏绣,向来以其工艺精湛、针法________而闻名世界。A.细密 细腻 细致 B.细腻 细致 细密 C.细腻 细密 细致 D.细致 细密 细腻 17.从修辞角度看,在下列各句横线处,依次填入最恰当的词语。

苦瓜苗就像一个________一样,自生自长,蔫蔫的,一副要死不活的样子。然而,它却努力地活了下来,便渐渐地长大。它似乎满是敌意,一天比一天不规矩,或绕着南瓜藤,或在冬瓜架上________。我不得不找来一根棍子,把它绑在棍子上,________。A.病儿 胡搅蛮缠 限制它的生长 B.弃儿 胡乱缠绕 限制它的生长 C.病儿 胡搅蛮缠 限制它的行为 D.弃儿 胡乱缠绕 限制它的行为 18.在下列各句横线处,依次填入最恰当的关联词。

①上千吨的轮船碰上这么大的风浪也得上下颠簸,________这么一只小船。②他的动作完成得简直无可挑剔,________全体裁判员无一例外地亮出了满分。③我们在对某些教育现象进行分析后,________应该思考教育的更深层次问题。A.况且 以至 进而 B.何况 以至 进而 C.况且 以致 从而 D.何况 以致 从而 19.在下面文字横线处,填入上下文衔接最好的一句话。

只有经济大大发展了,经济实力和综合国力大大增强了,________。

A.国家才能长治久安,人民的生活才能不断改善,我们的腰杆子才能更硬,在国际上说话才更有分量。

B.国家才能长治久安,我们的腰杆子才能更硬,人民的生活才能不断改善,在国际上说话才更有分量。

C.人民的生活才能不断改善,国家才能长治久安,我们的腰杆子才能更硬,在国际上说话才更有分量。

D.人民的生活才能不断改善,我们的腰杆子才能更硬,国家才能长治久安,在国际上说话才更有分量。20.在下面文字横线处,依次填入最恰当的标点符号。

现在的小孩________从小生活在爸爸身边________爸爸长得高矮胖瘦________说话的口头语________喜欢的京剧还是球赛________他都清楚________而我尚未出生________爸爸就去打日本鬼子去了________不到四岁________爸爸已经战死沙场了________ A.。,„„。,。B.,,、„„。,。C.,,„„。,。D.。,、„„。,。21.________不是秦始皇执政时所为。

A.分天下为三十六郡 B.焚书坑儒 C.统一度量衡 D.盐铁国营 22.________是国民党爱国将领中第一个率军奋起抗日的。A.马占山 B.张自忠 C.蔡廷锴 D.冯玉祥

23.“小小寰球,有几个苍蝇碰壁。嗡嗡叫,几声凄厉,几声抽泣。蚂蚁缘槐夸大国,蚍蜉撼树谈何易。正西风落叶下长安,飞鸣镝。多少事,从来急;天地转,光阴迫。一万年太久,只争朝夕。四海翻腾云水怒,五洲震荡风雷激。要扫除一切害人虫,全无敌。”这首《满江红》是________的作品。

A.鲁迅 B.郁达夫 C.郭沫若 D.毛泽东 24.“已所不欲,勿施于人”这句话出自________。A.《诗经》 B.《道德经》 C.《庄子》 D.《论语》 25.________是陆文夫的小说。

A.《美食家》 B.《活着》 C.《顽主》 D.《人到中年》

26.宪法的修改必须由全国人大常委会或者________以上全国人民代表大会提议,并由全国人民代表大会以________以上的多数通过。

A.十分之一,全体代表的三分之二 B.五分之一,全体代表的三分之二 C.五分之一,到会代表的三分之二 D.三分之一,到会代表的三分之二 27.________是描述收入分配差异程度,反映贫富差距的一个重要指标。A.收入弹性系数 B.恩格尔系数 C.基尼系数 D.消费价格指数 28.“近朱者赤,近墨者黑”是指________因素对人发展的影响。A.遗传 B.环境 C.教育 D.个人活动

29.冻土问题是青藏铁路建设的难题之一。冻土是由固体矿物颗粒,未冻水、冰和气体组成的一种特殊土壤。铁路路基的修建,改变了冻土的物理特性,引起冻土融化下沉,或者冻结膨胀,严重危及路基的_________,会给列车运营带来隐患。A.透气性 B.防腐蚀性 C.稳定性 D.可塑性

30.________是一种由于脂类物质如胆固醇堆积而引起的血管壁增厚的疾病,限制了血液在动脉流动的空间。

A.心脏瓣膜衰坏 B.高血压 C.心肌梗塞 D.动脉粥样硬化

三、阅读理解题

(一)阅读下面短文,回答下列五道题。车站旁有一棵婆娑的老树。老树下两个孩子做着游戏——

“我们都是木头人,不会说话不会动。一不许笑,二不许动,三不许交头接耳听。看谁的意志最坚定。”

我欣然——这是一个古老的游戏了。

“我们都是木头人,不会说话不会动。一不许笑,二不许动,三不许交头接耳听。看谁的意志最坚定。”

我哑然——这是一个古老的游戏了!车不来。孩子依旧做着——

“我们都是木头人,不会说话不会动。一不许笑,二不许动,三不许交头接耳听。看谁的意志最坚定。”

我陶然——这是一个古老的游戏了!

“我们都是木头人,不会说话不会动。一不许笑,二不许动,三不许交头接耳听。看谁的意志最坚定。”

我惘然——这是一个古老的游戏了!!车不来。孩子依旧做着——

“我们都是木头人,不会说话不会动。一不许笑,二不许动,三不许交头接耳听。看谁的意志最坚定。”

我慨然——这是一个古老的游戏了!!

“我们都是木头人,不会说话不会动。一不许笑,二不许动,三不许交头接耳听。看谁的意志最坚定。”

我愕然——这是一个古老的游戏了!!!车依旧不来。孩子也依旧做着„„ 老树下——我已怆然!!!

(路东之《!!!》,载《世界华文微型小说大成•作品》,上海文艺出版社,1992)

31.对于以六个惊叹号为题目,最正确的理解是 A.孩子做了六次游戏 B.我看了六次游戏

C.我等了六次车 D.我看游戏时感情的六次变化 32.对孩子做游戏时所说的六句话,理解最肤浅的是 A.只是一个古老的游戏 B.陈腐的礼教束缚个性 C.古老的传统根深蒂固 D.腐朽的教育是培养木头人 33.对文中“老树”二字肤浅的理解是

A.象征环境的古老 B.将树之老与孩子小加以对比

C.实写故事发生的地点 D.老树却依然婆娑,象征陈腐势力仍然顽强 34.对以下词语理解错误的是

A.欣然—看到孩子能做古老游戏,觉得欣然自慰 B.哑然—观看孩子做古老游戏而哑然不语,正在沉思 C.陶然—观看孩子做着古老游戏,感到陶然有趣 D.惘然—孩子虽然做着古老游戏,却是惘然不解其意 35.对以下词语理解错误的是

A.慨然—为什么要让孩子成为木头人呢?我慨然不满

B.愕然—为什么要做这种比赛成为木头人的游戏呢?我感到惊愕 C.怆然—车依旧不来,我失望了

D.怆然—由孩子们的游戏,我联想到很多,深感痛苦、悲伤

(二)阅读下面短文,回答下列五道题。

何谓文化?向来狭义的解释,只指学术技艺而言,其为不当,自无待论。说得广的,又把一切人为的事都包括于文化之中,然则动物何以没有文化呢?须知文化正是人之所以异于他动物的。其异点安在呢?凡动物,多能对外界的刺戟而起反应,亦多能与外界相调适。然其与外界相调适,大抵出于本能,其力量极有限,而且永远不过如此。人则不然。所以人所处的世界,与动物所处的世界,大不相同。人之所以能如此,(一)由其有特异的脑筋,能想出种种法子。

(二)其手和足的作用分开,能制造种种工具,以遂行其计划。

(三)又有语言以互相交通,而其扩大的即为文字。此人之所知,所能,可以传之于彼;前人之所知,所能,并可以传之于后。因而人的工作,不是个个从头做起的,乃是互相接续着做的。不像赛跑的人,从同一地点出发,却像驿站上的驿夫,一个个连接着,向目的地进行。其所走的路线自然长,而后人所达到的,自非前人所能知了。然则文化,是因人有特异的禀赋,良好的交通工具而成就的控制环境的共业。动物也有进化,但他的进化,除非改变其机体,以求与外界相适应,这是要靠遗传上变异淘汰等作用,才能达到目的的,自然非常迟慢。人则只须改变其所用的工具,和其对付事物的方法。我们身体的构造,绝无以异于野蛮人,而其控制环境的成绩却大不相同,即由其一为生物进化,一为文化进化之故。人类学上,证明自冰期以后,人的体质,无大变化。埃及的尸体解剖,亦证明其身体构造,与现今的人相同。可见人类的进化,全是文化进化。恒人每以文化状况,与民族能力,并为一谈,实在是一个重大的错误。遗传学家,论社会的进化,过于重视个体的先天能力,也不免为此等俗见所累。至于有意夸张种族能力的,那更不啻自承其所谓进化,将返于生物进化了。从理论上说,人的行为,也有许多来自机体,和动物无以异的,然亦无不披上文化的色采。如饮食男女之事,即其最显明之例。所以在理论上,虽不能将人类一切行为,都称为文化行为,在事实上,则人类一切行为,几无不与文化有关系。可见文化范围的广大。能了解文化,自然就能了解社会了。人类的行为,源于机体的,只是能力。其如何发挥此能力,则全因文化而定其形成。(选自吕思勉《中国文化史•绪论》,《中国通史》,华东师范大学出版社,1991)36.本文的主旨在于

A.解释文化的范围和重要性 B.强调人与动物的不同 C.突出人类社会的文化进化 D.证明遗传学家的错误 37.根据本文的意见,人区别于动物在于

A.动物不能适应和改变环境 B.人类个体的先天能力很强

C.人类有良好的交通工具 D.文化的进化和承继使人类能力剧增 38.按照本文,下列说法不正确的是

A.语言文字是人类的文化能力 B.饮食男女之事与文化无关 C.动物适应环境往往出于本能 D.了解文化也就能了解社会 39.按照本文,下列说法正确的是 A.人类行为都是文化行为 B.制造工具和制定计划,使人类优于动物 C.人类的进化主要体现在文化的继承上 D.现代人的身体构造因文化而优于野蛮人 40.按照本文,种族论者的思想错误在于 A.不承认人能制造工具和制定计划 B.不承认人具有控制环境的能力 C.否认遗传上的变异淘汰等作用 D.将种族的能力归因于生物进化

(三)阅读下面短文,回答下列五道题。

上海人爱吃大闸蟹,阳澄湖的大闸蟹更是名噪国内外。但吃完大闸蟹,看到满桌狼藉的蟹骸,你可曾想过它的用途,可曾想过变废为宝?

其实,蟹壳中还有一种丰富的自然资源,即甲壳素。甲壳素又名蟹壳素、聚乙酰氨基葡萄糖等,是一种维持和保护甲壳动物和微生物躯体的线性氨基多糖,广泛存在于节足动物类的翅膀或外壳中。壳聚糖又称聚氨基葡萄糖、可溶性甲壳素,是由甲壳素经脱乙酰化反应转化变成的分子量为12~59万的生物大分子。从勃拉康于1811年描述甲壳素至今,甲壳素和壳聚糖已有100多年的发展历史。

也许你没有发现,在我们的日常生活中时时可见壳聚糖的身影。壳聚糖具有很好的抗菌活性,将其添加到固液食品中,既会对汁液有一定的澄清作用,又可以起到防腐保鲜作用,又由于壳聚糖只能溶解在弱酸中,因此特别适合酸性或低酸性的食品保鲜,如常添加于腌制食品中或用于海产、水果和保鲜。

用壳聚糖做原材料,让其溶液在聚乙烯膜上蒸发,进而在一个钢膜中加压即可形成软性接触眼镜,即隐形眼镜。壳聚糖作为原材料的制备的隐形眼镜,其优异的透氧性和促进伤口愈合的特性,为发炎或受伤的眼睛的辅助治疗提供了美好前景。

壳聚糖具有良好的生物相容性和生物降解性,降解产物一般对人体无毒副作用,在体内不积蓄。用甲壳素、壳聚糖等原料制成的人工皮肤吸水性、透气性、组织相容性良好。若与乙酸合用,还有镇痛、抗感染等功效。

用甲壳素制成的吸收型外科手术缝合线力学性能良好,组织反应好,柔软性好,易打结,具有创伤治愈效果,伤口修复快,创口平整漂亮。在手术伤口愈合过程中,甲壳素缝线在体内的抗张强度逐渐下降,并在溶菌酶作用下首先分解成低聚糖,然后经一系列的化学反应,一部分以二氧化碳的形式由呼吸道排出体外,另一部分则以糖蛋白的形式为人体吸收利用。

(选自奚同庚主编《原来如此•无所不在的材料》,上海科学技术文献出版社,2005)41.以下各项壳聚糖的作用,本文并未提及的是 A.食品包装 B.汁液澄清 C.食物保鲜 D.食品防腐 42.以下关于壳聚糖的描述有误的是 A.是分子量为12~59万的生物大分子 B.来源于脱乙酰化反应的甲壳素 C.可溶于任何浓度的酸性溶液

D.从蜘蛛类和甲壳类动物的外壳中可制取出壳聚糖

43.关于在手术伤口愈合过程中甲壳素缝线变化的描述,不恰当的是 A.抗张强度逐渐下降 B.溶菌作用下分解成低聚糖 C.化学反应后最终得到多样产物 D.所有的生成物质都将被排出体外 44.关于用甲壳素制成外科手术缝合线的描述,错误的是 A.具有良好的吸水性、透气性 B.具有良好的抗菌活性 C.具有良好的力学性能和柔软性、易打结 D.具有良好的生物相容性和生物降解性

45.根据本文内容,以下关于壳聚糖利用的论述正确的是 A.在发炎或受伤的眼睛的辅助治疗方面疗效显著 B.因其良好的组织相容性可制成人工皮肤 C.因其良好的成膜性被制成隐形眼镜 D.特别适合于碱性或弱碱性食品的保鲜

(四)阅读下面短文,回答下列五道题。

利用机会就是乘时:机会稍纵即逝,所以______而______以外,还该______而______。治生或者说致富的过程,在那时有三个阶段:第一,“无财作力”,以劳力赚钱。稍有积蓄,便是第二,“少有斗智”,因为资本太小,一方面亏蚀不起,一方面要争取暴利,不得不费尽心机,想出许多“花招”来取胜,于是第三:“既饶争时”,默察时势环境的变化,掌握机会,大量进货、抛售,或作远程的投资。

这个过程与目前的情况,几乎完全吻合;所谓“少有斗智”,正就是中小企业经营之所以特感吃力之故,一旦有了数千万的资本,就不必再去冒险投资,只要挑有益于国计民生的企业投资,必有盈利,成为“不窥市井,不行异邑,坐而待收,身有处士之义”的现代“封君”。但是,这样舒服的做法不大可能成为巨富的。要成巨富,还须“争时”,放远眼光、走在别人前面;当然,仅有眼光是不够的,作为一个第一流的货殖家,要像刁间那样具备“智”、“勇”、“仁”、“强”四种品德。

这是白圭的理论,而为刁间所充分实践。白圭是洛阳人,生在战国初期,周威烈王的那个时代;其时有个有名的经济学家,名叫李悝,帮助魏文候从事经济革新,获得极大的成功。李悝是个重农主义者,所以他的经济学说,偏重在农业方面的改革,概括而言,可得八个字:增加生产、调节盈虚。

(选自高阳《假官真做清官册》,《高阳作品集》,上海三联书店,2003)46.根据上下文,在文中横线处按顺序填入词语,最恰当的一组是 A.坐 争时 起 乘时 B.坐 乘时 起 争时 C.起 乘时 坐 争时 D.起 争时 坐 乘时 47.根据上下文,刁间是

A.富有智谋的“封君” B.成功的商人 C.著名的经济学家 D.优秀的社会实践者 48.以下经商的行为,符合原文论述的是

A.不把鸡蛋放在一个篮子里 B.船小好调头

C.该出手时就出手 D.拣了芝麻,丢了西瓜 49.按照白圭的理论,最有可能成为巨富的人是 A.眼光超前、行动迅速的人

B.懂得争时且具有非凡商业智慧的人 C.商业意识超前、品德优秀的人 D.市场洞察力敏锐、决策果断的人 50.根据原文,下列理解不正确的一项是 A.在开始创业的时候,只能靠劳力积累资本

B.为了避免风险,中小企业应该注重商业策略的运用 C.商业道德对于商业成功也是非常重要的 D.一流的企业家应该具有敏锐的市场洞察力

第一部分参考答案

题号 1 2 3 4 5 6 7 8 9 10 答案 B B A A C C D C D D 题号 11 12 13 14 15 16 17 18 19 20 答案 A B B B C B D B C A 题号 21 22 23 24 25 26 27 28 29 30 答案 D A D D A B C B C D 题号 31 32 33 34 35 36 37 38 39 40 答案 D A C D C A D B D D 题号 41 42 43 44 45 46 47 48 49 50 答案 A C D A B B B C C B 第三部分逻辑推理能力测试(50 题。每题2 分。满分100 分)

1.没有一个宗教命题能够通过观察或实验而被验证为真。所以无法知道宗教任何命题的真 实性。

为了合乎逻辑的推出上述结论,需要假设下面哪项为前提?

A.如果一个命题能够通过观察或实验能被证明为真,则其真实性是可以知道的。B.只凭或实验无法证实任何命题的真实性。

C.只要知道一个命题的真实性,需要通过观察或实验证明它为真。D.人们是通过信阳来认定宗教命题的真实性。

2.对待知识分子的态度,标志着一个民族的文明程度;对待工人和农民的态度,则靠验着 这个民族的良知与良心。因此_______ A.因此应该给知识分子、工人和农民同等的待遇。

B.如何对待工人和农民,甚至比如何对待知识分子更重要。C.知识分子在待遇方面可以高于工人和农民一倍。D.应该善待知识分子,也同样也应该善待工人和农民。

3.自从有皇帝以来,中国的正史都是皇帝自己家的日记,那是皇帝的标准像,从中不难看 出皇帝的真实形态来。要了解皇帝的真面目,还必须读野史,那是皇帝的生活写照。以下哪 项陈述是上述论证所依赖的假设?

A.所有正史记述的都是皇帝家私人的事情。

B.只有读野史,才能知道皇帝那些鲜为人知的隐私。C.只有将正史和野史结合起来,才能看出皇帝的真面目。

D.正史记述的是皇帝治国的大事,野史记述的则是皇帝日常的小事。

4.近年来我国房价快速攀升。2004 第四季度政府各部门出台多项措施,以压制房价的过快增

长,但2005 年第一季度全国房价仍逆势上扬。有人断言:土地上涨势房价猛涨的罪魁。以 下哪项如果为真,最能对断言提出质疑?

A.2005 年第一季度上海房价比去年同期增长19.1% B.2005 年第一季度北京住宅价格比去年同期增长7.2%,住宅用地增长了0.37% C.华远地产董事长认为,随着土地开发成本的提高,房价一定会增加。D.永泰开发公司董事长说:“房价的暴涨是因为供应量没有跟上需求。”

5.经济的良性循环是指,不过分依靠政府的投资,靠自身的力量来实现社会总供给和社会总 需求的基本平衡,实现经济增长。近几年,我国之所以会出现经济稳定增长的态势,是靠政 府加大投资实现的。如果以上陈述为真,最能支持以下哪项结论? A.只靠经济自身所产生的投资势头和消费势头就能实现经济的良性循环。B.经济的良性循环是实现社会总供给与总需求的基本平衡的先决条件。C.某一时期的经济稳定增长不意味着这一时期的经济已经转入良性循环。D.近年来,我国的经济增长率一直保持在7%以上。

6.文化体现在一个人如何对待自己,对待他人,对待自己所处的自然环境。在一个文化环境 厚实的社会里,人懂得尊重自己——他不苟且,不苟且才有品味;人懂得尊重别人——他不 霸道,不霸道才有道德;人懂得尊重自然——他不掠夺,不掠夺才有永续的生命。下面哪一 项不能从上面这句话中推出? A.如果一个人苟且,则他无品味。B.如果一个人霸道,则他无道德。

C.如果人类掠夺自然,则不会有永续的生命。D.如果一个人无道德,则他霸道并且苟且。

7.一位编辑正在考虑报纸理论版稿件的取舍问题。由E、F、G、J、K 六篇论文可供选择。考虑到文章的内容、报纸的版面等因素。

(1)如果采用论文E,那么不能用论文F 但要用论文K;(2)只有不用论文J,才能用论文G 或论文H。(3)如果不用论文G,那也不用论文K。(4)论文E 是向名人约的稿件,不能不用。以上各项如果为真,下面那一项一定时真的? A. 采用论文E,但不用论文H。B. G 和H 两篇文章都用。C. 不用论文J,但用论文K。D. G 和J 两篇文章都不用。

8.积极的财政政策用发国债的办法来弥补财政赤字,旧债到期了,本息要还,发行的新债中 有一部分要来还旧债。随着时间的推移,旧债越来越多,新债中用来还旧债的也越来越多,用来投资的就越来越少,经济效益就越来越差。以下哪项陈述是以上论证所依赖的假设? A.积极的财政政策所产生的经济效益是递减的。

B.积极的财政政策所筹集的资金只能用于基础设施的建设。C.用发国债的办法来弥补财政赤字的做法不能长期使用。D.国债在到期之前,其投资回报不足以用来偿还债务。

9.历史的真实不等于真实的历史鲁迅说《史记》是“史家之绝唱,无韵之离骚”。好的史学 作品必须突破那层僵化的历史真实观,直接触及到历史人物的灵魂,写出历史的本质真实来。以下哪项陈述是上述论证所依赖的假设?

A.好的史学作品既忠实地报导历史事实,又生动的刻画人物的灵魂。B.仅仅忠实地记述历史事实的史学作品不是好的史学作品。C.在所有史学作品中,只有《史记》是好的史学作品。

D.只是生动刻画历史人物灵魂,没有报导历史事实的作品不是史学作品。

10.如果你要开办自己的公司,你必须在一件事情上,让人知道你很棒,比如你的产品比别 人做的好;别人也做得一样好时,你比别人快;别人也同样快时,你别别人成本低,别人的 成本也一样低时,你比别人附加值高。下面哪一项最不接近上面这段话的意思? A.只有至少在一件事情上做得最好,你的公司才能够在市场竞争中站稳脚跟。B.如果你的公司在任何事情上都不是最好,它就很可能在市场竞争中败下阵来。C.如果你的公司至少在一件事情上做得最好,它就一定能获得巨额利润。

D.除非你的公司至少在一件事情上做得最好,否则,它就不能在市场竞争中获得成功。11.关于一项重要的实验结果的报告是有争议的,在某科学家的指导下重复了这项实验,但 没有得到与最初实验相同的结果;该位科学家由此得出结论:最初的实验结果是由错误的测 量方法造成的。以下哪项是这位科学家推理的假设?

A.如果实验一项的结果是正确的,那么,在相同条件下进行实验应得到相同的结果。B.由于没有足够详细地记录最初的实验,所以,不大可能完全重复这一实验。C.重复实验不会像最初实验那样由于错误的测量方法而导致有问题的结果。D.最初的实验结果使得某个理论原则受到质疑,而该原则本身的根据是不充分的。12.肯定有一个外部世界存在,因为如果不是在我之外有某种东西可以发光或反光,将光照 射到我眼睛里,使我产生了视觉经验,我就看不到建筑。人群和星星这些东西。并且,不仅 有我这样的视觉经验,他人也有这样的视觉经验;书本知识也反复告诉我们,在我们之外有 一个外部世界。下面哪一项不够成对上述论证的怀疑或反驳?

A.用感官证据说明外部世界的存在,需要在心灵中预先假定外部世界的存在。B.你如何证明他人与你有类似的视觉经验?

C.既然视觉经验是可靠的,海市蜃楼不是所谓的幻觉,而是真实的存在。

D.如果没有一个外部世界的存在,自然科学知识不是对它的真实反映,那么,自然科 学为什么会在实践中获得如此巨大的成功呢?

13.要么采取紧缩的财政政策,要么采却扩张的财政政策,由于紧缩的财政政策会导致更多 的人下岗,所以,必须采取扩张的财政政策。以下哪一问题,对评论上述论证最重要? A.紧缩的财政是否还有其它不利影响?

B.既不是紧缩的也不是扩张的财政政策是否存在? C.扩张的财政政策能否使就业率有大幅度的提高? D.扩张的财政政策是否能导致其它的不利后果?

14.陕西出土的秦始皇兵马俑,其表面涂有生漆和影烩。这位研究秦代军人的服色提供了重 要信息。但兵马俑出土后,表面的生漆会很快发生翘和卷曲,造成整个彩绘层脱落,因此,必须用防护液和单体渗透两套方法进行保护,否则不能供研究使用。而一旦采用这两套方法 对兵马俑进行保护,就会破环研究者可能从中获得的有关秦代彩绘技术的全部信息。如果以上陈述为真,以下哪项必然为真?

A.采取保护措施后的秦兵马俑只能提供秦代军人服色方面的信息。

B.一个供研究秦代军人服色的兵马俑,不能成为了解秦代彩绘技术的新信息的来源。C.秦兵马俑是了解秦代彩绘技术的唯一信息来源。

D.一个没有采取保护措施的兵马俑能够比采取保护措施后的兵马俑提供更多信息。15.某城市的房地产开发商只能通过向银行直接贷款或者通过预售商品房来来筹集更多的开 发资金。政府不允许银行增加对房地产业的直接贷款,结果使得该市的房地产开发商无法筹 集到更多的开发资金,因为_______ 以下哪个选项能够合逻辑完成上述论证?

A.有的房地产开发商预售商品房手携款潜逃,使得工程竣工遥遥无期。B.中央银行取消了商品房预售制度。C.建筑施工企业不愿意垫资施工。

D.部分开发商销售起房后延期交房,使得很多购房者对开发商心存疑惑。

16.如果不设法提供低收入者的收入,社会就不稳定;假如不让民营经济者得到回报,经济 就上不去。面对收入与分配的两难境地,倡导“效率优先,兼顾公平”是正确的,如果听信 “公平优先,兼顾效率”的主张,我国的经济就会到“既无效率,又无公平”的年代。以下哪项陈述是上述论证所依赖的假设? A.当前社会的最大问题是收入与分配的两难问题

B.在收入与分配的两难境地之间,还有第三条平衡的道路可走。

C.“效率与公平并重”优于“效率优先,兼顾公平”和“公平优先,兼顾效率” D.倡导“效率优先,兼顾公平”不会使经济回到“既无效率,又不公平”的年代。17.大学作为教育事业,属于经济行业,其产出难以用货币指示、实物指标测定,故大学排 名不想企业排名那样容易。大学排名还必须以成熟的市场经济体制,稳定的制度为前提,必 须有公认的公证排名机构等。在我国,大学排名的前提条件远不具备,公认的大学排名机构 还未产生。因此,我国目前不宜进行大学排名。以下哪一项不能构成对上述论证的反驳? A.大学排名对学校声誉考生报考有很大影响。

B.大学排名与成熟的市场经济制度之间没有那么紧密的关系。C.企业排名也不容易,并且也不尽准确,仅供参考价值。D.公认的排名机构只能从排名实践中产生。18—19 题基于以下题干:

研究人员发现,每天食用五份以上的山药、玉米、胡萝卜、洋葱或其它类似蔬菜可以降低 患胰腺癌的风险。他们调查了2230 名受访者,其中有532 名胰腺癌患者,然后对癌症患症 的农产品加以分类,并询问他们其它的生活习惯,比如总体饮食和吸烟情况,将其与灵外 1701 的生活习惯作比较。结果发现,每天至少食用五份蔬菜的人患胰腺癌的几率是每天食 用蔬菜两份以下人的一半。

18.以下哪一个问题不构成对上述研究结论可靠性的质疑? A.受访者在调查中所说的话都是真的吗? B.在胰腺癌患者中,男女各占多大比例?

C.调查所涉及的胰腺癌患者与非胰腺癌患者在生活习惯方面的差异是否有重要遗漏? D.胰腺癌患者有没有遗传方面的原因?

19.以下哪一项办法最有助于证明上述研究结论的可靠性?

A.查明在肉食为主,很少使用以上蔬菜的群体中胰腺癌患者的比例有多大。B.研究胰腺癌患者中有那些临床标现及其治疗方法。C.尽可能让胰腺癌患者生活愉快,以延长他们的寿命。D.通过实验室研究,查明上述蔬菜中含有哪些成分。

20.在国内,私有化的概念说起来好像就是把国有资产分掉。而实质上则是对私有财产所有 权的保护问题。如果没有对这个权利进行保护的法律基础,国有资产能够被分掉,分得的财 产也随时可以被没收。

如果以上上述为真,最有力支持以下哪项陈述?

A.如果没有私有财产可以保护,保护私有财产的法律就毫无意义。B.即使有保护私有财产的法律,不能有效的执行也无济于事。C.私有化的制度是建立在拥有私有财产的合法权利的基础上的。D.私有化和市场自由化是社会主义市场经济都应该重视的问题。

21.淮州市的发展前景不容乐观,它的发展依赖于工业,工业为居民提供岗位和工资,而它 的自然环境则取决于消除工业污染,工业污染危及到它的空气、水和建筑。不幸的是,它的 工业不可避免的产生污染。

如果以上所说都是真的,则它们最有力的支持下面哪项陈述? A.淮州的生活质量只取决于它的经济增长和自然生存环境。B.淮州市一定会遇到经济发展停滞或自然环境恶化的问题 C.近年来淮州的经济环境已经恶化 D.淮州市空气、水和建筑物的污染主要是化工企业造成的。22—23 题基于以下题干

对于上市公司而言,有分红的企业才能发行新的股票。可是,如果一个企业有分红,那它 就不需要资金。如果它需要融资,就没有办法分红。22.如果以上陈述为真,一下哪项陈述不可能假? A.一个上市公司不需要融资,或者不是有分红的企业。B.一个上市公司需要融资,或者是没有分红的企业。

C.一个上市公司不需要发行新股票,或者不是有分红的企业。D.一个上市公司融资的唯一渠道是发行新股票。23.如果以上陈述为真,一下哪项陈述不可能为真? A.一个上市公司需要融资,而且没有办法分红。B.一个上市公司不是需要融资,就是没有办法分红。C.一个上市公司不需要融资,就一定会分红。D.一个上市公司及需要融资,也有办法分红。

24.对中国31 个省市自治区的商人信任度的调查结果表明,一半本地人都认为本地人值得信

任。如北京人为北京人打出的可信任度分数是57.9。而为天津人打出的分数时15,有一个 地方例外,就是海南人自己并不信赖海南人。

如果以下陈述为真,除了那项之外,都能对上述的例外提供合理解释? A.海南本来就骗子多,互不信任。

B.海南绝大多数的被抽查者是是从外地去那里经商留下的。C.外地人对海南商人不了解,给他们打得信任分数很低。D.在海南经商的大多数商人不是本地人。

25.历史并非清白之手编制的网,使人堕落和道德沦丧的一切原因中,权利是最永恒,最活 跃的。因此,应该设计出一些制度,限制和防范权力的滥用。下面那个假设能够给予上述推理最强的支持? A.应该设法避免使人堕落和道德沦丧。B.权利常常使人堕落和道德沦丧。

C.没有权利的人就没有机会在道德上堕落。D.一些堕落和道德沦丧的人通常拥有很大的权力。

26.资本的特性是追求利润。2004 年上半年我国物价上涨的幅度超过了银行存款的利率。1 —7 月份,居民收入持续增加,但居民储蓄存款增幅持续下滑,7 月外流存款达1000 亿元左

右,同时定期存款在全部存款中的不忠不断下降。

以下哪项如果为真,最能够解释这达1000 亿元储蓄资金中大部分资金的流向? A.由于预期物价持续上涨,许国居民的资金只能存活期,以便随时购买自己所需的商。

B.由于预期银行利率降上调,许国居民的资金只能存活期,准备利率上调后改为定期。C.由于国家控制贷款规模,广大民营企业资金吃紧,民间借贷活跃,借贷利息一远远 高于银行存款利率。

D.由于银行存款利率大低,许多居民考虑是否买股票或是基金。

27.如果危机发生时,公司能够采取非常有效的办法来消除危机,实际上能够增加公司的声 誉。一个非常好的声誉,可能因为一个事件。转眼就被破坏;而一个不好的声誉,往往需 要很长的时间的努力才能消除它。

如果以上陈述为真,最能支持一下哪项陈述? A.破坏一个好声誉比消除一个不好的声誉更容易。B.如果声誉的风险不算风险的话,就不存在风险了。C.维持公司声誉是董事会最重要的职责。

D.消除一个不好的声誉比赢得一个好的声誉还难。

28.中国的历史上,一般都给官员比较低的薪水,这样皇帝好控制他,因为薪水低了以后,官员肯定要贪污。皇帝就可以抓住这个把柄,想治他就治他。如果薪水高了,官员不用贪污 的话,皇帝就没办法治他了。以下哪项是上述论证说依赖的假设? A.迫使官员贪污是皇帝控制官员最愚蠢的方法。B.迫使官员贪污是皇帝控制官员最廉价的方法。C.迫使官员贪污是皇帝控制官员的唯一方法。D.迫使官员贪污是皇帝控制官员最好用的方法。

29.在过去五年里,新商品房的平均价格每平方米增加了25%。在同期的平均家庭预算中,购买商品房的费用所占的比例保持不变。所以,在过去五年里,平均家庭预算也一定增加 了25%。

以下哪项关于过去五年情况的陈述是上面论述所依赖的假设? A.平均每个家庭所购买的新商品房的面积保持不变。

B.用于食品和子女教育方面的费用在每个家庭预算中所占的比例保持不变。C.全国范围内用来购买新商品房的费用的总量增加了25%。D.所有与住房有关的花费在每个家庭预算中所占的比例保持不变。

30.我可以设身处地的把一些外在符号跟一些内心时间关联起来,比如,将呻吟和脸的扭曲、跟痛的感受关联起来。我从痛的体验中得知,当我有痛感时,往往就会呻吟和脸扭曲。因此 一旦我看到他人有相同的外在符号时,我就是理所当然的认为,他们也有与我相同的内心活 动事件。毕竟我和他人之间,在行为举止和通常的生理功能方面,显然是相类似的,为什么 在内心活动方面不也像类似呢?

下面那一项能够最有力地支持上面的论证? A.相似的结果一定有相似的原因。

B.痛感与呻吟和脸扭曲之间可能有密切联系。C.行为举止与内心活动也许有某种内在关联。D.人与人之间很多方面都是相似的。

31.在一个物质过剩的世界里却有人因为物质短缺而死去,这种道德上令人厌恶和智力上的 荒谬愚蠢,令我感到震惊和羞耻。下面哪一项是上面这句话的明显含义?

A.在一个物质过剩的世界里,有人因为物质短缺而死去,是因为他过于懒惰。B.在一个物质过剩的世界里,有人因为物质短缺而死去,是因为他愚蠢。

C.从道德和智力这两个层面而言,我们本来应该设计出一种分配制度,以便让每个社 会成员都过上体面而有尊严的生活。

D.在一个物质过剩的世界里,有人因为物质短缺而死去,是因为他在性格上有缺陷。32.企业的职工所赚的钱是计入成本的工资,老板赚的钱是不计入成本的利润。成本若搞利 润就低了,利润若高成本就低了。

如果以上陈述为真,最有力地支持以下哪项结论? A.职工持有企业的股份,并且与老板在利益上有矛盾。

B.如果职工持有企业的股份,老板与职工在利益上就没有矛盾。C.如果职工没有本企业的股份,老板与职工就有利益上的矛盾。D.老板赚的钱总是多于职工所赚的钱。

33.过去所有企业的问题都是这样一个问题;“我应该干什么,干什么我能赚钱?”加入WTO 之后,这个问题则变成:“干什么我能成为最好的?” 以下哪项陈述对上述问题的转变给出了最合理的解释? A.问题的转变意味着从企业的竞争过渡到了行业的竞争。B.企业只有成为最好的,才能赚大钱。

C.只有从事赚钱的行业,才能把企业做到最好。D.过去由行业的好坏之分,现在只有七业的好坏之分。

34.假如我和你辩论,我们之间能够分出真假对错吗?我和你都不知道,而所有其他的人都 有成见,我们请谁来评判?请与你观点相同的人来评判。他既然与你观点相同,怎么能评判? 请与我观点相同的人来评判,他既然与我观点相同,怎么能评判?请与你我观点都不相同的 人来评判,他既然与你我的观点都不相同,怎么能评判?所以,“辩无胜”。下面哪一项最准确的描述了上述论证的缺陷?

A.上述论证严重忽视了有超出辩论者和评论者之外的实施标准和逻辑标准。B.上述论证有“混淆概念”的逻辑错误。C.上述论证中的理由不真实,并且相互不一致。D.上述论证犯有“文不对题”的逻辑错误。

35.公司智力取决于立法者所制定的法律。然而,仅有法律是不够的,还必须依赖位管理者 制定的最有行动准则。比如,“公司的董事应该具有卓越的才能”这条准则,对于什么是“卓 越的才能”,法律不能给出它的标准定义。最有行动准则的优势就是它采纳弹性比较大的标 准。

以下哪项陈述时上属论证所依赖的假设?

A.只有当法律能够实施的时候,法律才会有作用。B.采纳弹性比较小的标准不能发挥最优行动准则的优势。C.采纳弹性比较大的标准制定法律会给法律的实施带来麻烦。D.即时只能发挥最有行动准则的优势,法律还是不能缺少的。

36.今天的巴黎雍容美丽一如以往,因为占领他的德国指挥官在接到希特勒“撤退前彻底毁 掉巴黎”的命令时,决定抗命不从,以自己的生命为代价保住一座古城。梁漱溟在日本军用 飞机的炮弹在身边炸开时,静坐落园中,继续读书,思索东西文化和教育的问题。—对“价 值”和“秩序”有所坚持,对破坏这种“价值”和“秩序”有所抵抗,就是文化。下面的哪一项不能由题干的意思推出? A.能够识字读书不等于有文化。B.具有广博的知识不等于有文化。

C.文化意味着有所为,有所不为:一点不忍的念头,是生民生物之根芽;一段不为 的气节,是撑天撑地之柱石。

D.文化是不同民族不同个人相互区别的标志。

37.企业竞争以效率为根本,而效率是以亲情为核心的东西。我国的各种制度不是要破坏亲 情,而是要把亲情发挥到最高点。

如果以下陈述为真,哪一项将最严重地消弱上述结论?

A.亲情不但能建立在私德的基础之上,也能建立在公德的基础之上。B.制度的主要作用是谈话亲情,防止人们利用亲情不干好的事情。C.亲情能给企业带来效率,一旦反目成仇也能给企业带来灾难。D.制度虽然能激发亲情,但制度本人却不容半点亲情。

我国个人所得税法修正案(草案)将工薪所得的费用扣除标准由原来的800 元/月提高到

1500 元/月;当人月收入低于1500 元时,无需纳税;高于1500 元时对减去1500 元后的收入

征税,一位官员对此评论说:个人所得税起征点不宜太高,因为纳税也是公民的权利,起点 太高就剥夺了低收入者最为纳税人的荣誉。

以下哪项为真,能对这位官员的论点提出最大质疑?

A.世界各国在征收个人所得税时,都是将居民基本生活费用予以税前扣除,以保证 社会劳动力的再生产。

B.个人多得税交的少也会影响低收入者最为纳税人的荣誉。C.个人所得税的作用之一是调节社会分配,缩小贫富差距。

D.中国得税制以商品为主,一个人只要购买并消费商品,就向国家交了税。39—40 题基于以下题干:

一个人到底是做出好的行为还是作出坏的行为,跟他生命的长短有关。如果他只活一天的 话,他去偷人家东西是最好的,因为他不会遭受担心被抓住的痛苦。对于还能活20 年的人 来说偷人家东西就不是最好的,因为他会遭受担心被抓住的痛苦。39 如果以下各项陈述为真,除了那项外,都能削弱上述论证? A.只有遭受担心被抓住的痛苦,才不会去偷人家东西。B.对于只活一天的人来说,最好的行为可能是饱餐一顿牛肉。C.生命的长短不是一个人选择做出好行为或坏行为的充分条件。

D.对于某些偷人家东西的人来说,良心的谴责会造成比担心抓住更大的痛苦。40.以下哪项陈述时上述论证所依赖的假设?

A.一个人在决定是否去偷人家东西之前,能确切的知道他还能活多久。B.凡是去偷人家东西的人都活不了几天。

C.只要没有被抓住,担心被抓住不会给人带来痛苦,因为偷东西的人早有思想准备。D.一个知道自己活不了几天的人,通常会选择做些好事而不是去做坏事。41—45 题基于以下题干:

印刷组和装订组的成员主要来自以下七名员工——F,G,H,J,K,L 和M,每个组的成员必须满 足下列条件:(1)每个组至少有三名员工。(2)F 和K 不能在同一组。

(3)如果K 在某个组,J 也必须在这个组。(4)M 至少是这连个组中的成员之一。(5)两个组只要有一个相同的员工。

41.下列哪项列出的是这两个组可接受的成员选择? A.印刷:F,G,H 装订:G,J,K,L B.印刷: F,H,J 装订:G,H,L,M C.印刷: F,H,M 装订:G,K,L,M D.印刷: F,H,J,K 装订:H,L,M 42.如果印刷组的成员有F,H,L 和M 组成,而装订组的成员由G,H 和J 组成,那么K 可以替换两组的那一个成员而不违反任何给出的条件? A.F B.G C.H D.L 43.如果印刷组只有G,H 和L 三个成员,而在条件充裕的情况下,两个组有尽可能多的共 同成员,那么以下哪项陈述一定是真? A.装订组合印刷组一定有一个共同成员。B.装订组合印刷组恰好有两个共同成员。C.装订组合印刷组成员的数量相同。

D.装订组成员的数量至少比印刷组的多一个。

44.如果K 和L 都是身兼两组的成员,而且印刷组正好有三个成员,那么以下哪项陈述一定 为真?

A.F 在印刷组B。F 在装订组 C.G 在印刷组D。M 在装订组

45.最多有几名员工可以兼印刷组和装订组的成员? A.四B。五 C.六D。七

46——50 基于以下题干: 在一次魔术表演中,从七位魔术师——G.H.K.L.N.P 和Q 中,选择六位上场表演,表演时 分成两队:1 队和2 队每队由前中后三个位置,上场的魔术师恰好每人占个位置,魔术师的

选择和位置安排必须符合下列条件:

(1)如果安排G 或H 上场,他们必须在前位。(2)如果安排K 上场,他必须在中位。(3)如果安排L 上场,他必须在1 队。(4)P 和K 都不能与N 在同一个队。(5)P 不能与Q 在同一个队。

(6)如果H 在2 队,则Q 在1 队的中位。46.以下哪项列出的是2 队上场表演可接受的安排? A.前:H 中:P 后:K B.前:H 中:L 后:N C.前:G 中:Q 后:P D.前:G 中:Q 后:N 47.如果H 在2 队,那项列出的是1 队可以接受的安排? A.前:L 中:Q 后:N B.前:G 中:K 后:N C.前:L 中:Q 后:G D.前:G 中:K 后:L 48.以下哪项烈处的是1 队上场表演可以接受的安排? A.前:Q 中:L 后:N B.前:L 中:K 后:Q C.前:Q 中:K 后:P D.前:G 中:K 后:P 49.如果G 在1 队,以下哪一对魔术师可以在1 队? A.K 和L B.K 和P C.L 和N D.L 和Q 50.如果G 在1 队并借K 在2 队,下列哪个魔术师一定在2 队的后位? A.L B.N C.P D.Q

第三部分2005 年GCT 逻辑试题参考答案(以下答案仅供参考)1C 2D 3C 4B 5C 6D 7C 8D 9B 10C 11A 12D 13D 14D 15B 16D 17A 18B 19D 20C 21B 22A 23D 24C 25B 26C 27A 28C 29A 30A 31C 32C 33D 34A 35B 36D 37B 38D 39A 40A 41B 42B 43D 44D 45C 46D 47A 48A 49D 50C 第四部分 外语运用能力测试(英语)(50题,每题2分,满分100分)Part One Vocabulary and Structure Directions: There are ten incomplete sentences in this part.For each sentences there are four choices marked A, B, C and D.Choose the one that best completes sentence.Mark your answer on the ANSWER SHEET with a single line though the center.1.To speed _______ your entry, please bring your Admission Card with you.A.up B.on C.cut D.down 2.More thunderstorms _______ in summer than any other time of the year.A.happened B.have happened C.happen D.will happen 3.Experts say the space rock is probably _______ more than U.S.$30,000.A.weighty B.costly C.valuable D.worth.4.Susan will come to watch him _______ at Wimbledon this week.A.played B.play C.to play D.playing 5.Children don’t _______ understand what are reciting, but gradually it will have in impact on thinking.A.played B.play C.to play D.playing 6.Every year, thousands of college students apply for the CCTV Cup English Speech _______.A.Argument B.Quarrel C.Debate D.Contest 7.________ shall we forget the day when we received the admission into Harvard University.A.No time B.Never C.No sooner D.Nonetheless 8.The newly released movie was ________ as to arouse so much sensation among the young people.A.No time B.Never C.No sooner D.Nonetheless 9.Today’s popular clothing chains ________ teenagers, who can be counted upon to change their tastes every 30 days.A.resort to B.attend to C.appeal to D.apply to 10.There is going to be _______ time for people to assess whether or not we have made the right decision in this time of urgency.A.sufficient B.additional C.efficient D.consequent Part Two Reading Comprehension Directions: In this part there are three passages and one advertisement, each followed questions or unfinished statements.For each of them, there are four choices marked A, B, C and D.Choose the best one and.Mark your answer on the ANSWER SHEET with a single line though the center.Questions 11—15 are based on the following passage: The first ancient Olympics were held in 776 B.C.The games got their name from Olympia, the Greek city where they took place.Like the summer Olympics of today, the ancient Olympics were held every four years.Thousands of people from all over the Greek world came to watch.The main stadium held about 45,000 people.“We have accounts of visitor and pilgrims setting up tents all around the site.” Lisa Cerrato of Tufts University said.During the first Olympics, there was only one competition—a 200-meter race.But over time the games grew to include wrestling, chariot racing, boxing, and other sports.Women were not allowed to compete, but they had their own separate games.“The ancient athlete became celebrities(名人), just like today.They often lived the rest of their lives being treated to free dinners.” Cerrato said, “City-states even tried to steal away each other’s athletes by offering them various awards.”

The ancient Olympics existed until A.D.393.But the modern Olympics are still going strong.11.Where did the ancient Greeks hold their first Olympics? A.In Athens B.In Olympics C.In a town D.In a state 12.How did researchers know that ancient Greeks rushed to watch the Olympics? A.Thousands of people came to watch.B.The main stadium is still not big enough.C.They have found the related record of events.D.Many of them were visitor and pilgrims.13.What did women do since they were not allowed to participate in the Olympics? A.They stayed at home doing nothing.B.They organized protests in the city.C.They had their own games.D.They set up tents around the site.14.The ancient athletes must have felt honored when they ___________.A.completed in a 200-meter race.B.wrestled with each other C.read the accounts of the games D.received the treatment of free dinners 15.It can be inferred from the fourth paragraph that __________.A.the ancient athletes liked to celebrate their victories B.free dinners were offered during the competition C.city-states competed with each other to win the Olympics D.awards were often stolen to honor the athletes

Questions 16—20 are based on the following passage: Why are mobiles so popular? Because people love to talk to each other.And it is easier with a mobile phone.In countries like Russia and China, people use mobile phone in places where there is no ordinary telephone.Business people use mobiles when they’re traveling.In some countries, like Japan, many people use their mobile phones to send e-mail message and access the Internet.They use a new kind of mobile phone called “i-mode”.You can even use a mobile phone listen to music.Mobile phones are very fashionable with teenagers.Parents buy mobile phones for their children.They can call home if they are in trouble and need help.So they feel safer.But teenagers mostly use them to keep in touch with their friends or play simple computer games.It’s cool to be the owner of a small expensive mobile.Research shows that teenage owners of mobile phone smoke less.Parents and schools are happy that teenagers are safer and smoke less.But many people dislike them.They hate it when the businessman opposite them on the train has a loud conversation on his phone.Or when mobile phones ring in a café or restaurant.But there is a much more serious problem.It’s possible that mobile phone can heat up the brain because we hold the phone so closed to our bead.Scientists fear that mobiles can perhaps be bad for your memory and even give you cancer.16.Mobile are popular among people because _______ A.they think mobile are fun B.they believe mobile are safe C.they love to listen to music D.they feel it easier to talk to each other 17.It is stated in the passage that Russia and China _______.A.ordinary telephone service is available everywhere B.people prefer mobile phone to ordinary telephones C.mobile are used in places without ordinary telephone D.people use mobiles to send e-mail and access the Internet 18.Parents buy mobile phone for their children because ________.A.mobile phone are very fashionable with teenagers B.they can call home if they are in trouble and need help C.they can use mobile to play simple computer games D.mobile phone enable teenagers to give up smoking 19.Why can mobile phone be a much more serious problem? A.Because people hate the businessmen using mobile in public places.B.Because of use of mobile phones can be physically harmful.C.Because there is strong evidence for the problems of mobiles.D.Because people feel their privacy unprotected in a café or restaurant.20.Which of the following can be the title of this passage? A.Why Are Mobile So Popular? B.Mobiles—Useful or Useless C.Mobile—A Danger to health? D.Why Do We Design Mobile? Questions 21—25 are based on the following passage: At the International Snow Sculpture Championships in Colorado on January 27, people will carve the white stuff into art.Planning begins months before the first snow falls.Teams submit applications and sketches of their sculptures in July.Then, a panel of judges choose 14 teams for the championship.The rules are simple: Electric tools are not allowed.Teams carve snow with everything but the results are not entirely in the sculptors’ hands,” if it is extremely sunny and warm.” DeWall, the competition’s director of public relations, explained, “we will erect old sail from sail boats into the air to block the sun from melting the sculptures.” If it snows, she continued, teams have to work extra hard to scrape(刮掉)the new snow off their work..The judges look for creativity, technical skill, and overall impact on the viewer.The winner does not receive any money.“There is no cash prize because the event began with the concept of global camaraderie(情谊).” DeWall explained.Instead of focusing on money, she continued, “winners revel(纵情)in the friendship, the art, and the hard work.”

21.What does the phrase “white stuff” in the first paragraph refer to ? A.Wood B.Snow C.Ice D.Rock 22.What do people start to plan for the Championships? A.After the first snow fall in Colorado.B.Once they submit their applications.C.Before their sculptures are sketched D.As soon as the judges have chosen the teams.23.The sculptors cannot control the result because ________.A.it is extremely and warm B.it shine and/or snows C.the sun melts their sculptures D.old sails are raised into the air 24.What is the viewer’s role in the championships? A.Sculptors like to work together with them.B.Judges consider their responses as choosing the winner.C.Viewers watch and help to product the sculptures.D.The organizers rate their involvement and enthusiasm.25.The winners enjoy all of the following EXCEPT ________.A.the cash prize B.the friendship C.the art D.the hard work Questions 26—30 are based on the following from New York Times’ electronic Classified Ads: JOB INFORMATION Job Title : Banking Employer : Confidential Source : NY Times Classified Ads Location : New York, NY Date : 09-16-2005 JOB DESCRIPTION Description: BANKING OPENING NEW BRANCHES We are growing domestic bank who seek exp’d staff for our new branches.Branch managers 2+ yrs exp in Br Mgmt, knwlg in all bkg products and regs, excellent Mgmt skills a must, sales and mktg skills.Tellers(F/T, P/T)Banking experience preferred.Customer Serv Reps(F/T, P/T)Banking background a must.Qualified candidates should e-mail resume to hr@doralbankny.com or fax to 212-329-3745.Doral Bank is EOE.For more details visit www.xiexiebang.com 26.Which of the following is NOT shown above? A.Job title B.Annual payment C.Ad source D.Job description 27.Based on what you read, the ad is intended for the __________.A.recruitment B.product promotion C.banking service D.banking reform 28.New staff are needed because of _______.A.the opening of the new branches B.the updating of the banking service C.the retirement of the experienced staff D.the lack of excellent management 29.Which of the following is NOT mentioned in the ad? A.Branch managers B.Tellers C.Customer Service Reps D.Sale managers 30.Qualified candidates are expected to _________.A.e-mail their resumes to hr@doralbankny.com B.visit Doral Bank in New York in person C.call 212-329-3745 for more information D.visit www.xiexiebang.com for a interview Part Three Cloze Directions: There are ten blanks in the following passage.For each numbered blank, there are four choices marked A, B, C and D.Choose the best one and.Mark your answer on the ANSWER SHEET with a single line though the center.Double Income and No Kids(DINK)becomes fashionable in China.The DINK couples are usually regarded as those who have higher educations and __31__ careers with higher incomes.The increase in DINK families has shattered the Chinese traditional idea of the family and __32__ typical.A survey conducted recently in Beijing by a market survey company __33__ that about 3.3 percent of 1,300 surveyed families in Beijing said they have __34__ plans to have children.It is estimated there are about 600,000 DINK families in large cities like Beijing, Tianjin, Shanghai, and Chongqing.Why they choose such a lifestyle is concluded in __35__ reasons.Some are showing great worry for the rapid growth of population;some are indulged in building a more well-off family;some are showing sharp __36__ to get themselves free from the obligation of raising children.__37__, most people still believe it is necessary to bear a child to keep the family line on.As an old Chinese saying goes, there are three aspects in failing to be a filial son and the __38__ serious one is to have no heir for the family.So childless couples will suffer discrimination __39__ family members and neighbors.But it is clear that the new tide of ideas has come, which suggests young people __40__ to choose their own way of life.They are installing modern ideas into traditional families and society.In the modernization process, personal choices will be highly respected.31.A.stable B.available C.achievable D.liable 32.A.had become B.may become C.became D.becomes 33.A.directed B.induced C.indicated D.dictated 34.A.no B.not C.hardly D.scarcely 35.A.elegant B.abundant C.similar D.various 36.A.tension B.attention C.intention D.interaction 37.A.Moreover B.However C.According D.Generally 38.A.most B.more C.latest D.less 39.A.into B.to C.at D.from 40.A.wanted B.should want C.want D.had wanted Part Four Dialogue Completion Directions: In this part, there are ten short incomplete dialogues between two speakers, each followed by four choices marked A, B, C and D.Choose the one that most appropriately suit the conversational context and best completes the dialogue.Mark your answer on the ANSWER SHEET with a single line through the center.41.Receptionist: Can I help you? Customer: ________.Where do I pay my fees? A.Yes, please B.Thank you C.As you please D.Yes, you can 42.Operator: _______, May I help you? David: I’d like to set up a telephone service, please.A.Speaking B.Hey C.Pacific Bell D.I’m John Smith

43.Cathleen: Let’s take a coffee break, shall we?

Yolanda: _______, but I can’t.A.We shall B.Yes, let’s C.You will D.I wish I could 44.Beth: You look a little pale.Are you OK? Jerry: _______, I feel terrible.A.To speak out B.To begin with C.To sum up D.To tell you the truth 45.Ken: Gee, Martin, I’d love a cup of coffee.Martin: ________, Is instant OK? A.Sure thing B.I’m sure C.No kidding D.Sure I do 46.Joanne: Hey, you look concerned._______ Harry: The final exam.I’m not fully prepared yet.A.What’s on your mind? B.What a lovely day!C.What has attracted you? D.What about seeing the doctor? 47.Johnson: I got home very late last night.I hope I didn’t disturb you.Landlady: ________.A.No, I heard a lot of noise.B.No, I didn’t hear a thing.C.Yes, I didn’t hear any noise.D.Yes, I didn’t hear you.48.Peter: What’s there to do at night.Clerk: There are clubs, concerts, players and so on._______!A.You make it.B.You find it.C.You manage it.D.You name it.49.Ray: _________.Where was I ? Brenda: You were talking about your trip to South Africa.A.Let’s back up.B.What did I hear.C.Let’s check in.D.What were you talking about.50.Edie: I thinking Professor Holt is smart and she’s really good teacher.Rosa: OK.I’ll try to get into her class.Edie: _______!A.You can’t miss it.B.Forget it.C.Mind you.D.You won’t be sorry.第四部分参考答案 1-5ACDBB 6-10 DBDCA 11-15 ACCDC 16-20 DCBBA 21-25 BABBA 26-30 BAADA 31-35 ADCAD 36-40 DBADC 41-45 ACDDA 46-50 ABDAD

第二篇:2011年GCT考试逻辑真题及答案

2011年GCT考试逻辑真题及答案 第三部分逻辑推理能力测试

(50题,每题2分,满分100分)

1. 有人说道:读万卷书不如行万里路,行万里路不如阅人无数,阅人无数不如名师指路,名师指路不如自己领悟。以下各项都构成对上述观点的质疑,除了 ..A.阅人无数自会遇到名师指路。

B.书中自有乾坤,读万卷书如同行万里路。

C.若无名师指路,仅凭读万卷书和行万里路,自己仍无法领悟。D.行万里咱,游历大千世界,必定阅人无数。

2. 爱因斯坦发表狭相对论时,有人问他:预计公众会有什么反正应?他答道:很简单,如果我的理论是正确的,那么,德国人会说我是德国人,法国人会说我是欧洲人,美国人会说我是世界公民;如果我的理论不正确,那么,美国人会说我是欧洲人,法国人会说我是德国人,德国人会说我是犹太人。如果爱因斯坦的话是真的,以下哪项陈述一定为真? A.有人会说爱因斯坦是德国人。B.有人会说爱因斯坦是世界公民。C.有人会说爱因斯坦是犹太人。D.法国人会说爱因斯坦是欧洲人。

3. 世界卫生组织报告说,全球每年有数百万人死于各种医疗事故。在任何一个国家的医院,医疗事故致死的概率不低于0.3%。因此,即使是癌症患者也不应当去医院治疗,因为去医院治疗会增加死亡的风险。为了评估上述论证,对以下哪个问题的回答最为重要?

A. 在因医疗事故死亡的癌症患者中,即使不遭遇医疗事故最终也会死于癌症的人占多大比例?

B.去医院治疗的癌症患者和不去医院治疗的癌症患者的死亡率分别是多少?

C.医疗事故致死的概率是否因医院管理水平的提高而正在下降? D. 患者能否通过自身的努力来减少医疗事故的发生?

4. 因对微博的无知,某局长和某主任在微博上汇露个人隐私,暴露其不道德的行为,受到有关部门的查处。有网友对他们的行为冷潮热讽,感慨到:知识改变命运,没有知识也改变命运。

以下哪项陈述最接近该网友所表达的意思? A. 无论是否有知识,都会改变命运。

B.“知识就是力量”这一说法过于夸张,实际上,权力和金钱才是力量。C.有知识导致命运由不好向好的方向改变,没有知识导致命运由好向坏的方向改变。

D.“命运”的本义就是先天注定,它不会因有无知识而改变。

5. 在大学里,许多温和宽厚的教师是老教师,但有些严肃且不讲情面的教师也是好教师,而所有好教师都有一个共同特点:他们都是学识渊博的人。如果以上陈述为真,以下哪项陈述一定为真? A.许多学识渊博的教师是温和宽厚的。B.有些学识渊博的教师是严肃且不讲情面的。C.所有学识渊博的教师都是好教师。D.有些学识渊博的教师不是好教师。

6. 《与贸易有关的知识产权协定》规定,不得仅仅因为成员国本国法律禁止某些发明的商业性实施就不授予那些发明专利权。

已知A国是《与贸易有关的知识产权协定》的成员国,以下哪项陈述与上述规定不一致? ...A.从A国法律禁止一项发明的商业性实施推不出不能授予该项发明专利权。B.从A国法律允许授予一项发明专利权推不出允许该项发明的商业性实施。C.在A国,一种窃听装置的商业性实施是被法律禁止的,因此不允许授予其专利权。

D.在A国,一种改进枪支瞄准的发明被授予了专利权,但该项发明的商业性实施被禁止。

7. 每一个一政客都不得不取悦他的选民。尽管马英九是一位诚实的人,但他也是一位政客。如果不偶尔说出一些含糊其辞的话,任何人都不能取悦他的选民。

以上陈述为真,以下哪项陈述一定为真? A.马英九不会说含精其辞的话。B.马英九会说出一些含糊其辞的话。C.说含糊其辞的政客是不实的人。D.有的诚实的人不是政客。

8. 具有能够让一个乐队特别是一流乐队反复进行排练的权威,这是一个优秀指挥家的标志。这种威望不是轻而易得的,一个指挥家必须通过赢得乐队对他所追求的艺术见解的尊重才能获得这种威望。在上文的论述过程中,作者预先假设了以下哪项陈述?

A.优秀的指挥家在与不同的乐队合作时,对同一首作品会有不同的艺术见解。

B.优秀的指挥家都是完美主义者,即使对一流乐队的表演,也们也从不满意。

C.如果优秀的指挥家认为附加的排练是必须的,一流乐队总是时刻准备加班排练。

D.即使一种艺术见解还没有被充分表现出来,一流乐队也能够领悟这种艺术见解的优点。

9.2010年,卫生部推出新的乳业国家标准,将原奶蛋白质含量由原来的2.95%降至2.8%。新标准不升反降,引发了一片质疑。某业内人士解释说,如果我们的牛奶检测标准把蛋白质含量定的太高,奶农为了达标就会往奶里添加提高蛋白质检测含量的东西,如三聚氰胺;2008年的三聚氰胺事件就说明原来的标准太高了。

以下哪项推理含有与该业内人士的推理相同的逻辑错误?

A.真正的强者都不惧怕困难或挑战,赵涛害怕困难或挑战,说明赵涛不是真正的强者。

B.如果台风在海口登陆,飞往海口的航班就会被取消。现在飞海口的航班没有被取消,说明台风没有在海口登陆。

C.要是铁路部门的管理存在漏洞,铁路运输就会出事故。7.23温州动车事故说明铁路部门的管理存在漏洞。D.仅当人们信任一个慈善机构时才会向该机构捐款,所以,得不到捐款的慈善机构一定是丧失了公众的信任。

10.现代社会中有很多人发胖,长有啤酒肚,体重严重超标,因为他们常常喝啤酒。

对以下各项问题的回答都可能质疑上述论证,除了 ..E.如果人们每天只喝啤酒,吃很少的其他食物,特别是肉食品,他们还会发胖吗?

F.为什么美国有很多女人和孩子常喝可乐,吃炸鸡和匹萨饼,其体重也严重超标?

G.发胖的人除常喝酒外,是否经常进行体育锻炼?

H.很多发胖的人也同时抽烟,能够说“抽烟导致发胖”吗?.经济学家:中国外汇储备在过去10年的快速增长是中国经济成功的标志之一。没有外汇储备的增长,就没有中国目前的国际影响力。但是,不进行外汇储备投资,就不会有外汇储备的增长。外汇储备投资面临风险是正常的,只要投资寻求收益,就要承担风险。

以下哪项陈述能从这位经济学家的论述中合逻辑地推出? I.如果能够承担风险,就会有外汇储备的增长。J.如果不进行外汇储备投资,就不用承担风险。K.只要进行外汇储备投资,中国就能具有国际影响力。L.中国具有目前的国际影响力,是因为中国承担了投资风险。12.山西醋产业协会某前副会长称,在市面上销售的山西老陈醋中,只有5%是不加添加剂的真正意义上的山西老陈醋。中国调味协会某副会长就此事件接受记者采访时说:“只要是按照国家标准加添加剂,都没有安全问题。有些企业强调自己未加添加剂,这对按正常标准加添加剂的企来说是不公平的。”

以下哪项陈述能够从该调味品协会副会长的话中合逻辑的推出? A.为了保证公平性,企业或者不应该生产高于国家标准的产品,或者要对产品质量高于国家标准的事实秘而不宣。

B.要想促进行业的技术创新,就应当提高行业的国家标准。C.某个行业的国家标准定得太高,不利于该行业的良性发展。D.如果不按国家标准加添加剂,就会有安全问题。

13.午夜时分,小约翰安静的坐着。他非常希望此时是早晨,这样他就可以出去踢球了。

他平心静气,祈祷太阳早点升起来。在他祈祷的时候,天慢慢变亮了。他继续祈祷。太阳逐渐冒出地平线,升上天空。小约翰想了想所发生的事情,得出这样的结论:如果他祈祷的话,他就能够把寒冷而孤寂的夜晚变成温暖而明朗的白天。他为自己感到自豪。

下面哪项陈述最恰当地指明了小约翰推理中的缺陷? A.小约翰只是个孩子,他懂得很少很少。B.太阳环绕地球运转,不管他祈祷还是不祈祷。

C.一件事情在他祈祷之后发生,并不意味着因为他祈祷而发生。D.他有什么证据表明:如果他不祈祷,该事情就不会发生?

14.借债者的信用等级越低,其借债的成本越高,债主的收益率也越高。2011年8月5日,标曾将美国主权信用评级从AAA降为AA+。按照常理,投资者出于避险需求应当减持美国国债,从而推动其价格走低,收益率上升。然而,8月8日美国国债价格上涨,收益率除低,最新10年期美国国债收益率降至2.34%,创今年以来新低。

如果以下哪项陈述为真,能够最好地解释上述违反常理的理象? A.8月5日晚间,美国财政部回应称,标普进行评级计算时有2万亿美元的误差。

B.与欧盟、日本以及其他国家的债券相比,美国国债依然是相对安全和流动性最高的投资品。

C.美国主权信用评级下调后,俄罗斯开始减持美国国债,而中国仍增持美国国债。

D.欧盟为打破标普等三家美国评级机构的垄断地位,计划建立一家欧洲的信用评级机构。

15.在《反省的生命》一书中,诺齐克写到:“我不会像苏格拉底一样,说未经反省的生命是不值得过的——那是过分严苛了。但是,如果我们的人生是由深思熟虑的反省所引导,那么,它就是我们为自己活的生命,而不是别人的。从这个意义上说,未经反省的生命是不完整的生命。” 以下各项都能从诺齐克的陈述中推出,除了 ..A.诺齐克认为,值得过的生命是经过反省的生命。B.诺齐克认为,只有为自己活的生命才是完整的生命。C.诺齐克认为,完整的生命都是经过反省的生命 D.诺齐克认为,未经反省的生命不是完整的生命。16.中国人仇富,居然有那么多人为骗子说话,只因为他们骗的是富人。我敢断定,那些骂富人的人,每天都在梦想成为富人。如果他们有机会成为富人,未必就比他们所骂的人干净。况且,并非所有的富人都为富不仁,至少我周围的富人不是,我看到他们辛勤工作且有慈悲心怀。——有网友对达芬奇家具造假事件的网上评论如是说

根据该网友的说法,不能合逻辑地确定以下哪项陈述的真假? ..A.有的仇富者是中国人。B.有的富人并非为富不仁。

C.那些每天都在梦想成为富人的人却在骂富人。D.有的辛勤工作且有慈悲心怀的人是富人。

17.目前我国有3种转基因水稻正等待商业化种植审批,每种至少涉及5-12项国外专利;有5种转基因水稻正处于研发过程中,每种至少涉及10项国外专利。有专家认为,水稻是我国的主要粮食作物,如果我国允许转基因水稻商业化种植,国家对主要粮食作物的控制权就可能受到威胁。如果以下哪项陈述为真,将最有力地支持该专家的观点?

A.转基因水稻的优势在于抵抗特定的害虫,但我国的水稻很少有这些害虫。

B.目前还没有任何一种转基因水稻能超过我们超级稻、杂交稻等品种的产量和品质。

C.美国引入转基因种子后,玉米、棉花、大豆等种子的价格大幅长涨。D.如果我国商业化种植转基因水稻,国外专利持有人就会禁止我国农民保留种子,迫使他们每个播种季都高价购买种子。18.孔子非常懂得饮食和养生的道理,《论语·乡党》就列出了很多“食”和“不食”的主张,比如“不时不食”,意思是说不要吃反季节蔬菜。以下哪项陈述是上述解释所必须依赖的假设? A.孔子在饮食方面的要求很高。

B.孔子生活的时代既有当季蔬菜,也有反季节蔬菜。C.我们可以选择吃当季蔬菜,还是吃反季节蔬菜。D.饮食不仅滋养人的身体,还塑造人的心灵。

19.在众声喧哗中,尽可能打捞那些沉没的声音,是社会管理者应尽之责。以政府之力,维护弱势人群的表达权,使他们的利益能够通过制度化规范化渠道正常表达,这是构建和谐社会的关键所在。只有这样,才能让“说话”、“发声”不仅是表达诉求的基本手段,更成为培育健康社会心态的重要环节,成为社会长治久安的坚实基础。

如果以下哪项陈述为真,将最有力地支持上述论证? A.有些弱势者缺医少药,甚至得不到温饱。

B.弱势者是社会中“沉默的大多数”,一旦真正发怒,其力量足以颠覆整个国家机器。

C.有些弱势者的诉求长期得不到表达和满足,容易患各种心理疾病。D.甚至像美国这样的国家,也有很多食不果腹、居无定所的弱势者。20.天津许云鹤一案:许云鹤在驾车行驶途中,遇王老太翻越马路中心护栏,后王老太倒地受伤,许把她扶起。王老太坚称是许将她撞伤。经司法鉴定,许的车没有碰到她。法院一审认为,许见到王老太时离她仅四、五米远,她摔倒定然是由于受到许的车惊吓,判决许承担40%的责任,赔偿王老太108606元。以下哪项陈述最好地指出了该案一审中的纰漏之处?

A.法官推理悖于常理:王老太违章翻越马路护栏,应该预期机动车道上有车,而受惊吓通常是看到未预期的事物。

B.该判决违背传统道德:像南京彭宇案一样,会使大众不敢向受伤者伸出援助之手。

C.该判决没有弄清事实,证据一严重不足,有可能王老太先摔倒,许才开车过来。

D.该判快颠倒了原因和结困。

21.英国医生约翰·斯诺的“污水理论”开启了流行病学研究的历史。1854年,伦敦爆发了大规模的霍乱,约翰·斯诺发现,大多数死亡病例都曾经饮用同一个水泵汲取的水,而使用其他水泵或水井的人最初都没有感染霍乱。后经调查,下水道的废水汅染了那个水泵,从而引发了霍乱。以下哪一选项是约翰斯诺的推理没有运用的方法或原则 ..A.在被研究现象出现的各个场合都存在的因素很可能是该现象的原因。B.在被研究现象不出现的各个场合都不出现的因素很可能不是该现象的原因。

C.当被研究现象变化时,同步发生最变的那个因素很可能是该现象的原因。

D.在被研究现象出现的场合与该现象不出现的场合之间的差异很可能是该现象的原因。22.尼禄是公元一世纪的罗马皇帝。每一位罗马皇帝都喝葡萄酒,且只用锡壶和锡高脚酒杯喝酒。无论是谁,只要使用锡器皿云饮酒,哪怕只用过一次,也会导致中毒。而中毒总是导致精神错乱。

如果以上陈述都是真的,以下哪项陈述一定为真? A.那些精神错乱的人至少用过一次锡器皿云饮葡萄酒。B.不管他别的方面怎以样,尼禄皇帝肯定是精神错乱的。C.使用锡器皿是罗马皇帝的特权。

D.在罗马王朝的臣民中,中毒是一种常见现象。

23.巴勒斯坦准备在2011年9月申请加入联合国,已经争取到140个国家的支持。如果美国在安理会动用否决权,阻止巴勒斯坦进入联合国,会在整个阿拉伯世界引燃反美情绪;如果美国不动用否决权,则会得罪以色列并使奥巴马失去一部分支持以色列的选民。如果以上陈述为真,以下哪项陈述一定为真?

A.美国会在安理会动用否决权,阻止巴勒斯坦进入联合国。B.美国不会得罪以色列,却会在整个阿拉伯世界引燃反美情绪。C.美国会在阿拉伯世界引燃反美情绪,或者奥巴马会失去一部分支持以色列的选民。

D.即使美国动用否决权,联合国大会仍打算投票表决,让巴勒斯坦成为具有国家地位的观察员。

24.通过风险投资方式融资创建的公司比通过其他渠道融资创建的公司失败率要低。可见,与企业家个人素质、公司战略规划或公司管理结构等因素相比,融资渠道对于一个新公司的成功发展是更为重要的因素。如果以下哪项陈述为真,将最有力地削弱上述论证? A.有大约一半的新公司在创立后5年内就倒闭了。

B.初创公司的管理结构通常不如发展中公司的管理结构那样合理。C.与其他投资人相比,风险投资人对初创公司在财务需求方面的变化更为敏感。

D.在决定是否为初创公司提供资金时,风险投资人必须考虑企业家的个人素质、公司战略规划等因素。

25.未来的中国,将是一个更加开放包容、文明和谐的国家。一个国家、一个民族,只有开放包容,才能发展进步。唯有开放,先进和有用的东西才能进得来;唯有包容,吸收借鉴优秀文化,才能使自己充实和强大起来。如果以上说为真,以下哪项陈述一定为假? .A.一个国家或民族,即使不开放包容,也能发展进步。B.一个国家或民族,如果不开放包容,它就不能发展进步。C.一个国家或民族,如果要发展进步,它就必须开放包容。D.一个国家或民族,即使开放包容,也可能不会发展进步。

26.大约在12000年前,当气候变暖时,人类开始陆续来到北美洲各地。在同一时期,大型哺乳动物,如乳齿象、猛犸和剑齿虎等,却从它们曾经广泛分布的北美洲土地上灭绝了。所以,与人类曾与自然界其它生物和平相处的神话相反,早在12000年前,人类的活动便导致了这些动物的灭绝。上述论证最容易受到以下哪项陈述的质疑? A.该论证未经反思地把人类排除在自然界之外。B.人类来到北美洲可能还会导致乳齿象、猛犸和剑齿虎之外的其它动物灭绝。

C.乳齿象、猛犸和剑齿虎等大型哺乳动物的灭绝,对于早期北美的原始人来说,具有非同录常的意义。

D.所提出的证据同样适用于两种可选择的假说:气候的变化导致大型哺乳动物灭绝,同样的原因使得人类来到北美洲各地。

27.平均而主,今天受过教育的人的读书时间明显少于50年前受过教育的人的读书时间。但是,现在每年的售书册数却50前增加了很多。以下各项陈述都有助于解释上述现象,除了 ..A.今天受过教育的人比50年前受过教育的人的数量大大增加。B.与现在相比,50年前的人们更喜弞从图书馆借阅图书。

C.与现在相比,50年前的人更喜欢通过大量藏书来显示其良好的教育和品位。

D.现在的书往往比50年前的书更薄,也更容易读。

28.思想观念的价值,在竞争中才会彰显,在实践中才能检验。“我不同意你的看法,便我誓死捍卫你说话的权利”,这是一种胸怀,更是一种自信。那种扣帽子、抓辫子的辩论方式,“不同即敌对”的思维模式,本质上都是狭隘虚弱的表现,无助于和谐社会的构建。全社会都应该以包容的心态对待“异质思维”,尤其是执掌权柄者。

以下各项陈述都支持上述观点和论证,除了 ..A.让人说话天不会塌下来,科学是在不断颠覆或改进“正统”观念中前进的。B.个别的异质思维者是偏执狂,其言行近乎疯子,理应受到严格管制。C.每个人,甚至是当今的执掌权柄者,者有可能在某一天成为异质思维者,成为少数派。

D.如果社会成员的思想和情绪得不到适当的表达和宣泄,容易导致各种极端行为。

29.一家商场按下述方式促销商品:一年中任何时候,或者有季节性促销,或者有节日促销,或两者兼而有之。每一种促销都会持续一个月。在任何一个月,如果商场想要把某一类商品清仓,就宣布季节性促销;如果某个月份有节日并且仓库中仍有剩余商品,就宣布节日促销。不过,11月没有节日而且这个月份仓库中他没有剩余商品。

以下哪项陈述能从上文中合逻辑的推出?

A.如果某个月没有季节性促销,这个月一定有节日促销。B.如果节日促销没有进行,那一定是在11月份。

C.如果季节性促销在某个月进行,这个月仓库中一定有剩余商品。D.如果在某个月中有节日,但仓库中没有剩余商品,则宣布节日促销。30.在对6岁儿童所做的小学入学前综合能力测试中,全天上甲学班达9个月的儿童平均得分58,只在上午上甲学前班达9个月的平均行分52,只在下午上甲学前班达9个月的平均得分51;全天上乙学前班达9个月的平均得分54;而那些来自低收入家庭且没有上过学前班的6岁儿童,在同样的小学入学前综合能力测试中平均得分32。在统计学上,32分与上述其他分数之间的差距有重要意义。

从上面给定的数据,可以最合理地得出下面哪个假设性结论? A.得50分以上的儿童可以上学。

B.要做出一个合情理的假设,还需要做更多的测试。C.是否上过学前班与小学入前的综合能力之间有相关性。D.应该给6岁以下儿童上学前班提供更多的经费支持。

31.稀土是储量较少的一类金属的统称,广泛应用尖端科技和军工领域。世界绝大多部分稀土增自中国。1998年以来,中国开始减少稀土开采量,控制稀土出口配额,加强稀土行业的集中度。对此,一此国家指责中国垄断资源,对世界其他国家“有极大的破坏性”,要求中国放宽对稀土产业的控制。如果以下哪项陈述为真,最适合用来反驳这些国家对中国的指责? A.稀土是不可再生的重要战略资源。

B.目前按人均计算,中国已经属于稀土资源相对稀缺的国家。

C.从1980年代起,中国一些地方对稀土滥采滥挖,造成资源的严重浪费和对环境的极大破坏。

D.2009年,中国稀土储量占全球储量的36%,美国占13%,俄罗斯占19%,澳大利亚占5.4%;中国产量占世界产量的97%,其他3个国家均为零。

32.政治家:大约4年前,我们党一开始执政就致力于治理通货膨胀,并成功地将消费者物价指数(CPI)的涨幅从当时的4% 降到现在的1.5%。反对党在前一个4年的执政期间,CPI涨幅都超过了4%.。因此,在我们党的领导下,商品的价格越来越低。

这个政治家的论证最容易受到以下哪项批评?

A.没有详细地说明反对党的主张,而是简单的将它忽略了。

B.用来支持这一结论的证据实际上为否定这一结论提供了更多的支持。C.没有提到反对党执政期间是国内经济过热的时期,而现在欧美出现了经济危机。

D.没有提到这样一种可能性:反对党执行了治理通货膨胀的政策,但该政策的效果要经过一段时间能显现。

33.一般性况下,上市公司的业绩与其股票价格是正相关的:一个公司公布年报时,如果利润高于市场预期,则该公司的股票价格会上涨。然而,2011年3月31日,我国从事核电等电力机组生产的东方电气公司公布年报,67%的净利润增速超过了市场预期,但收盘时期股票价格却下挫1.59%,跑输大盘0.73个百分点。

如果以下哪项陈述为真,能够最好地解释上述反常现象?

A.2011年3月11日,日本发生地震和海啸,福岛第一核电站发生严重核泄漏事故,我们有人抢购碘盐。

B.2011年3月16日,中国国务院决定对核没施进行全面检查并暂停审批核电项目。

C.2011年3月底至4月26日,世界各地举办纪念切尔诺贝利核电站爆炸25周年的活动。

D.经过长期辩论,2011年5月德国执政联盟决定,将于2022年前关闭德国所有核电站

34.在某公司内部,新近设立了许多非常专门化的部门。这表明,该公司对下述问题非常感兴趣:如何以更精准的方式抓住其消费群体? 上述推理缺少下面哪一个选项?

A.前提:这些新部门在以更精准的方式去抓住其消费群体。B.结论:管理部门要求采取新的措施去抓住其消费群体。C.前提:在设立这些新部门之前,该公司未能抓住其消费群体。D.结论:该公司将很快做更多的努力去抓住其消费群体。

35.我国农村的宅基地属集体所有,农民只使用,不能买卖、出租和继承。宅基地制保证了农民的生存权益。农民在宅基地上建造的房屋是农民的资产。如果允许农民出卖自己的房屋,则实际上允许出让宅基的使用权。如果宅基地的使用权被另人买走,则会损害农民的生存权益。但如果不允许农民出卖自己的房屋,则侵害了农民的资产权益。如果以上陈述为真,以下哪项陈述一定为真? A.农民在宅基地上建造的房屋没有产权证。

B.如果农民工在城市里购买了住房,则不应当现在农村老家占有宅基地。C.如果不损害农民的生存权益,则会侵害农民的资产权益。

D.对绝大多数农民工而言,农村老家的宅基地和责任田是他们最后的生存保障。

36.自从1989年阿拉斯加埃克森油轮灾难和1991年中东战争以来,航空油料的价格已经巨幅上涨。在同一时期内,几种石油衍生品的价格也大幅上扬。这两个事实表明:航空油料是石油衍生品。以下哪项陈述最好地评价了上述论证? A.好的思维,因为航空油料是石油衍生品。B.坏的思维,没有精确地陈述所有的事实。

C.坏的思维,同一时期内食品价格也上涨了,但这不能证明航空油料是食品。D.坏的思维,给定关于石油衍生品的事实,不能得出关于航空油料的任何结论。

37.2011年8月,中国第一艘航空母舰“瓦良格”号出海试航。中国拥有航母会对美国海军要西太平洋的制海权构成潜在的挑战,美军对此感到担忧并持防范态度。然而,美国一位富有实战经验的海军专却希望中国建造更多的航母。

如果以下哪项陈述为真,能够最好地解释这位专家看似矛盾的态度? A.2011年8月,越南与美国在南海进行了联合军事演习。

B.美国、日本、印度等国借中国航母试航之机,掀起新的“中国威胁论”的鼓噪。

C.中国海军并未充分部署支持航母的运输船和加油船,护卫航母的驱逐舰和潜艇也十分脆弱。

D.航母需要支持和护卫,建造航母越多,用来制造支持航母的其他舰艇和飞机的资源越少。

38.哲学家:在18世纪,某篇关于运动是绝对的论文断言,一个物体在一段时间内的位置变化可以不参照任何其他物体的位置而测得。不过,一位颇受尊敬的物理学家声称,这篇论文是连贯的。既然一篇不连贯的论文不能认为是对实在的描述,故运动不可能是绝对的。

这个哲学家的论证使用了下面哪一种论证方法或技巧? A.使用专业术语来说服别人。

B.依赖某个专家的权威来支持一个前提。

C.使用实验结果来说明所提到的位置变化是和合理的。D.观察到某物在实验条件下是某种情况,推出该物在任何情况下也是这种情况。

39.国内油价与国际油价相比一向是“涨快跌慢”,原因之一是我国成品油调价依据三地原油价格变动超过4%这个边界条件。比如,从100美元一桶涨动104美元时,我们就跟着涨价;但是从104美元回调4%再降价,则要等价格回落到99.84美元。两个条件实际上是不同的,降价要求的条件更高,更不容易达到。

以下哪项陈述最好的指出了上述论证的缺陷?

A.上述论证没能说明,国内油价不合理是由多种原因造成的。B.上述论证没能说明,国内油价不合理的根本原因在于行业垄断。C.上述论证没能说明,以4%为边界条件,则油价越高,国内反应越迟钝。D.上述论证没有从相同的价格基点出发进行比较,如果涨价的基点是100美元,对降价的条件分析也应该以100美元为基点。

40.在某个班级中,L同学比X同学个子矮,Y同学比L同学个子矮,但M同学比Y个子矮。所以Y同学比J同学个子矮。

必须增加以下哪项陈述做前提,才能合逻辑地推出上述结论? A.J同学比L同学个子高。B.X同学比J同学个子高。C.L同学比J同学个子高 D.J同学比M同学个子高。41—45题基于以下共同的题干: 一家剧院计划在秋季的7周内上演7个剧目,它们是F、G、J、K、O、R、S。每周上演一个剧目,每个剧目恰好了演出一周。剧目安排必须满足以下条件:

(1)G必须在第三周上演。(2)O和S不能连续演出。

(3)K必须安排在J和S之前上演。(4)F和J必须安排在连续的两周中演出。

41.如果把F安排在第五周上演,以下哪项准确列出了所有可以安排在第七周上演的剧目?

A. J、S B. O、R C. O、R、S D. K、O、R 42.对于任何一种可接受的安排,以下哪项一定为真?

A.F被安排在K之后某一周。B.G恰好被安排在O之前的那一周。C.J被安排在第一周。

D.R被安排在第二周或第七周。

43.如果把R安排在第五周,把O安排在第二周,则可供选择的安排方式有

A.2种 B.3种 C.5种 D.6种 44.如果把S安排在第六周,那么必须把R安排在哪一周?

A.第二周 B.第四周 C.第五周 D.第七周

45.如果O恰好被安排在J之前的哪一周,以下哪项一定为真?

A.把F安排在O之前。B.K被安排在G之前的某一周。C.R被安排在第一周或第二周。D.S恰好安排在K之后的哪一周。46—50题基于以下共同的题干:

一个委员会工作两年,每年都由4人组成,其中2名成员来自下面4位法官:F、G、H和I,另外2名成员来自下面3位科学家:V、Y和Z。每一年,该委员会有1名成员做主席。在第一年做主席的成员在第二年必须退出该委员会。在第二年做主席的人在第一年必须是该委员会的成员,该委员会的成员满足下面的条件:

G和V不能在同一年成为该委员会的成员。H和Y不能在同一年成为该委员会的成员。

每一年,I和V中有且只有一位做该委员会的成员。46.下面哪项列出了能够在第一年成为该委员会成员的名单?

A.F,G,V,Z B.F,H,V,Z C.H,I,Y,Z D.G,H,I,Z 47.如果V在第一年做该委员会主席,下面哪一选项列出了在第二年必须做该委员会成员的两人? A.G和Y B.V和Y C.H和I D.I和Y 48.如果H在第一年做席,下面哪一位能哆在第二年做主席?

A.F B.G C.Y D.I 49.如果F在某一年是该委员会成员,下面任何一位都可以在那一年是该委员会成员,除了 A.I B.H C.G D.Y 50.下面哪项一定为真?

A.H在第一年是该委员会成员。B.F在第二年是该委员会成员。C.I在两年之内都是该委员会成员。D.Z在第二年是该委员成员。答案:

1、C

2、A

3、A

4、C

5、B

6、C

7、B

8、C

9、C

10、C

11、D

12、C 13 20、A

21、C

22、B 23 30、C

31、D

32、D 33 40、A

41、C

42、A 43 50、D、C

14、B

15、C

24、D

25、B

34、A

35、C

44、D

45、B

16、C

17、A

26、D

27、C

36、C

37、B

46、B

47、D

18、C

19、B

28、B

29、B

38、B

39、D

48、A

49、B、A、C、C

第三篇:2012年GCT真题-2012年GCT考试真题及答案(含英语、逻辑、语文、数学4科)

GCT 2012年考试试卷

外语运用能力测试(英语)

(50题,每题2分,满分100分)

Part One Vocabulary and Structure

Directions:

There are ten incomplete sentences in this pαrt.For each sentence there αre four

choices marked A,B,C and D.Choose the one that best completes the sentence.Mark

your answer on the ANSWER SHEET with a single line through the center.1.I realized I had let myself in something from which there was no turning _____.A.around B.back

C.away D.down

2.You are to stay at the hotel where rooms _________for you.A.have been booked B.are booked

C.would be booked D.were booked

3.If you ______to my advice,you wouldn't be in this mess right now.A.listen B.listened

C.had listened D.would listen

4.Those close to him are _________ that he hopes to stay on till the end of his term.A.ensured B.confirmed

C.suppose D.convinced

5.Working women haven't left the family role behind: now they are _____to work

even harder to do both.A.expected B.wished

C.hoped D.desired

6.Modem technology has brought ______communication between people far apart.A.competent B.convenient

C.conscious D.complete

7.Under no_______ are children allowed to tell lies to their parents.A.circumstances B.situations

C.occasions D.moments

8.We need a more capable leader, _____with a strong will as well as good humor.A.who B.that

C.one D.which

9.The lectures,_____ the current hot issues,were well received.A.that covered

B.covered

C.covering

D.to cover

10.Being an intelligent boy,he ______such a foolish mistake.A.needn't have made B.can’t have made

C.won't have made D.wouldn’t have made Part Two Reading Comprehension

Directions:

In this part there are three passages and one chart,each followed by jive questions

or unfinished statements.For each of them,there are four choices marked A,B,C and

D.Choose the best one and mark your answer on the ANSWER SHEET with a single

line through the center.Questions 11-15 are based on the following passage:

Ever won the lottery? No? But did that stop you buying another lottery ticket? If the answer is another “no“,you might call yourself an optimist.According to researchers at University College London,human beings are sanguine creatures.It is all in the brain,they say.A study suggests that human brain is very efficient at processing good news: about 80% of people have a tendency to see the glass as half-full,not half-empty,even if they don't consider themselves to be optimists.The good news is that this brings a health benefit , Having a positive outlook on life reduces anxiety.A study of nearly 100,000 women showed a lower risk of death from heart disease among optimists.But there are problems in always having an optimistic attitude.The authors of the study point out that the 2008 financial crisis may have been caused by analysts overestimating their assets' performance even in the face of clear evidence to the contrary.There are personal health risks too.Dr Tali Sharot,lead researcher,said: '''Smoking Kills' messages don't work since people think their chances of cancer are low.There's a very fundamental tendency in the brain.”

But,as they say,every cloud has a silver lining.Even if seeing the world through rose-colored glasses poses a risk to our health,it's not something that is likely to cause us to lose sleep.Let's just keep our chins up and keep smiling!

11.What does “sanguine”(para.2)mean ?

A.Depressed B Cheerful C.Lucky D.Emotional

12.The study mentioned in Paragraph 2 indicates that people______.A.are more optimistic than they believe

B.are less optimistic than they believe

C.like good news more than they think

D.like good news less than they think.What study mentioned in Paragraph 2 indicates that people _____.A.Collecting false information

B.OveremJ1hasizing evidence.C.Misjudging the situation.D.Giving a pessimistic forecast.14.The author suggests in the last paragraph that we______

A.adjust our goals in life

B.learn to release bad mood

C.avoid being overoptimistic

D.maintain a positive attitude.what is the main of the passage ?

A.People tend to be optimistic even in crisis.B.Optimists enjoy life better than pessimists.C.Being optimistic has both benefits and risks.D.Optimism is what keeps us going forward.Questions 16-20 are based on the following passage:

The International Olympic Committee(IOC)has launched an investigation into allegations that its officials in more than 50 countries have been selling London Olympics tickets on the black market for profit.The IOC met at an emergency session on Saturday to look into a pile of evidence uncovered by Britain's Sunday Times newspaper.The paper claims that high-ranking Olympic officials have been selling tickets for the games at hugely inflated prices.The highest priced tickets on the black market were for the men's 100-meter final.The roc has issued a statement saying it takes these allegations very seriously and has immediately taken the first steps to investigate.“

IOC rules forbid national Olympics committees from selling tickets overseas,increasing ticket prices or selling tickets to unauthorized,third-party resellers.Despite this,Sunday Times undercover report posing as illegal ticket sellers say they have recorded evidence of 27 officials selling tickets distributed to 54 countries One of the most serious allegations was against the Greek Olympic Committee president Spyros Capralos.He denies saying he had ”pulled strings“ with the head of the London Olympics Sebastian Coe even though the paper posted videos of its reporters' negotiations with Capralos on its website.Mr Capralos has so far refused to comment.The IOC is investigating its own officials for _______.A.running the black market B.secretly meeting reporters

C.illegally selling tickets D.criticizing Sunday Times

17.The word “allegations “(Para.1)”probably means “_______”

A.discussions B.claims

C.announcements D.opinions

18.Sunday Times revealed _______.A.details about IOC's emergency session

B.evidence against some IOC officials

C.ticket prices for men's 100-meter final

D.pricing policy of the London Olympics

19.Which of the following is NOT against the IOC rules concerning its officials?

A.Selling tickets internationally.B.Raising the prices of the tickets.C.Selling tickets to unofficial resellers.D.Giving tickets to their friends..Faces with the evidence against him , Capralos _____

A.blamed other officials

B.admitted the facts

C.issued a statement

D.gave no response Questions 21-25 are based on the following passage:

Pressed by competition and its own success,the popular search engine Google has created an automated way to search for new employees who are fully appropriate as well as high-achieving.In a project,the 100,000 people who fill in online job applications for Google each month will be asked to complete a complicated questionnaire(问卷)exploring their attitudes,behaviour,personality and backgrounds going back to their school days.The questions range from whether applicants have ever set a world record,to whether their workspace is messy or tidy or what magazines they read.Answers are studied by Google's mathematicians to calculate a score meant to predict how well a person will fit into the organization's diversified and competitive culture.Psychometric tests(心理测试)are already used by more traditional companies to select workers,but they are unheard of in a company like Google,which is built on a belief in individual talent.The online questionnaire is based on the answers to 300 questions sent out last summer to every employee at the head office in California's Silicon Valley.Some questions were factual: What programming languages are you familiar with? What internet mailing lists are you on?

Other questions,however,tried to establish personality and behavioural characteristics: Have you ever tutored another person?

”We wanted to cast a very wide net,“ said Laszlo Bock,Google's Vice-President for People Operations.”It is not unusual to walk into our office and bump into dogs.Maybe people who own dogs have some personality feature that is useful.“ Google has created a new way of recruiting in order to_____.A.find the most appropriate employees

B.better compete with other companies

C.spread its unique co叩orate culture

D.conduct an online research project

22.The information gained from the questionnaire will be _____.A.analyzed in a report

B.summed up in a figure

C.organized into a chart

D.kept in its raw state.What is true about the Google online questionnaire?

A.It was based on a survey of its employees.B.It focuses on background questions.C.It was designed by some mathematicians

D.It has been filled out by 100,000 people

24.What distinguishes Google online questionnaire?

A.Preferring those who own pets.B.Stressing overall personal qualities.C.Asking more complex questions.D.Believing in high-achieving experience.25.By ”casting a wide net“(last paragraph),Laszlo Bock means

A.recruiting a wide range of talents

B.finding candidates with peculiar personality

C.identifying candidates with wide interests

D.picking out as many applicants as possible

Questions26-30are based on the following chart :

Cigarette Smoking by Grade Level: 2005-2009

□2005 □2007 ■.2009

(说明:2005年为白框,2007年为灰框,2009年是黑框,扫描问题无法显示。)

26.The purpose of the chart is to show________

A.the percentages of smokers in schools

B.the increase of smokers in grades 9-12 during 2005-2009

C.the number of teenage smokers in grades 9-12

D.the amount of money spent by young smokers-over the years

27.Which grade had the highest percentage of smokers in 2005

A.The 9th.B.The 10th.C.The 11th.D.The 12th

28.The percentage of the 9th grade smokers in 2009 was ______

A.23.2 B.25.2

C.31.2 D.33.4

29.Which grade saw the greatest increase of smokers over the four years?

A.The 9th.B.The 10th.C.The11th可 D.The 12th.30.What tendency can we find from the chart?

A.Smokers in all grades have increased.B.The higher the grade,the more the smokers.C.The older a student,the more he smokes.D.Low graders smoke less heavily.Part Three Cloze

Directions:

There are ten blanks in the following passage.For each numbered blank,there are

four choices marked A,B,C and D.Choose the best one and mark your answer on the

ANSWER SHEET with a single line through the center.Working memory, or short-term memory,involves the ability to hold and use

information in the immediate future.31 is only held in working memory for about 20 seconds.The challenge that students32is to move information from their working memories into their long-term memories.If they don't do this in about the first few minutes after receiving the information,that information can be lost.To keep this newly learned material from33away,it needs to enter the network of the brain's wiring.After repeated practice,working memories are set down as permanent neuronal

(神经的)circuits34to be activated(激活)when the information is needed.When a memory has been recalled35 its neuronal circuits are more highly developed

because of their repeated activation.36exercising a muscle,these circuits then become more efficient and easier to access and activate.Practice results37 repeated stimulation of the memory circuit.Like hikers along a path 38 eventually leave a depression in the road,repeated practice stimulates cells in the memory circuit such that the circuit is reinforced and becomes 39.This means it can be quickly turned from off to on,and switched.40through a variety of cues coming in from the senses.31.A.Material B.Information C.Knowledge D.Ability

32.A.have B.take C.face D.hold

33.A slipping B.turning C.moving D.stepping

34.A.useful B.good C.open D.ready

35.A.usually B.often C.well D.soon

36.A.During B.By C.Like D.Unlike

37.A.in B.at C.from D.with

38.A.where B.which C.what D.who

39.A.stronger B.weaker C.longer D.shorter

40.A.over B.back C.off D.on

Part Four Dialogue Completion Directions:

In this part, there are ten short incomplete dialogues between two speakers, each followed by four choices marked ,B,C and D.Choose the one that most appropriately suits the conversational context and best completes the dialogue.Mark your answer on the ANSWER SHEET with a single line through the center.41 A:I've called you a hundred times today.B: ________ I was busy.A.Sorry about that.B.Sounds fine.C.I appreciate that.D.That's all right.42.A: I thought you were working until 6:30.B: ___________,but we finished our meeting at 5:30 and were let go.A.I tried to B.I was supposed to

C.I hoped so D.I 'd love to

43.A:Are you ready for the test tomorrow?

B: __________

A: Come on.I am sure you will do well.A.Why do you ask? B.Sure,no problem.C.What about you? D.No.I'm afraid not.44.A:That necklace looks really lovely on you.B: __________

A.It cost me a fortune.C.It will be nice on you too.B.It's very nice of you.D.I hope you like it.45.A: This is like the coolest CD I've ever heard.B:_________I'm trying to concentrate on my work.A.Cut it short.B.Keep it down.C.Turn it over.D.Bring it up.46.A: I need to get this done by noon.Can you give me a hand?

B: ____________

A.I can give you both hands

B.I'll watch your back.C.I'm all yours

D.The pleasure is mine.47.A: Good morning.I'd like to book a table for two for 8:30 on Friday,please.B: ____________

A.Yes,speaking.B.Hold the line.C.Your name,please? D.After a while.48.A:It's Janet Smith._________

B: Yes,I'd like to have some information about having a phone installed.A.Who's that? B.What's wrong?

C.What's up? D.May I help you?

49.Shop assistant: What do you have in mind?

Customer :I'm thinking about jewelry or something valuable.Shop assistant: We've got beautiful jewelry here.If you're interested,A.I can show you B.you can see it

C.I'll get it D.you should buy it

50.A: I like this apartment very much,but I'll come back this evening with my wife

and kids.Will that be convenient?

B: ________

A.It's good for you.B.That's fine with me.C.It's thoughtful of you.D.That's right.外语运用能力测试(英语)

1--5:CACDA

6--10:BACCB

11--15:BACDC

16--20:CBBDD

21--25:BBABA

26--30:BCDAA

31--35:BCADB

36--40:CADAA

41--45:ABDBB

46--50:CCDAB

逻辑推理能力测试

(50题,每题2分,满分100分)

1.中世纪阿拉伯人有许多古希腊文手稿。当需要的时候,他们就把它们译成阿拉伯语。中世纪阿拉伯哲学家对亚里士多德的《诗论》非常感兴趣,这种兴趣很明显不被阿拉伯诗人所分享,因为一个对《诗论》感兴趣的诗人一定想读荷马的诗,亚里士多德本人就经常引用荷马的诗。但是,荷马的诗一直到当代才被译成阿拉伯语。

如果以下陈述为真,哪一项最强地支持上面的论证?

A.有一些中世纪阿拉伯翻译家拥有古希腊文的荷马诗手稿。

B.亚里士多德的《诗论》经常被现代阿拉伯诗人引用和评论。

C.亚里士多德的《诗论》的大部分内容与戏剧有关,中世纪阿拉伯人也写戏剧作品,并表演这些作品。

D.中世纪阿拉伯的系列故事,如《阿拉伯人之夜》,与荷马史诗的某些部分很相似。

2.美联储一直想推出第三次量化宽松货币政策(简称QE3),以推动美国经济复苏。如果美联储推出QE3,则全球美元供给将再次大幅增加,各国要维护汇率稳定,就不得不购买美元资产。如果各国购买美元资产,则会加大本国通货膨胀压力。如果不想输入通货膨胀,各国就要让本币升值。如果本币升值,则会抑制本国出口,导致经济滑坡。

如果以上陈述为真,以下哪一项陈述一定为真?

A.如果美联储推出QE3,其他国家若想避免本国经济滑坡,就不要购买美元资产。

B.其他国家或者面临输入性通货膨胀的压力,或者面临经济滑坡的危险。

C.如果其他国家没有输入性通货膨胀,也没有本币升值,则美联储未推出QE3.D.如果其他国家未遇到输入性通货膨胀的压力,就不会让本币升值。

3.2012年8月10日,韩国总统李明博访问了与日本存在主权争议的独岛(日本称”竹岛“)。舆论调查结果显示,在李明博访问独岛之后,其支持率由25.7%升至34.7%.如果调查结果属实,以下哪一项陈述一定为真?

A.支持李明博访问独岛的韩国人多于不支持的人。

B.在李明博访问独岛之后,一部分先前不支持他的人现在转而支持他。

C.2012年8月10日前支持李明博的韩国人现在继续支持他。

D.李明博访问独岛是其支持率提升的原因o

4.近代法国思想家帕斯卡曾这样论证说:或者有一个基督教的上帝,或者没有。假设你相信上帝存在,并按基督教的方式生活,那么,如果上帝确实存在,你将得到无尽的福祉:如果上帝不存在,你所失去的东西也非常少。但是,假设你不相信上帝存在,且不按基督教的方式生活,如果上帝确实不存在,你不会失去什么:如果上帝确实存在,你会受到无尽的惩罚。所以,不必再犹豫了,选择相信上帝存在并按基督教的方式去生活吧。

以下哪一项最合理地说明了此论证的逻辑漏洞?

A.该论证错误地预设一个人可以随意选择相信还是不相信上帝存在。

B.帕斯卡使信仰变了质,把信仰变成了有关自我利益计算的问题。

C.假如上帝不存在,我们却选择相信上帝,这会导致我们对世界的错误认知,稀里糊涂地度过一生。

D.这个世界有多种宗教,每种宗教都有法力无边的神(上帝).都会对不信仰自己的人施加无限的惩罚。

5.政治家”那些声称去年全年消费物价涨幅低于3%的经济学家是错误的。显然,他们最近根本没去任何地方买过东西。汽油价格在去年一年中涨了10%.我乘车的费用涨了12%.报纸涨了15%.清洁剂涨了15%.面包涨了50%“政治家的上述论证最容易受到批评,因为

A.它指责经济学家的品德,而不是针对他们的论证进行反驳。

B.它使用了一个不具有代表性的小样本作为证据。

C.它试图通过诉诸感情的方式来达到说服的目的。

D.它错误地表明,所提到的那些经济学家不是消费价格领域的专家。6.像”X X集团举行周年庆典,您的手机号码获得了10万元大奖“这类并不高明的手机诈骗短信,即使经媒体曝光后仍然一再出现。职业骗子宁肯使用低劣的诈骗短信,也不去设计一些更具欺骗性、更易让人上当的短信,只能说明骗子太笨、太不敬业了。

如果以下陈述为真,哪一项最强地反驳了上述结论?-

A.骗子一定是聪明的,否则不可能骗得了别人。

B.骗子行骗时会想方设法不引起警察的注意。

C.如果一种骗术毫无作用,骗子早就将它淘汰了。

D.骗子使用这样的短信”钓取“可能上当的人,他们希望一开始就将聪明人过滤掉。

7.试考虑三个数: 0.9、1、1.1,后一个数与它前一个数的差只有0.1。若让每个数与自身连乘10次,0.9变成了0.31,1仍然是1,1.1变成了2.85,它是0.31的近10倍,1的近3倍。差距就是这样产生的!

从以上陈述不能合理地引出下面哪一个结论?

A.失之毫厘,差之千里。

B.细节决定成败,性格决定命运。

C.微小差别的不断累积和放大,可以产生巨大的差别。

D.每个人都必须当心生命过程中的每一步:小胜有可能积成大胜,小过有可能铸成大错。

8.经济学家:美国的个人所得税是累进税,税法极其复杂。想诚实纳税的人经常因理解错误;而出现申报错误;而故意避税的人总能找到税法的漏洞。一般而言,避税空间的大小与税制的复杂程度成正比,避税能力的高低与纳税人收入水平成正比。复杂税制造成的避税空间大多会被富人利用,使得累进税达不到税法规定的累进程度,其调节分配的功能也大大弱化。

如果以下陈述为真,哪一项对经济学家的上述论证提供了最强的支持?

A.在申报纳税时,美国有60%的人需要雇请专业人士代理申报,有22%的人需要用报税软件帮助计算。

B.美国人在1981年就提出了”废除累进税率,实行单一税率“的设想。

C.1988年至2006年,美国1最富人群的收入占全国收入的比重从15%上升为22%,但他们的平均税率却从24%降到22.8%。

D.2011年9月17日美国爆发了占领华尔街运动,示威者声称代表美国99的民众抗议金融业的贪婪腐败及社会不公。

9.一个作战计划中的弱点是绝密中的绝密,不会向外泄露。但是,日本媒体所公布的日本自卫队关于钓鱼岛的”夺岛“计划,除预测未来钓鱼岛可能出现的3种事态、自卫队将分5步夺岛外,还详细列出自卫队的弱点:没有能力从北海及九州岛迅速向冲绳大规模运送兵力,以满足登陆作战需要。

如果以下陈述为真,哪一项最好地解释了日本这种违反常理的做法?

A.日本公布”夺岛“计划意在试探中国的反应:如果中国反应不大,日本将在钓鱼岛及周边岛屿驻军。

B.日前中国与菲律宾在黄岩岛对峙,日本公布”夺4岛“计划是为了拉拢菲律宾,联手牵制中国。

C.日本自曝弱点是为建造运输能力强大的两栖战舰在国内造舆论:两栖战舰属于进攻力量,而日本的”和平宪法“不允许自卫队发展进攻力量。

D.许多日本人希望美国从日本撤军,日本自曝自卫队的弱点旨在告诉国民:日本在军事上还需要美国的保护。

10.野生动物保护组织:没有买卖就没有杀戮;没有杀戮,人与自然才能和谐相处。

如果以上陈述为真,以下哪一项陈述一定为真?

A.只要有杀戮,就一定有买卖。

B.只要禁止了买卖,人与自然就会和谐相处。

C.只有禁止了买卖,人与自然才会和谐相处。

D.人与自然之所以没能和谐相处,是因为存在杀戮。

11.腊有人论证说:探究是不可能进行的,因为一个人既不能探究他所知道的,也不 能探究他所不知。他不能探究他所知道的,因为他知道它,无需再探究;他不能探究他所不知道的,因为他不知道他要探究的东西是什么。以下哪一项最准确地指出了该论证的逻辑漏洞?

A.虚假预设:或者你知道你所探究的,或者你不知道你所探究的。

B.循环论证:把所要论证的结论预先安置在前提中。

C.强词夺理:理性上黔驴技穷,只好胡搅蛮缠。

D.歧义性谬误:其中”知道“有两种不同涵义:知道被探究问题的答案是什么:知道所要探究的问题是什么。

12.银行的信用卡章程规定:凡使用密码进行的交易,均视为持卡人本人所为。这意味着,只要信用卡被盗刷时使用了密码,银行均视为持卡人本人所为,对所发生的损失概不负责。因此,为了使自己的信用卡更安全,应当不设密码。

如果以下陈述为真,都能削弱上述结论,除了

A.有关专家认为信用卡不设密码更安全,但专家的话也不一定全对。

B.犯罪分子伪造设有密码的信用卡时,必须另行设法获取其密码才能盗刷成功。

C.信用卡遗失时,信用卡的密码能够有效阻止他人刷卡交易。

D.盗刷的案件中,如果信用卡未设密码,法院通常认定卡主人有一定过错,需承担不分损失。

13.有些未受过大学教育的人成了优秀作家,而更多的优秀作家是受过大学教育的。优秀作家都是敏感而富有想象力的人,只有敏感而富有想象力的人才能写出打动人心的作品。

如果以上陈述为真,以下哪一项陈述一定为真?

A.只有优秀作家才能写出打动人心的作品。

B.有些敏感而富有想象力的作家不是优秀作家。

C.能写出打动人心作品的作家都是优秀作家。

D.有些敏感而富有想象力的作家未受过大学教育。

14.一般来讲,某种产品价格上涨会导致其销量减少,除非价格上涨的同时伴随着该产品质量的改进。在中国,外国品牌的葡萄酒是一个例外。很多外国品牌的葡萄酒价格七涨往往导致其销量增加,尽管那些品牌的葡萄酒的质量并没有什么改变。

如果以下陈述为真,哪一项最好地解释了上述反常现象?

A.许多消费者在决定购买哪种葡萄酒时,依据大众媒体所刊登的广告。

B.定期购买葡萄酒的人对葡萄酒的品牌有固定的偏好。

C.消费者往往根据葡萄酒的价格来判断葡萄酒的质量。

D.葡萄酒零售商和生产者可以通过价格折扣来暂时增加某种葡萄酒的销量。

15.对所有不道德的行为而言,以下两个说法成立:其一,如果它们是公开实施的,它们就伤害了公众的感情:其二,它们会伴有内疚感。

如果以上陈述为真,以下哪一项陈述一定为假?

A.每一个公开实施的伴有内疚感的行为者都是不道德的。

B.某些非公开实施的不道德的行为不会伴有内疚感。

C.不道德的行为是错误的,仅仅是因为有内疚感伴随。

D.某些伤害公众感情的行为如果是公开实施的,它们就不会伴有内疚感。

16.在两个试验大棚内种上相同数量的茄子苗,只给第一个大棚施加肥料甲,但不给第二个大棚施加。第一个大棚产出1200公斤茄子,第二个大棚产出900公斤茄子。除了水以外,没有向这两个大棚施加任何其他东西,故必定是肥料甲导致了第一个大棚有较高的茄子产量。

如果以下陈述为真,哪一项最严重地削弱了上面的论证?

A.少量的肥料甲从第一个大棚渗入第二个大棚。

B.在两个大棚中种植了相同品种的茄子苗。

C.两个大棚的土质和日照量有所不同。

D.第三个大棚施加肥料乙,没有施加肥料甲,产出1000公斤茄子。

17.某中学自2010年起试行学生行为评价体系。最近,校学生处调查了学生对该评价体系的满意程度。数据显示,得分高的学生对该评价体系的满意度都很高。学生处由此得出结论:表现好的学生对这个评价体系都很满意。

该校学生处的结论基于以下哪一项假设?

A.得分低的学生对该评价体系普遍不满意。

B.表现好的学生都是得分高的学生。

C.并不是所有得分低的学生对该评价体系都不满意。

D.得分高的学生受到该评价体系的激励,自觉改进了自己的行为方式。

18.目前俄罗斯在远东地区的耕地使用率不足50%,俄罗斯经济发展部有意向亚太国家长期出租农业用地。该部认为:如果没有外国资本和劳动力注入,俄罗斯靠自己的力量将无法实现远东地区的振兴。但是,如果外国资本和劳动力进入远东地区,该地区有可能被外国移民”异化“。

如果俄罗斯经济发展部的判断是正确的,以下哪一项陈述一定为真?

A.如果俄罗斯把外国资本和劳动力引进远东地区,该地区将实现振兴。

B.如果俄罗斯靠自己的力量能实现远东地区的振兴,该地区就不会被外国移民”异化“。

C.如果俄罗斯在将外国资本和劳动力引进远东地区的同时不断完善各项制度,该地区就不会被外国移民”异化“。

D.如果不靠自己的力量又要实现远东地区的振兴,俄罗斯将面临该地区可能被外国移民”异化“的问问题。

19.趋同是不同种类的生物为适应同一环境而各自发育形成一个或多个相似体貌特征的过程。鱼龙和鱼之间的相似性就是趋同的例证。鱼龙是海生爬行动物,与鱼不属于同一个纲。为了适应海洋环境,鱼龙使自身体貌特征与鱼类的体貌特征趋于一致。最引人注意的是,鱼龙像鱼一样具有鳍。

如果以上陈述为真,以下哪一项是上面陈述的合理推论?

A.栖居于同一环境的同一类生物的成员,其体貌特征一定完全相同。

B.不能仅因为一个生物与某类生物的成员有相似的体貌特征就把它们归为一类。

C.一种生物发育出与其他种类生物相似的体貌特征,完全是它们适应生存环境的结果。

D.同一类生物成员一定具有一个或多个使它们与其他种类生物相区别的体貌特征。

20.反核活动家:关闭这座核电站是反核事业的胜利,它表明核工业部门很迟才肯承认它们不能安全运作核电站的事实。核电站经理:它并不表明这样的事实。从非核资源可以得到便直的电力,再加上强制性的安全检查和安全维修,使继续经营这座核电站变得很不经济。因此,关闭这座核电站不是出于安全考虑,而是出于经济方面的考虑。该经理的论证是有缺陷的,因为

A.它不承认电力公司现在可能相信核电站是不安全的,即使关闭这座核电站不是出于安全考虑。

B.它把采取安全预防措施导致的费用上升看作单纯的经济因素。

C.它忽略了这样的可能性:从中可以得到便宜电力的那些资源本身也可能有安全问题。

D.它把关闭这座核电站对公众意味着什么的问题错误地当成关闭该电站的理由是什么的问题。

21.自2003年B市取消强制婚前检查后,该市的婚前检查率从10年前接近100%降至2011年的7%,为全国倒数第一。与此同时,该市的新生儿出生缺陷发生率上升了一倍。由此可见,取消强制婚前检查制度导致了新生儿出生缺陷率的上升。对以下各项问题的回答都与评价上述论证相关,除了

A.近十年来该市的生存环境(空气和水的质量等)是否受到破坏?

B.近十年来在该市育龄人群中,熬夜、长时间上网等不健康的生活方式是否大量增加?

C.近十年来该市妇女是否推迟生育,高龄孕妇的比例是否有较大提高?

D.近十年来该市流动人口的数量是增加还是减少了?

22.胼胝体是将大脑两个半球联系起来的神经纤维集束。平均而言,音乐家的.胼胝体比非音乐家的胼胝体大。与成年的非音乐家相比,7岁左右开始训练的成年音乐家,.胼胝体在体积上的区别特别明显。因此,音乐训练,特别是从幼年开始的音乐训练,会导致大脑结构上的某种变化。

以下哪一项是上面论证所依赖的假设?

A.在音乐家开始训练之前,他们的胼胝体并不比同年龄的非音乐家的胼胝体大。

B.在生命晚期进行的音乐训练不会引起大脑结构上的变化。

C.对任何两个从7岁左右开始训练的音乐家而言,他们的胼胝体有差不多相同的体积。

D.成年的非音乐家在其童年时代没有参与过任何能够促进胼胝体发育的活动。

23.由于中国代表团没有透彻地理解奥运会的游戏规则,因此在伦敦奥运会上,无论是对赛制赛规的批评建议,还是对裁判执法的质疑,前后几度申诉都没有取得成功。为使上述推理成立,必须补充以下哪一项作为前提?

A.在奥运舞台上,中国还有许多自己不熟悉的东西需要学习。

B.有些透彻理解奥运会游戏规则的代表团,在赛制赛规等方面的申诉中取得了成功。

C.奥运会上在赛制赛规等方面的申诉中取得成功的代表团都透彻理解了奥运会的游戏规则。

D.奥运会上透彻理解奥运会游戏规则的代表团都能在赛制赛规等方面的申诉中取得成功。

24.在我国,交强险是保险公司自主选择的险种。自2006年推出以来,只有2008年小幅赢利,其余年份均为亏损,且亏损额逐年加大,2011年全国交强险实际经营亏损达92亿元。奇怪的是,目前巨额亏损下的交强险依然是各保险公司争抢的业务。

如果以下陈述为真,哪一项最好地解释了保险公司争抢交强险业务的奇怪现象?

A.2011年,36家承担交强险的公司中有3家公司在这个险种上是赢利的。

B.在交强险赔付中,有些车辆赔付过高,部分不该赔付的案例被判赔付。

C.拖拉机享受惠农政策,许多地方将”运输车辆“登记为”拖拉机“,从而享受低税率。

D.商业车险利润丰厚,车主通常不会将交强险和商业车险分别投保两家公司。

25.销售专家认为,在一个不再扩张的市场中,一个公司最佳的销售策略就是追求最大的市场份额,而达到目标的最佳方式就是做一些能突出竞争对手缺点的比较广告。在国内萧条的奶粉市场中,A牌奶粉与B牌奶粉进行了两年的比较广告战,相互指责对方产品对婴儿的健康造成有害影响。然而,这些广告战并没有使各自的市场份额增大,反而使很多人不再购买任何品牌的国产奶粉。

以上陈述最强地支持下面哪一个结论?

A.不应该在一个正在扩张或可能扩张的市场中使用比较广告。

B.比较广告冒有使它们的目标市场不是扩张而是收缩的风险。

C.比较广告不会产生任何长期效益,除非消费者能很容易地判断那些广告的正确性。

D.如果一个公司的产品比其竞争对手产品的质量明显高出一筹的话,比较广告在任何情况下都能增加该公司产品的市场份额。

26.对于通过延迟退休年龄来解决养老金缺口问题的提议,很多网友心存疑虑,认为让本应退休的老年人继续留在职位上,会挤压年轻人的就业空间,加重年轻人求职难的问题。专家对此解释说:晚退休人群既是生产者也是消费者,他们的消费可以创造新的就业岗位。

如果以下陈述为真,哪一项最强地质疑了专家的解释?

A.延迟退休年龄会遭到在私营企业工作的人们的反对。

B.只有通过剌激经济发展才能从根本上降低失业率。

C.养老金缺口源于我国的养老制度设计不合理。

D.专家的解释基于在职老年人的消费能力明显高于退休的同龄人这一未经证实的假定。

27.2012年9月,欧盟对中国光伏电池发起反倾销调查。一旦欧盟决定对中国光伏产品设限,中国将失去占总销量60%以上的欧洲市场。如果中国光伏产品失去欧洲市场,中国光伏企业将大量减产并影响数十万员工的就业。不过,一位中国官员表示”欧盟若对中国光伏产品设限,将搬起石头砸自己的脚。“

如果以下陈述为真,哪一项将给中国官员的断言以最强的支持?

A.中国光伏产业从欧洲大量购买原材料和设备,带动了欧盟大批光伏上下游企业的发展。

B.欧盟若将优质低价的中国光伏产品挡在门外,欧洲太阳能消费者将因此付出更高的成本。

C.太阳能产业关乎欧盟的能源安全,俄罗斯与乌克兰的天然气争端曾经殃及欧盟各国。

D.目前欧洲债务问题继续恶化,德国希望争取中国为解决欧债危机提供更多的帮助。

28.让所有的实验鼠奔跑1小时。第一组实验鼠跑前1小时喝西红柿汁。第二组跑后喝西红柿汁。第三组奔跑到30分钟后喝西红柿汁,休息1时后再跑30分钟。对照组实验鼠只饮水。运动过后6小时测量实验鼠血液中标志动物疲劳的物质“TGF-b”的浓度,结果是:与只饮水的实验鼠相比,第一组和第三组实验鼠的这一指标减少50%至60%.而第二组实验鼠几乎没有差别。

以下哪一项最适合作为上述实验的结论?

A.饮用西红柿计可以消除运动引起的疲劳。

B.运动前饮用西红柿计可以减轻运动疲劳。

C.前3组实验鼠与只饮水的实验鼠是以同样的速度奔跑。

D.在运动强度和运动量相同的情况下,运动间隙中较长时间的休息可以减轻疲劳。

29.长期以来,床上抽烟是家庭火灾的主要原因。尽管在过去20年中,抽烟的人数显著下降,但死于家庭火灾的人数却没有显著减少。如果以下陈述为真,都有助于解释上面明显的不一致,除了

A.床上抽烟的人通常烟瘾很大,与那些不在床上抽烟的人相比,他们更不可能戒烟。

B.过去20年中人口密度一直在增加,现在一次家庭火灾造成的死亡人数比20年前的多。

C.由床上抽烟引起的火灾通常发生在房主入睡之后。

D.与其他类型的家庭火灾相比,床上抽烟引起的家庭火灾造成的损失通常较小。

30.有两类恐怖故事:一类描写疯狂科学家的实验,一类讲述凶猛的怪兽。在关于怪兽的恐怖故事中,怪兽象征着主人公心理的混乱。关于疯狂科学家的恐怖故事则典型地表达了作者的感受:仅有科学知识不足以指导人类的探索活动。尽管有这些区别,这两类恐怖故事具有如下共同特点:它们描述了违反自然规律的现象:它们都想使读者产生恐惧感。

如果以上陈述为真,以下哪一项一定为真?

A.对怪兽的所有描写都描述了违反自然规律的现象。

B.某些运用了象征手法的故事描述了违反自然规律的现象。

C.大部分关于疯狂科学家的故事表达了作者反科学的观点。

D.任何种类的恐怖故事都描写了,心理混乱的人物。

31.2012年入夏以来,美国遭遇50多年来最严重的干旱天气,本土48个州有三分之二的区域遭受中度以上旱灾,预计玉米和大豆将大幅度减产。然而,美国农业部8月28日发布的报告预测.2012年美国农业净收入有望达到创纪录的1222亿美元,比去年增加3.7%。

如果以下陈述为真,哪一项最好地解释了上述看似矛盾的两个预测?

A.2012年,全球许多地方遭遇干旱、高温、暴雨、台风等自然灾害。

B.目前玉米和大豆的国际价格和美国国内价格均出现暴涨。

C.美国农场主可以获得农业保险的赔款,抵消一部分减产的影响。

D.为应对干旱,美国政府对农场主采取了诸如紧急降低农业贷款利率等一系列救助措施。

32.仙客来是一种著名的观赏花卉,在气候炎热的地带很难生长。在干旱的地区很难种植水稻。在某个国家的大部分地区,或者仙客来很容易生长,或者很容易种植水稻。

如果以上陈述为真,以下哪一项陈述一定为假?

A.这个国家有一半的地区既干旱又炎热。

B.这个国家大部分地区的气候是寒冷的。

C.这个国家的某些地区既不炎热也不干旱。

D.在这个国家里不可能种植仙客来。

33.今年,华泰集团旗下的泰丰公司创下了销售新纪录。这颇令人感到意外,因为泰丰公司的潜在市场是最小的,而且它一直是华泰集团旗下所有分公司中销售额最低的部门。

以下哪一项陈述指出了以上论述存在的问题?

A.由于泰丰公司的潜在市场最小,它是华泰集团销售额最低的部门并不令人感到意外。

B.如果今年华泰集团的总销售额上升了,则泰丰公司的销售额增加并不令人感到意外。

C.每个分公司的销售额是否创销售纪录只是该公司自我比较的结果,不需要与其他分公司比较。

D.如果集团旗下其他分公司都创造了销售纪录,则泰丰公司创销售新纪录并不令人感到意外。

34.污水处理要消耗大量电力。美国某大学的研究人员最近开发出一项新的微生物电池技术,使污水产出电力的效率比原来提高了10至50倍。运用这项技术,污水处理厂不仅可以实现电力自给,还可将多余的电力出售。可以期待,一旦这项技术投入商业运作,企业对污水处理的态度会变得积极主动,从而减轻污水排放引发的环境污染。

对以下哪个问题的回答与上述判断的评估最具相关性?

A.采用这种方式进行污水处理的技术转让和设备成本会不会很高?

B.这种技术能否有效地处理化工厂污水中的重金属?

C.这种污水处理方式会不会因释放甲烷而造成空气污染?

D.环保部门是否会加大对企业排污情况的监管?

35.一则公益广告建议,喝酒的人应该等到能够安全开车时再开车。然而,一次医院调查发现,喝完酒后立即被询问的人低估了他们恢复开车能力所需的时间。这个结果表明,许多在开车前喝酒的人在遵从广告的建议方面有困难。

如果以下陈述为真,哪一项最强地支持以上论述2

A.如果有些人在喝酒之后必须回家,他们会忍住不喝酒。

B.许多打算喝酒的人会事先安排一个不喝酒的人开车送他们回家。

C.在医院中的被调查者也被问到恢复那些对安全驾驶影响不大的能力所需时间的长短。

D.与在医院外的被调查者相比,在医院中的被调查者对自己能力的估计要保守一些。

36.“安慰剂效应”是指让病人在不知情的情况下服用完全没有药效的假药,却能得到与真药相同甚至更好效果的现象。“安慰剂效应”得到了很多临床研究的支持。对这种现象的一种解释是:人对于未来的期待会改变大脑的生理状态,进而引起全身的生理变化。

以下陈述都能支持上述解释,除了

A.安慰剂生效是多种因素共同作用的结果。

B.安慰剂对丧失了预期未来能力的老年痴呆症患者毫无效果。

C.有些病人不相信治疗会有效果,虽然进行了正常的治疗,其病情却进→步恶化。

D给实验对象注射生理盐水,并让他相信是止痛剂,实验对象的大脑随后分泌出止痛物质内啡肽。

37.美国的枪支暴力惨案再度引发了枪支管控的讨论。反对枪支管控者称,上世纪80年代美国枪支暴力案飙升,1986年有些州通过法律手段实施严格的枪支管控,但实施严格枪支管控的这些州的平均暴力犯罪率却是其他州平均暴力犯罪率的1.5倍。可见,严格的枪支管控无助于减少暴力犯罪。

如果以下陈述为真,哪一项最强地削弱了以上论证?

A.自1986年以来,美国拥有枪支家庭的比例显著下降。

B.自1986年以来,实施严格枪支管控的这些州的暴力犯罪数持续下降。

C.在那些实施严格枪支管控法律的州,很少有人触犯该项法律。

D.犯罪学家对比了各种调查结果,并未发现私人拥有枪支的数量与枪支暴力犯罪有明显的相关性。

38.中秋节来临,某超市为了吸引消费者,推出了一次购物满500元赠送一盒月饼的促销活动。超市经理说,促销活动开始以来收银机单次进款500元以上的单子增加了近30%这表明促销活动很成功,达到了扩大市场份额的目的。

如果以下陈述为真,哪一项最有力地削弱了经理的断言?

A.习惯于小额购物的顾客不太会受促销活动影响。

B.有些在活动期间一次购物满500元的顾客,平时购物也总是高于500元。

C.促销活动中,大多数一次购物满500元的人是这家超市的长期顾客,他们增加单次购物的额度,却减少了购物的次数。

D.被促销活动吸引到该超市购物的顾客在活动结束后可能不会再来了。

39.营养学家:宣传任何一种保健品能治病都是骗人的。但是,对于饮食不规律的人群来说,服用某些保健品是必要的。如果接受该营养学家的看法,则必须接受以下哪一项陈述?

A.有些保健品是某些人有必要服用的。

B.已经生病的人不应服用保健品,因为保健品不能治病。

C.并不是所有的人都有必要服用保健品。

D.对于饮食规律的人群来说,服用任何保健品都是不必要的。

40.今年上半年,北京凯华出租汽车公司接到的乘客投诉电话是北京安达出租汽车公司的2倍,这说明安达出租汽车公司比凯华出租汽车公司的管理更规范,服务质量更高。

如果以下陈述为真,哪一项最能支持上述结论?

A.凯华出租汽车公司的投诉电话号码数不如安达出租汽车公司的多。

B.凯华出租汽车公司的投诉电话数量比安达出租汽车公司的上升得快。

C.安达出租汽车公司的在运营车辆是凯华出租汽车公司的2倍。

D.打给凯华出租汽车公司的投诉电话通常比打给安达出租汽车公司的投诉电话时间更长。41~45题基于以下共同的题干:

一位音乐制作人正在一张接一张地录制7张唱片: F、G、H、J、K、L和M,但不必按

这一次序录制。安排录制这7张唱片的次序时,必须满足下述条件:

(l)F必须排在第二位。

(2)J不能排在第七位。

O)G既不能紧挨在H的前面,也不能紧接在H的后面。

(4)H必定在L前面的某个位置。

(5)L必须在M前面的某个位置。

41.下面哪一项可以是录制这7张唱片从l到7的顺序?

A.F,K,G,L,H,J,M

B.G,F,H,K,L,J,M

C.G,F,H,K,L,M,J

D.K,F,G,H,J,L,M

42.如果M在J之前的某个位置和K之前的某个位置,下面哪一项一定是真的?

A.K第七

B.L第三

C.H或者紧挨在F的前面或者紧接在F的后面

D.L或者紧挨在G的前面或者紧接在G的后面

43.下面哪一项列出了可以被第一个录制的唱片的完整且准确的清单?

A.G , J , K B.G, H, J, K C.G, H,J, L D.G, J, K , L

44.录制M的最早的位置是

A 第一 B 第三 C 第四 D 第五

45如果G紧挨在H的前面但所有其他条件仍然有效,下面的任一选项都可以是真的,除了

A.J紧挨在F的前面 B.K紧挨在G的前面

C.J紧接在L的后面 D.J紧接在K的后面 46~50题基于以下共同的题干:

某校有7名优秀的学生G、H、L、M、U、W和Z。暑假期间,学校将派他们去英国和美国考察。该校只有这7名学生参加这次活动,每人恰好去这两个国家中的一个,考虑到每个

学生的特长,这次活动必须满足以下条件:

(1如果G去英国,则H去美国。

(2)如果L去英国,则M和U都去美国。

(3)w所去的国家与Z所去的国家不同。

(4)U所去的国家与G所去的国家不同。

(5)如果Z去英国,则H也去英国。

46.以下哪一项可以作为去英国的学生的完整且准确的名单?

A.G、H、M、W

B.G、L、Z

C.H、M、Z

D.M、U、W

47.以下哪两个人不能一同去美国?

A.H和W

B.G和W

C.G和H

D.M和U

48.最多可以有几个学生一起去英国?

A.2个

B.3个

C.4个

D.5个

49.如果M和W都去英国,则以下哪一项可以为真?

A.G和L都去英国

B.G和U都去美国

C.H和Z都去英国

D.L和U都去美国

50.如果G去美国,则以下哪一项一定为真?

A.H去英国

B.L 去美国

C.M 去英国

D.W去美国 逻辑推理能力测试

1.A 2.B 3.B 4.D 5.B

6.C 7.B 8.C 9.C 10.A

11.D 12.A 13.D 14.C 15.D

16.C 17.B 18.D 19.B 20.B

21.D 22.A 23.C 24.D 25.B

26.D 27.A 28.B 29.C 30.B

31.B 32.A 33.C 34.A 35.D

36.A 37.B 38.C 39.A 40.C

41.B 42.C 43.B 44.C 45.D 46.D 47.A 48.C 49.D 50.B

第一部分 语言表达能力测试

(50题,每题2分,满分100分)

一、选择题

1.下面各组词语,没有错别字的一组是

A.船坞 沉寂 人才倍出 浑水摸鱼

B.惊垫 没落 干均一发 无人问津

C.挑衅 振动 改弦更张 味同嚼蜡

D.匮乏 坚韧 迥迥有神 百战不殆

2.下面加点的词,意义相同的一组是

A.①整个建筑建造精美,典雅大方,又带有时代气息。

②易玛活泼大方,总是有说有笑,是个令人愉快的姑娘。

B.①临时政府实力单薄,根本无法控制首都以外的地区。

②有些章节内容单薄,一些问题虽提出,但未能展开充分的论证。

C.①学员们集中精力,端正学风,取得了比较好的学习成绩。

②南洋杉的树形端正、细长,姿态秀丽,是世界著名的观赏树木。

D.①张正一口纯正流利的英语配音,给观众留下了深刻的印象。

②政府正努力建设更多的知识园地,给孩子们以清雅纯正的文化环境。

3.下面各句,语义明确、没有歧义的一句是

A.他跟我去天津开过学术研讨会。B.这些人连厂里的领导都不认识。

C.他曾先后在深圳、广州搞调研。D.筐中苹果余下的一半给了老王。

4.下面各句,没有语病的一句是

A.那一刻,我们几个第一次看到他露出的真正属于破子般的灿烂笑容。

B.机动车进胡同,对机动车噪声是否只有被动接受,居民们早有微词。

C.我们的教育工作者积极发现人才、培养人才,这本身就是一种贡献。

D.殖民时代的业余运作方式,直接导致了澳门低忐平的竞技体育运动。

5.下面各句,加点成语使用不恰当的一句是

A.缅甸和我国接壤的地区,也就是臭名昭著的“金三角”地区,鸦片种植曾一度十分猖獗。

B.汽车越来越时尚化,连最保守的德国人都在汽车外形设计上做出了令人张口结舌的改变。

C.亚洲杯决赛的前30分钟,中国足球队的后防线险象环生,不断出现被对方突破的情形。

D.欧元下跌狂潮不仅使己陷入金融危机的欧洲经济雪上加霜,也给我国造成了负面影响。

6.下面诗句中,“锦书”一词意思与其他三句差别较大的一句是

A.青家曾无尺寸归,锦书多寄穷荒骨。(陈陶《关山月?)

B.云中谁寄锦书来?雁宇回时,月满西楼。(李清照《一剪梅?)

C.山盟虽在,锦书难托,莫!莫!莫!(陆游《钗头凤?)

D.上元锦书传宝字,王母琼箱荐金约。(王勃《七夕赋?)

7.下面女性形象,依次出现在《西厢记??桃花扇》和《长生殿》中的是

A.李香君 杨玉环 崔莺莺 B.崔莺莺 李香君 杨玉环

C.崔莺莺 杨玉环 李香君 D.杨玉环 崔莺莺 李香君

8.下面字母词与汉语释义不符的一组是

A.①GOP:国民生产总值 ②GNP:国内生产总值

B.①NBA:美国职业篮球联赛 ②FIFA:国际足球联合会

C.①APEC:亚太经济合作组织 ②OPEC:石油输出国组织

D.①WTO:世界贸易组织 ②WHO:世界卫生组织

9.“多头领导”是指由于组织结构设置不合理导致一个下级同时接受多个上级领导的现象。

下面俗语涉及这一现象的是

A.三个和尚没水吃 B.一朝君子一朝臣

C.狗拿耗子多管闲事 D.一山难容二虎

10.商品差价是指同一商品由于流通环节、购销地区、购销季节以及质量不同而形成的价格

差额。下面情形属于商品差价的是

A.白萝卡一元一斤,胡萝卡五角一斤 B.国产啤酒与进口啤酒销售价格不同

C.在广西买荔枝比在北京买便直得多 D.同名书籍的精装版比简装版贵五元钱

11.下面关于文史知识的表述,不正确的一项是

A.孔子是先秦思想家,其思想核心是“仁”。

B.王充是东汉思想家,代表作是《论衡》。

C.刘艇、是南朝文学理论家,代表作是《文心雕龙》。

D.王阳明是宋代理学家,他宣扬“心学”。

12.?侵权责任法》第13条规定“法律规定承担连带责任的,被侵权人有权请求部分或者全

部连带责任人承担责任”。现有甲、乙、丙三人需对丁承担连带责任。根据此规定,下面

表述中正确的一项是

A.丁只能要求甲、乙、丙三人中的一人承担责任

B.丁不能要求甲、乙、丙三人中的二人承担责任

C.丁不能要求甲、乙、丙三人共同承担责任

D.丁可以只要求甲承担责任

13.?合同法》第44条规定:“依法成立的合同,自成立时生效。”同时该法第210条规定:”自

然人之间的借款合同,自贷款人提供借款时生效。“结合此两条规定,下面合同在成立时

不能立即生效的是

A.甲公司与乙银行签订的借款合同 B.甲公司与丙信托公司签订的借款合同

C.公民丁与公民戊签订的借款合同 D.甲公司与公民丁签订的借款合同

14.光纤通讯中信号传播的主要载体是光导纤维,其内芯和外套材料不同,利用光从折射率

大的介质进入折射率小的介质时发生全反射的性质将大量信息高速传输。下面关于光导

纤维的说法,正确的是

A.内芯的折射率比外套的大,光传播时由内芯进入外套时发生全反射

B.内芯的折射率比外套的小,光传播时由内芯进入外套时发生全反射

C.内芯的折射率比外套的大,光传播时由外套进入内芯时发生全反射

D.内芯的折射率比外套的小,光传播时由外套进入内芯时发生全反射

15.风是相对于地表面的空气运动。形成风的直接原因,是气压在水平方向分布的不均匀。另 外,风还受地球自转、地形、水域等不同因素的综合影响。因此,形成风的原因不包括

A.气压梯度 B.海陆分布

C.地表摩擦力 D.地转偏向力

二、填空题

16.在下面横线处,填入上下文衔接最恰当的一段话。

在现代社会里,相信科学就是相信牢靠的一面,相信奇迹则是相信不牢靠的一面。

________

A.可惜的是,多数人并不了解这些严谨、乏味的科学知识是大家的安全和幸福所系,对

此缺少一种慎重和敬意。时值今日,全体人类的生存,都靠科学技术来保障。一旦老

百姓不听科学的招呼,生灵涂炭的大祸就在眼前。

B.可惜的是,多数人并不了解这些严谨、乏味的科学知识是大家的安全和幸福所系,对

此缺少一种慎重和敬意。一旦老百姓不听科学的招呼,生灵涂炭的大祸就在眼前。时

值今日,全体人类的生存,都靠科学技术来保障。

C.一旦老百姓不听科学的招呼,生灵涂炭的大祸就在眼前。时值今日,全体人类的生

存,都靠科学技术来保障。可惜的是,多数人并不了解这些严谨、乏味的科学知识是

大家的安全和幸福所系,对此缺少一种慎重和敬意。

D.时值今日,全体人类的生存,都靠科学技术来保障。可惜的是,多数人并不了解这些

严谨、乏味的科学知识是大家的安全和幸福所系,对此缺少一种慎重和敬意。一旦老

百姓不听科学的招呼,生灵涂炭的大祸就在眼前。

17.在下面横线处,填入对修辞方法及其作用的准确表述。

”凡走狗,虽或为一个资本家所拳养,其实是属于所有的资本家的,所以它遇见所

有的阔人都驯良,遇见所有的穷人都狂吠。“以上文字_______,表达了强烈的憎恶之情。

A.运用比喻手法,具体生动地表现了势利小人的卑鄙

B.运用比拟手法,具体生动地表现了势利小人的卑鄙

C.运用比拟手法,形象生动地反映了资本家帮凶的本质

D.运用比喻手法,形象生动地反映了资本家帮凶的本质

18.在下面文字中的横线处,依次填入最恰当的关联词语。

中国人和日本人还是不同,______中国人和日本人的不同,在外表上很不容易看出来。

_______每一个国家的国民,都有它特别的遗传和环境,________自然就有了他的国民性。由这一点来讲,______不能理解一国的国民性,就很难欣赏一国的文学。

A.而且 再说 因此 既然 B.只是 因为 所以 如果

C.只是 再说 因此 如果 D.而且 因为 所以 既然

19.在下文横线处,依次填入最恰当的标点符号。

当今社会______人情风和关系风正污浊着人们的心灵_______金钱多少表示人情的轻重

________礼品贵贱显示关系的亲疏________

A.。!B。。

C.,:。D.,,!

20.在下面各句横线处,依次填入最恰当的词语。

①外表大大咧咧的她,其实内心里深藏着不为人知的丰富_______的感情。

②经过_______的调查研究,这家公司最终选择上海作为中国市场切入点。

③中国四大名绣之一的苏绣,向来以其工艺精湛、针法_______闻名世界。

A.细密 细腻 细致 B.细腻 细致 细密

C.细腻 细密 细致 D.细致 细密 细腻

21.王国维在《人间词话》中把境界分为”有我之境“与'?无我之境飞他说,”有我之境,以我现物,故物皆著我之色彩。无我之境,以物现物,故不知何者为我,何者为物。“下

面四个词句,______最具”无我之境“的特点。

A.鱼市孤烟袅寒碧,水村残叶舞愁红 B.街南绿树春绕絮,雪满游春路

C.泪眼问花花不语,乱红飞过秋千去 D.双燕归飞绕画堂,似留恋虹梁

22.在下面横线处,依次填入正确选项。

诞生几个重要的文学社团。_______的创作主张倾向于现实主义,_______主

张”为艺术而艺术“,______提倡新格律诗和”国剧“运动。

A.文学研究会 创造社 新月社 B.创造杜 文学研究会 新月社

C.文学研究会 新月社 创造社 D.新月社 创造社 文学研究会

23.”理论总是灰色的,而生命之树常青“,歌德的这句话不直用来说明_______的道理。

A.纸上得来终觉浅,绝知此事要躬行 B.践是检验真理的唯一标准

C.理论必须联系实际 D.理论应放之四海而皆准

24.博弈思维,是指由于事情的结果取决于自己和他人的策略选择,因此要理性分析各种各

选策略及其后果的可能性,从而使受益最大化的一种思维方式。_______反映了博弈思维。

A.塞翁失马,焉知非福 B.兵来将挡,水来土掩

C.种瓜得瓜,种豆得豆 D.城门失火,殃及池鱼

25.2012年7月24日,海南省地级三沙市在东沙群岛永兴岛挂牌成立。根据中央军委 的批复,三沙_______随之成立,隶属于海南省军区。

A.警备区 B.守备区

C.军分区 D.海监局

26.汉语有一系列用于表达称呼的”令~“类词,”令嫂“一词是对_______的称呼。

A.对方的女儿 B.对方的妻子

C.自己的女儿 D.自己的妻子

27.根据《宪法》规定,全国人民代表大会审议通过《中华人民共和国合同法?,应当由全体

代表_______通过

A.一致 B.过五分之四

C.过四分之三 D.过二分之一

28.苏格拉底教学法又称为 _______

A.问答法 B.讲授法

C.发现法 D.雄辩法

29.神舟九号飞船在2012年6月18日14时左右与天宫一号实施自动空间交会对接。空间交

会对接是指两个航天器在空间轨道上会合,并在结构上连成一个整体的技术。在交会对

接过程中,追踪飞行器的飞行将首先进入_______阶段,在地面测控的支持下,追踪飞行器

经过若干次变轨机动,进入到其敏感器能捕获目标飞行器的范围。

A.近程导引 B.远程导引

C.最终逼近D.对接停靠

30.冷暖气团交界面叫锋面。在锋面移动过程中,根据冷暖气团所占的主次地位不同,可以

将锋分为冷锋、暖锋等类型。冷锋过境时多出现雨雪、大风、降温等剧烈的天气变化,而暖锋过境时容易形成连续性降水或下雾、升温等天气状况。根据以上描述可知,”忽

如一夜春风来,千树万树梨花开“描述的是_______过境时的天气:”清明时节雨纷纷,路

上行人欲断魂“描述的是_______过境时的天气。

A.冷锋,暖锋 B.冷锋,冷锋

C.暖锋,暖锋 D.暖锋,冷锋

三、阅读理解题

(一)阅读下面短文,回答问题

所谓制艺,是指自末以来考试的文章而言。在唐时考试用诗;末时改为经义,即从四书或五经内出一题目,由考的人作一段文章,其形式全与散文相同;到明代便有了定型:文章的起首是破题,其次是承题,再次是起讲,后面共有八股,每两股作为一段,此平彼仄,两两相对。下面再有一段作为结尾。这便是所谓八股文。到明末清初时候,更加多了许多限制,不但有一定的形式,且须有一定的格调。这样,越来便越麻烦了。

为什么会有八股文这东西起来呢?据我想这与汉字是有特别关系的。汉字在世界上算是

最特别的一种,它有平仄而且有偏旁,_______可找些合适的字使之两两互对起来。例如”红花“可用”绿叶“作对,_______用”黄叶“或”青枝“等去对,_______小学生也知其不合适,因为”红花“和”绿叶“,不但所代表的颜色和物件正好相对,字的平仄也是正对的,_______红绿二字还都带有”牟“旁,其它的”青枝“"黄叶”等便不足这些条件了。

从前有人路过一家养马的门口,见所贴门联的一幅是“左手牵来千里马”,觉得很好,但及至看到下幅,乃是“右手牵来千里驹”,又觉得很不好了。这在卖马的人只是表示他心中的愿望,然而看门联的人则以为应当对得很精巧才成,仿佛“千”定要对“万”或“手”定妥对“足”才是。

这样子,由对字而到门联,由门联而到挽联,而到很长的挽联,便和八股文很接近了。

中国打“灯谜”的事也是世界各国所没有的,在中由各地方各界却都很普遍。譬如“人人尽道看花田”,打四书一句.“言游过矣”,又如“传语报平安”打“言不必信”等等,意思尽管是牵强附会,但倒转过来,再变化得高级一些,便成为八股文中破题的把戏,因此,我觉得八股文之所以造成,大部分是由于民间的风气使然,并不是专因为某个皇帝特别提倡的缘故。

关于破题有很多笑话,但虽是笑话,其作法却和正经的破题完全相同。如有人以极通俗的话作破题解释“三十而立”说?“两当十五之年,虽有椅子板凳而不敢坐也。”另外要举一正经的例子:题目是“子曰” 有人的破题是“匹夫而为百世师,一言而为天下法”。

从这些例子看来,便很可以明白,低级的灯谜,和高级的破题,原是同一种道理生出来的。

“破题”之后是“承题”,承题的起首必须得用一“夫”字,例如,要接着前面所举“三十而立”的破题作下去,其承题的起首一定是“夫椅子板凳所以坐者也„„”一类的话头。

总之,作文章的人,处处都受有限制,必须得模仿当时圣贤说话的意思,又必须遵守形式方面的种种条规。作一篇文章消磨很多的时间,作成之后却毫没价值。

八股文中的声调也是一件很主妥的成分。这大概是和中国的戏剧有关系的事。中国的歌曲早已失传,或者现在一般妓女所唱的小妈还有些仿佛吧,然而在民间已不通行。大多数国民的娱乐,只是在戏剧方面。现在各学校所常举行的蒋艺会欢迎会之类,在余兴一项内也大半都是唱些旧剧,老百姓在种地的时候,或走路害怕的时候,也都好唱几句皮簧之类,由此可见一般人对于戏剧的注意点是在于剧词的腔调方面。当我初到北京时是在光绪三十年顷,在戏院里见有许多当时的王公们,都脸朝侧面而不朝戏台,后来才知道这是因为他们所注意的只是唱者的音调如何,而不在于他们的表演怎样。西皮二簧甚至昆曲的词句,大半都作得不好,不通顺,然而他们是不管那些的,正如我们听西洋戏片,多半是只管音调而不管意思的。这在八股文内,也造成了同样的情形,只要调子好,规矩不错,有时一点意思也没有,都可以的。

总括起来,八股文和试帖诗都一样,其来源:一为朝廷的考试,一为汉字的特别形状,而另一则为中国的戏剧。其时代可以说自宋朝即已开始,无非到清朝才集其大成罢了。

〈节选自周作人《清代文学的反动(上)?,见《周作人散文全集?(6),广西师范大学出版社2009)

31.根据文中所述,八股文起源于

A.唐代 B.宋代

C.明代 D.清代

32.根据上下文,在第二自然段划线处依次填入最恰当的关联词语。

A.于是 又 所以 何况 B.并且 若 即使 何况

C.于是 若 即使 而且 D.并且 又 所以 而且

33.作者认为“打灯谜”和八股文的破题有相通之处,是因为“打灯谜”

A.难免牵强附会 B.可以变化得高级一些

C.是中国独有的 D.是对语句的设法解读

34.根据文中所述,与八股文的产生完全无关的是

A.中国的歌曲早已失传 B.某个皇帝特别提倡

C.对戏剧腔调格外关注 D.汉字分平仄有偏旁

35.通观全文,八股文之所以“作成之后却毫没价值飞是因为

A.破题曾有很多笑话 B.讲究两股平仄相对

C.写作上有许多限制 D.只重形式不重内容(二)阅读下面短文,回答问题

亚洲银行近日发布一份名为”亚洲和太平洋地区2010年关键指标“的报告,其中宣布,按照每日消费2-20美元的标准,亚洲”中产阶级“的人数早在2008年就已达到19亿。而按照这一标准,中国的中产阶级人数居然达到惊人的8.17亿。被好事者呼吁良久的”橄榄型社会居然就这么实现了。

不过,比亚洲银行名头更响的世界银行,曾经颁布过一个“贫困标准”,即每人每天收入低于1.25美元,就叫“绝对贫困”。当然“绝对贫困”并非如字面解读的那种“赤贫”状态,而是相对于“相对贫困”而言。也就是说不管在任何国家或地区,低于这个收入水平就是生活在贫困之中,而与他在社会中的相对住直元关。比如说在那些整体贫困的国家,即使是其中的中等收入阶层,其实际生活也可能在“绝对贫困”标准之下;反之在那些发达国家,即使是少数“相对贫困”人口,其实际生活水平也往往高于那些贫困国家的富裕阶层。

明乎此,亚洲银行把亚洲和中国的“中产阶级”的低限标准确定为仅仅高于“绝对贫困”标准0.75美元,实际是看低了整个亚洲的经济发展水平一一只有在一个整体收入水平低下的地区,其“中等收入”水平,才可能以2美元为起点。判断亚洲银行看低了中国人民的收入水平,自有中国的数据为证一一-每人每日消费2美元,大致折算为14元人民币,每月消费则为420元人民币。这个数字不妥说远远低于北京、上海这样的大城市,甚至竟远远低于中国大多数地区的最低工资保障水平。

但是有关中国中产阶级的标准和数量,从来都是一笔糊涂账。2005年,国家统计局根据抽样调查,给过一个中产阶层的收入标准一一-年收入在6万元到50万元之间的家庭,就是当今中国的“中产样的大城市,月收入在6000元左右的人或月收入为10000元左右的家庭,就可归入”中产“。虽然上述报告给出的结论都有根有据,但还是元一例外地,受到冷嘲热讽,几乎所有被划入”中产“的白领们,都在控诉”被中产“的冤屈和真实生活的窘迫和无奈。

但笔者认为,研究者的结论与被研究者的感受之间的巨大差异,倒并不一定是研究者的阴谋,而缘于对”中产“的定义不同。在研究者看来,”中产“只是一个相对概念,那些收入处于高、低之间的阶层,就是”中等收入阶层”。而在自认“被中产”的白领心中,“中产”则有一个被美国“中产阶级”定义和描述过的模板——稳定优厚的收入、庸常但优雅的品味、保守但理性的立场„„当无数中国“中等收入”的群体,正处在追求着这种生活的焦虑之中,却被宣布已经成为“中产”,其愤懣和怀疑是可想而知的——如果我们当下的生活就是中国的中产阶层,那我何时才能过上美国的中产生活?

其实一个元情的、显见的、确定的结论,却总是被中国的中产阶层拒绝承认,那就是中国的中产阶层,永远不可能过上美国中产阶级那样的生活。如果中国确如美国那样,变成一个中产阶层占多数的社会则中国的中产阶层队伍确乎要有近8亿人口。而人类绝不可能再拥有第二个地球,提供给这8亿中国中产阶层每家一栋别墅、两辆汽车。这结论不是被认为心怀巨测的中国研究者说的,而是奥巴马说的一一-这回你总得信了吧?

所以,尽管世界已经越来越“平”,尽管那个“美好”的美国就在天际线的尽头眨着鬼魅而诱惑的眼睛,但中国的“中产”们还是不得不把饥渴的眼睛,从美国“榜样”身上移开。中国的“中产”,由中国的社会发展水平和潜在的资源禀赋、发展空间定义,却与美国的中产阶级没有太多的相干。

(选自胡冉《难缠的“中产吟,?北京青年报?,2010年9月5日)

36.文章说有人认为某些研究者”心怀臣测“,其原因不包括

A.故意夸大了中国中产阶层的数量

B.未能关注中等收入群体追求中产的焦虑

C.无视被划入中产的白领真实生活的窘迫和无奈

D.由于缺乏客观依据才得到一笔糊涂账

37.根据文章的内容,在中国的研究者看来,中国的”中产阶层“不是指

A.收入处在中国高、低收入之间的人群

B.至少拥有一栋别墅和两辆汽车的家庭

C.月收入6000元到10000元之间的家庭

D.年收入在6万元到50万元之间的家庭

38.下面词语,与”尽管世界己经越来越‘平,“一句中”平“的意思最接近的是

A.平和 B.平衡

C.平均 D.平稳

39.根据作者的观点,”中产阶级(阶层)“是依据_______划分的阶层。

A.相对贫富标准 B.绝对贫富标准

C.美国模式标准 D.世界统一标准

40.以下对本文主旨的理解,正确的一项是

A.在现代社会中,”中产阶级“是社会的主要群体,要大力建设”橄榄型“社会

B.在中国,44中产阶级”现在不是主要群体,也永远不可能成为社会的主要群体

C.“中产阶级”是有统一标准的概念,不因国家发展水平的不同而制定不同标准

D.中国要按美国模式产生“中产阶级”,既不合中国的实际,也根本不可能实现

(三)阅读下面短文,回答问题

偏见可以说是思想的放假。它是没有思想的人的家常日用,而是有思想的人的星期日娱乐。假如我们不能怀挟偏见,随时随地必须得客观公平、正经严肃,那就像造屋只有客厅没有卧立,又好比在浴室里照镜子还得做出摄影机头前的姿态。魔鬼在但丁《地狱篇》第二十七句中自称.“敝魔生平最好讲理。”可见地狱之设,正为此草;人生在世,言动专求合理,大可不必。当然,所谓正道公理压根儿也是偏见。依照生理学常识,人心位直,并不正中,有点偏侧,并且时髦得很,偏倾于左。古人称偏僻之道为“左道”,颇有科学根据。不过,话虽如此说,有许多意见还不失禅宗所谓“偏中正”,例如学术理论之类。只有人生边上的随笔、热恋片的町等等,那才是老老实实、痛痛快快的一偏之见。世界太广漠了,我们圆睁两眼,平视正视,视野还是偏狭得可怜。狗注视着肉骨头时,何尝顾到旁边还有码呢?三于通常所谓偏见,只好比打靶的瞄准,川眼来看。但是,也有人'以为这倒是瞄中事物且心的看法。譬如说,柏拉图为人类下定义云:“人者,元羽毛之两足动物也。”可谓客观极了!但是按照来时铁斯《哲人言行录》六卷二章所载,偏有人拿着一只拔了毛的鸡向柏拉图去质问。偏激二字,本来相连;我们别有所激,见解当然会另有所偏。假使我们说“人类是不拘日夜,不问寒暑,发出声音的动物。”那又何妨?

禽喘于春,萤啼于秋,坟作雷于夏,夜则虫醒而鸟睡,风雨并不夭夭有,元来人犬不吠,不下蛋鸡不报。惟有人用语言,用动作,用机械,随时随地做,出声音。就是独处一室,与酬答的时候,他可以开留声机,听无线电,甚至睡眠时还发f±:似雷的鼻息。语言当然不就是声音;但是在不中听,不愿听,或着隔着墙壁和距离听不真的语言里,文字都丧失了圭角和轮廓,变成一固忽涨忽缩的喧闹,跟鸡鸣文吠同样缺乏意义,这就是所谓人籁!断送了睡眠,震断了思想,培养了神经衰弱。

这个世界毕竟是人类主宰管领的。人的声音胜过一切。聚合了大自然的万千喉舌,抵不上两个人同时说话的喧哗,至少从第三者的耳朵听来。人箱是寂静的致命伤,天籁是能和寂静溶为一片的。每日东方乍白,我们梦已回而困永醒,会听到无数禽声,向早晨打招呼。那时夜未全消,寂静还逗留着,来庇荫未找清的睡梦。数不清的麻斗室的呜噪,琐碎得像要啄破了这个寂静:乌鸪的声音清利像把剪刀,老藕鸟的声音滞涩而有刺像把锯子,都一声两声地向寂静来试锋口。但是寂静似乎太厚实了,又似乎太流动了,太宫于弹性了,给禽鸟啼破的浮面,立刻就填满。雄鸡引吭悠扬的报晓,也并未在寂静上划下一道卢迹。慢慢地,我们忘了鸟喘是在破坏寂静:似乎寂静已将鸟语吸收消化,变成一种有声音的寂静。此时只要有邻家小儿的啼哭,楼上睡人的咳嗽,或墙外早行者的脚步声,寂静就像宿雾见了朝阳,破裂分散得干净。人籁己起,人事复始,你休想更有安顿。在史l到身倦,或苦思冥想时,忽闻人籁嘈杂,最博爱的人道主义者也许有时杀心顿起,恨不能天口以博耳根清净。禽兽风涛等一切天赣能和寂静相安相得,善于体物的古诗人早已悟到。《诗经》:“萧萧马鸣,悠悠祷施”,下文就说明“有闻元声”;可见马嘶而元人喊,不会产生喧闹。《颜氏家训》也指出王籍名句“埠嗓林愈静,鸟鸣山史幽”,就是“有闻无声”的感觉;虫鸟鸣噪,反添静境。雪莱诗里,描写啄木鸟,也说鸟'系山史幽。柯律立治《风瑟》诗云.“海声远且幽,似告我以静。”假使这个海是人海,诗人非耳聋头痛不可。所以我们常把“鸦呜崔嗓”来比人声喧哗,还是对人类存三分回护的曲笔。常将一群妇女的说笑声比于“莺啼燕语”,那简直是对于禽类的侮辱了。

闹与热,静与冷,都有连带关系;所以在_______的地狱里,太阳也给人以_______之感。人声_______,冷屋会变成热锅,使人通身_______。叔本华《哲学小品》第二百七十八节中说,思想家应当耳聋,大有道理。因为耳朵不聋,必闻声音,声音热闹,头脑就很难保持冷静,思想不会公平,只能把偏见来代替。那时候,你忘掉了你自己也是会闸的动物,也曾嚷嚷以致隔壁的人不能思想和睡眠,你更顾不得旁人在说你偏见太深,你又添了一种偏见,又在人生边上注了一笔。

(节选自钱钟书《一个偏见?,见《写在人生边上)),三联书店2002)

41.下面符合第一段文意的观点是

A.偏见是思想的副产品 B.没有思想的人没有偏见

C.学术理论难免带有偏见 D有思想的人不能怀挟偏见

42.文中论说“人类是不拘日夜,不问寒暑,发出声音的动物”的用意在于

A.比较“人籁”和“天籁”,抨击人类生活的不自然

B.以“人籁”的嘈杂反衬“天籁”的生动,讽刺人性

C.借以对“天籁”和“人籁”进行褒贬取舍

D.借以说明“别有所激,见有所偏”的道理

43.选择恰当的词语,依次填入第四段的横线处

A.阴惨寂寥喧杂烦躁 B.寂寥阴惨喧杂烦躁

C.阴惨喧杂寂寥烦躁 D.寂寥阴惨烦躁喧杂

44.作者在文中常常引用他人的话来表达意见,这样做的主要作用在于

A.使文章更加通}I顶 B.使文章更有文采

C.使文章更有说服力 D.使文章更加艰深

45.下面对文章内容的理解和概括,不恰当的一项是

A.人生在世,言动不必专求合理

B.“人籁”具有唤起人类、激发思想、振奋精神之效

C.“天籁”能溶于寂静的原因在听者“有闻无声”

D.保持冷静是防止思想产生偏见的条件之一(四)阅读下面短文,回答问题

在4月22日第43个世界地球日到来前,?科学》发表的一篇文章,把人们关注气候变化与二氧化碳正在排放的视角从大气带到了海洋。这篇文章宣称,由来自不同大学的21位研究人员组成的科研小姐,经过检测和评估地质记录后,得出结论:目前的海洋酸化速度是3亿年来的最高值。

海洋酸化是指由于吸收大气中过量的二氧化吸导致海水逐渐变酸的过程。人类活动向大气释放的二氧化钱,以每小时100万吨以上的速率被海洋吸收,在吸收过程中,二氧化碳与水反应释放出氢离子,使得海水的pH值下降。海水应为弱域性,海洋表层水的pH值约为8.2。但到2012年,海水表层pH值降低了0.1。海水酸性的增加,会改变海水的种种化学平衡,使多种海洋生物乃至生态系统面临巨大威胁。因此,除了全球变暖,海洋酸化被称为与二氧化破排放相关的另一重大环境问题。

一份来自夏威夷附近海域20年的数据显示,工业革命以来,海水表层pH佳从1960年的8.15下降到8.05,这表示,海水中氢离子浓度增加了30。

不仅如此,海洋酸化的速度也越来越快。最近一项研究表明,海表吸收二氧化碳的速率及其所导致的海洋酸化速率比两万年前的末次冰期快了近100倍,而末次冰期被认为是最近一次的二氧化碳急剧上升期。根据政府间气候变化专门委员会CIpCC)的预测,如按照目前二氧化破排放量的水平进行,本世纪末,海水pH值将下降至7.8左右。

研究表明,在二氧化碳浓度加倍以后,大多数钙化生物的钙化速率均大幅下降。钙化速率的下降,不仅影响到了浮游性钙化生物,如颗石藻等向底层海洋的破输送,还会影响到钙

化动物的生长和发育。同时,珊湖藻以及造礁珊湖种类在加倍的二氧化项条件下,其钙化速率平均下降30。

多项研究表明,海洋酸化还可能通过食物链,造成原本不同种间的配子受精成功并形成杂交种,造成种质混乱,影响物种间的相王作用及生态系统的稳定性。例如,某些浮涝动物在喂食酸化海水中生长的浮将植物后,繁殖率显著下降。

有研究者在模拟实验中发现,严重酸化的海水中,小丑鱼幼鱼将失去听力、视力、嗅觉,元法发现敌害,也丧失了相应的逃逸和生存能力。对甲壳类、贝类、鱼类及林皮动物等海水养殖生物的研究表明,海洋酸化会显著地影响到幼体发育,降低成体的钙化率和呼吸活动,改变机体能量代谢方式,干扰感知和运动行为,抑制免疫防御系统的活性,引起生物体代谢异常、生长缓慢甚至死亡。已经有研究表明,海洋酸化是一种生理胁迫,会使得藤壶的成活率显著下降,二氧化破浓度升高同样会导致海胆的尺寸和重量均明显变小。

由于人类从未经历过这种变化,并且不同种类的海洋生物对酸化引起的海水化学变化敏感性不同,以至于元法确定海洋酸化的生物学效应,也就无法预测未来可能发生的变化。

(节选自游雪晴《越来越酸的海洋越来越迷的未来?,中国科技网,2012年4月22日)

46.下面关于海洋酸化的理解,正确的一项是

A.海水一旦吸收空气中的二氧化碳,即导致海洋酸、化

B.海水应为弱碱性,海洋酸化是指海水呈弱酸性或酸性

C.二氧化碳与水反应释放出氢离子,使海水的pH值升高

D.洋酸化使海水酸性增加,从而影响海水中的化学平衡

47.下面关于海洋酸化速率的描述,正确的一项是

A.夏威夷附近海域数据显示,20年来海水表层pH值下降了0.1

B.海洋酸化速率越来越快,己达到末次冰期的100倍

C.目前,海水中的氢离子浓度是1960年的1.3倍

D.到本世纪末,海水的pH值一定会在7.8左右

48.海洋酸化可能造成的后果是

A.某些浮游动物繁殖能力下降 B.成鱼依然具备较好的逃逸能力

C.海洋生物代谢能力提高 D.贝类和棘皮动物御敌能力不变

49.为减少二氧化碳排放,降低海洋酸化速度,下面措施不正确的是

A.购物循环使用环保袋 B.避免使用一次性餐具

C.使用面巾纸代替布于帕 D.提倡自行车、步行出行

50.下面表述符合文意的一项是

A.所有的浮游动物在喂食酸化海水中生长的浮游植物后,繁殖率都会显著下降

B.珊珊藻以及造礁珊珊在加倍的二氧化碳条件下,其钙化速率将会平均下降30

C.目前海洋酸化速度,是3亿年前的130

D.人类从未经历过海洋急剧酸化的过程,但己可以用科学预测未来变化 第一部分 语言表达能力测试

01C,02B,03C,04C,05B,06D,07B,08A,09A,10C;

11D,12D,13C,14A,15C,16D,17C,18B,19C,20B;

21B,22A,23D,24D,25A,26A,27D,28A,29B,30A;

31C,32C,33B,34A,35D,36D,37B,38C,39A,40D;

41A,42D,43A,44C,45B,46D,47B,48A,49C,50B.

第四篇:2014年在职硕士GCT逻辑考试真题

新东方在线在职硕士频道(http://shuoshi.koolearn.com/zzlk/)分享

2014年在职硕士GCT逻辑考试真题

1、教授:美国和加拿大等国早就招收写作学的硕士生、博士生了,而我们还在为争取写作学的学位授予权而竭力呼吁。这就是对应用性、技能性学科两种截然不同的态度。是我们错了,还是人家错了?

以下哪一项所表述的问题对评估上述论证的合理性最为重要?

A.如果允许我们招收写作学的硕士和博士,有多少人会报考这个专业?

B.我们在写作学的师资和学科研究水平上是否具备招收硕士和博士的条件?

C.我们在写作学以外的其它应用性、技能性学科是否招收了硕士和博士?

D.我们是否应该重视对应用性、技能性学科硕士和博士的培养?

2、一位音乐家在通讯商店买手机号时,发现一个号码比其它的便宜一半还多,就问店员:“这是为什么?”店员说: “这个号不好。”原来这个号的尾数是1414,很多人忌讳它的谐音“要死要死”,可这位音乐家开口一唱,却是哆发哆发,其谐音是“都发都发”,非常高兴地买了这个号码。

如果以上陈述为真,不能支持以下哪一项陈述?

A.对数字谐音所暗示含义的忌讳或悦纳是愚蠢的.B.数字谐音所暗示的含义会影响购买号码的行为.新东方在线在职硕士频道(http://shuoshi.koolearn.com/zzlk/)分享

C.人们可以将不同的思想附加到同样的数字上去.D.数字与它的谐音所暗示的含义并不具有唯一性.3、在南非的祖鲁兰,每17个小时就有一头犀牛被偷猎。“飞翔的犀牛”行动从乌姆福洛奇保护区精心挑选了114头白犀牛和10头黑犀牛,将它们空运到南非一个秘密的地区,犀牛保护者希望犀牛能在这里自然地繁殖和生长,以避免因偷猎而导致犀牛灭绝的厄运。

以下哪一项陈述不是“飞翔的犀牛”行动的假设?

A.对犀牛新家的保密措施严密,使偷猎分子不知道那里有犀牛。

B.给犀牛人为选择的新家适合白犀牛和黑犀牛的繁殖和生长。

C.住在犀牛新家附近的居民不会有人为昂贵的犀牛角而偷猎。

D.60年前为避免黑犀牛灭绝而进行的一次保护转移行动获得成功。

4、科学家:就像地球一样,金星内部也有一个炽热的熔岩核,随着金星的自转和公转会释

放巨大的热量。地球是通过板块构造运动产生的火山喷发来释放内部热量的,在金星上

却没有像板块构造运动那样造成的火山喷发现象,令人困惑。

如果以下陈述为真,哪一项对科学家的困惑给出了最佳的解释?

A.金星自转缓慢而且其外壳比地球的薄得多,便于内部热量向外释放.新东方在线在职硕士频道(http://shuoshi.koolearn.com/zzlk/)分享

B.金星大气中的二氧化碳所造成的温室效应使其地表温度高达485℃.C.由于受高温高压的作用,金星表面的岩石比地球表面的岩石更坚硬.D.金星内核的熔岩运动曾经有过比地球的熔岩运动更剧烈的温度波动.5、达里湖是由火山喷发而形成的高原堰塞湖,生活在半咸水湖里的华子鱼——瓦氏雅罗鱼,像生活在海中的鲑鱼一样,必须洄游到淡水河的上游产卵繁育。尽管目前注入达里湖的4条河流都是内陆河,没有一条河流向海洋,科学家们仍然相信:达里湖的华子鱼最初是从海洋迁徙来的。

以下哪一项陈述为真,对科学家的信念提供了最佳的解释?

A.生活在黑龙江等水域的雅罗鱼比达里湖的瓦氏雅罗鱼个头大一倍.B.捕捞出的华子鱼放入海水或淡水中只能存活

一、两天,死后迅速腐坏。

C.冰川融化形成达里湖,溢出的湖水曾与流入海洋的辽河相连。

D.科研人员将达里湖华子鱼的鱼苗放入远隔千里的柒盖淖,养殖成功.6、保护野生动物种群的法律不应该强制应用于以捕获野生动物为生却不会威胁到野生动物种群延续的捕猎行为。

如果以下陈述为真,哪一项最有力地证明了上述原则的正当性?

A.对任何以营利为目的而捕获野生动物的行为,都应该强制执行野生动物保护法。

新东方在线在职硕士频道(http://shuoshi.koolearn.com/zzlk/)分享

B.尽管眼镜蛇受到法律保护,由于人的生命安全受到威胁而杀死眼镜蛇的行为不会受到法律的制裁。

C.极地最北端的因纽特人以弓头鲸为食物,每年捕获弓头鲸的数量远远低于弓头鲸新成活的数量。

D.人类猎杀大象有几千年了,并未使大致种群灭绝,因而强制执行保护野生象的法律是没必要的。7、2014年多名明星因涉毒被警方抓获。8月13日,北京市演出行业协会和各大演出公司签订了《北京市演艺界禁毒承诺书》,承诺不录用、不组织涉毒艺人参加演艺活动。对于这种做法涉嫌就业歧视的质疑,某律师回应说: “这不存在就业歧视,因为还有很多别的职业可以选择。”

以下哪一项指出了该律师回答中存在的逻辑问题?

A.按该律师的说法.任何搞就业歧视的人都可以像他这样为自己辩护.B.该律师没有考虑到,—个受过多年职业训练且只擅长班演的人转行是很困难的.C.该律师的回答与国务院戒毒条例关于“戒毒人员在就业方面不受歧视”的规定不—致.D.该律师错误地假定演艺业要比其他行业具备里高的职业道德水准.新东方在线在职硕士频道(http://shuoshi.koolearn.com/zzlk/)分享

8、当一位演艺界明星受到偷逃个人所得税的控告时,她为自己辩护说:“多年来,我已经交纳了上百万元的个人所得税,比我表妹所在的国营机械厂所交的税还要多。难道这也是罪过吗?”

以下哪一项陈述最准确地指出了上文辩护中的缺陷?

A.个人所得税交的越多证明她越富有,这样的人偷税漏税应当受到重罚.B.一个人交税总额的多少并不能证明她在每一项收入上都交纳了应缴的税额.C.部分所具有的属性通常不为由之所构成的整体所具有.D.个人交的税比国营机械厂交的多,不意味着她对社会的贡献比国营机械厂大.9、某班学生全都是足球迷。在2014年世界杯足球赛期间,所有支持阿根廷队的学生都不支持德国队,凡是支持阿根廷队的学生也都不支持巴西队。有些支持美国队的学生支持阿根廷队,有些支持美国队的学生支持德国队,有些支持美国队的学生支持巴西队。

如果以上陈述为真,以下哪一项关于该班学生的陈述必然为真?

A.有些支持德国队的学生既不支持巴西队,也不支持美国队。

B.有些支持美国队的学生既不支持德国队,也不支持巴西队。

C.所有支持美国队的学生或支持阿根廷队,或支持德国队,或支持巴西队.D.有些支持巴西队的学生支持德国队。

新东方在线在职硕士频道(http://shuoshi.koolearn.com/zzlk/)分享

10、据统计,目前全球每年死于饥饿的人数高达千万。而中国人每年在餐桌上浪费的粮食约为800万吨,相当于1亿人一年的口粮。这意味着,如果我们能有效杜绝餐桌上的食物浪费,就能够救活千百万的饥民。

如果以下陈述为真,哪一项最适合用来质疑上述结论?

A.恶劣气候导致粮食价格波动,加剧了饥饿问题.B.当今农业发展的水平已经能够从总量上保障全世界的人免于饥饿.C.消费能拉动经济发展,富人帮助穷人的途径之一是增加消费.D.杜绝食物浪费只是解决饥饿问题的有利条件.11、某网友:几乎没有人会支持损害他们利益的提案。然而据《京华时报》报道,在历时17

天的“我为公共交通价格改革建言献策”活动中,参与活动的上万市民多数赞同上调地铁和公交票价。让大家多掏钱还点赞,这个结果一定是假造的。

如果以下陈述为真,除哪一项外,都质疑了该网友的判断?

A.只有对公共交通价格改革感兴趣的人才参与活动,统计结果仅仅反映了这一部分人的意见。

B.参与活动的一部分人很少乘坐地铁和公交,上调票价并不损害他们的利益。

C.有些乘坐地铁和公交上下班的人认为上调票价能缓解高峰时段的拥挤状况。

新东方在线在职硕士频道(http://shuoshi.koolearn.com/zzlk/)分享

D.很多市民都希望通过上调票价来改善乘车环境。

12、产能过剩,地方政府债务、房地产泡沫是中国经济面临的3大顽疾,如果处理不当,则可能导致中国经济硬着陆。3大顽疾形成的根本原因,是中国长期的资本低利率。只有让资金成本回归到合理的位置,产能过剩的需求才能受到控制,房地产投资过度的压力才能逐步释放.地方政府借钱搞开发的冲动才能被抑制。对股市而言,如果3大顽疾不能得到有效控制,牛市就很难到来.如果以上陈述为真,以下哪一项陈述必然为真?

A.如果中国股市还没有迎来牛市,那一定是3大顽疾还没有得到有效控制。

B.如果地方政府借钱搞开发的冲动没有被抑制,则国内资金成本还没有回归到合理的位置.C.如果中国股市迎来了牛市,那一定是国内资金成本回归到了合理的位置.D.只要国内资金成本回归到合理的位置,中国经济就不会硬着陆.13、唐三藏一行西天取经,遇到火焰山。八戒说,只捡无火处走便罢。唐三藏道,我只欲往有经处去哩。沙僧道,有经处有火。沙僧的意思是,凡有经处皆有火。

如果沙僧的话为真,则以下哪一项陈述必然为真?

A.有些无火处有经。

B.有些有经处无火。

新东方在线在职硕士频道(http://shuoshi.koolearn.com/zzlk/)分享

C.凡有火处皆有经。

D.凡无火处皆无经。

14、世界上第一辆自行车是在1817年发明的。自行车出现后只流行了很短一段时间就销声匿迹了,直到1860年才重新出现,为什么会出现这种情况?只有当一项新技术与社会的价值观念相一致时该技术才会被接受.所以,1817年到1860年期间社会价值观一定发生了某种变化。

以上论证中的推理是有缺陷的,因为该论证

A.忽视了对自行车重新被接受的其他可能的解释。

B.提出了一个与该论证的结论关系不大的问题。

C.错误地认为自行车在1860年重新出现表明它被真正的接受了.D.没有给出合理的说明就认定流行并不意味着真正的接受.15、脐带血指胎儿娩出,脐带结扎并离断后殁留在胎盘和脐带中的血液,其中含有的造血干细胞对白血病、重症再生障碍性贫血、部分恶性肿瘤等疾病有显著疗效,是人生中错过就不再有的宝贵的自救资源。父母为新生儿保存脐带血,可以为孩子一生的健康提供保障。

如果以下陈述为真,除哪一项外,都能削弱上面论述的结论?

A.目前中国因患血液病需要做干细胞移植的概率极小,而保存脐带血的费用昂贵。

B.现在脐带血与外周血、骨髓一起成为造血干细胞的三大来源。

新东方在线在职硕士频道(http://shuoshi.koolearn.com/zzlk/)分享

C.目前在临床上脐带血并不是治疗许多恶性疾病的最有效手段,而是辅助治疗手段。

D.脐带血的保存量通常为50毫升,这样少的数量对大多数成年人的治疗几乎没有效果。

16、诡辩者:因为6大于4,并且6小于8,所以6既是大的又是小的。

以下哪一项中的推理方式与上述诡辩者的推理最相似?

A.因为老子比孟子更有智慧,所以老子对善的看法比孟子对善的看法更好。

B.因为张青在健康时喝通化葡萄酒是甜的,而在生病时喝通化葡萄酒是酸的,所以通化葡萄酒既是甜的又是酸的。

C.因为赵丰比李同高,并且赵丰比王磊矮,所以赵丰既是高的又是矮的。

D.因为一根木棍在通常情况下看是直的,而在水中看是弯的,所以这根木棍既是直的又是弯的。17、2014年8月俄罗斯200多辆运送救援物资的卡车进入乌克兰东部地区。如果乌克兰政府军在东部的军事行动直接或间接袭击了俄罗斯车队,则可能引发俄方采取强硬措施;如果乌克兰政府军暂时停止在东部的军事行动以保证俄罗斯车队的安全,则会给处于下风的民间武装以喘息的机会。

如果以上陈述为真,以下哪一项陈述必然为真?

A.如果乌克兰政府军袭击俄罗斯车队,则处于下风的民间武装就没有喘息的机会。

B.如果乌克兰政府军不给民间武装以喘息的机会,则有可能引发俄方采取强硬措施。

新东方在线在职硕士频道(http://shuoshi.koolearn.com/zzlk/)分享

C.如果乌克兰东部民间武装得到了喘息机会,俄罗斯就不会采取强硬措施。

D.俄罗斯车队进入乌克兰是为了帮助乌克兰东部的民间武装。

18、中国多所高校在多伦多、纽约、波士顿、旧金山召开了4场人才招聘会,针对出席招聘

会的中国留学生所做的问卷调查显示:67%的人希望回国工作,33%的人会认真考虑回国的选择。可见,在美国工作对中国留学生已失去吸引力,人心思归已蔚然成风。

以下哪一项陈述准确地指明了上述论证的缺陷?

A.参加问卷调查的中国留学生表达的未必是他们最好的愿望。

B.如果北美的中国留学生回国找不到工作,那会让他们大失所望。

C.67%和33%加起来是100%,这意味着希望留在北美工作的人为零。

D.在北美的中国留学生中,那些不打算回国工作的人没有参加招聘会。

19、传统看法认为,《周易》八卦和六十四卦卦名的由来或是取象说,或是取义说,不存在其他的解释.取象说认为八卦以某种物象的名来命名,比如乾卦之象为天,乾即吉时的天字,故取名为乾;取义说认为卦象代表事物之理,取其义理作为一卦之名,比如坤卦之象纯阴,阴主柔顺,故此卦名为坤,坤即柔顺之义。

以下哪一项陈述为真,严重地动摇了卦名由来的传统看法?

新东方在线在职硕士频道(http://shuoshi.koolearn.com/zzlk/)分享

A.乾坤两挂之所以居六十四卦之首,这是因为乾卦代表天,地卦代表地,天地相交,万物才得以生。

B.卦名不能单靠取象说来解释,也不能单靠取义说来解释,只有将二者结合起来,才能给出所有卦名的解释。

C.卦名的由来虽然有诸多不同的解释,但万变不离其宗,或者归属于取象说,或者归属于取义说。

D.卦名出自卦辞记述的所占之事,坤卦占问的是失马之事,当初筮得≡≡象,认为牝马驯良可以找到,遍取名为坤。

20、在球类比赛中,利用回放决定判罚是错误的。因为无论有多少台摄像机跟踪拍摄场上比赛,都难免会漏掉一些犯规动作。要对所发生的一切明察秋毫是不可能的。

以下哪一项论证的缺陷与上述论证的最相似?

A.知识就是美德,因为没人故意作恶。

B.我们不该要警察,因为他们不能阻止一切犯罪活动。

C.试婚不是不道德的,因为任何买衣服的人都可以试穿。

D.信念不能创造实在,因为把某事当成真的并不能使之成为真的。

21、科学家假设,一种特殊的脂肪,即“P-脂肪”,是视力发育形成过程中所必需的。科学家观察到,用含P-脂肪低的配方奶喂养的婴儿比母乳喂养的婴儿视力要差,而母乳中

新东方在线在职硕士频道(http://shuoshi.koolearn.com/zzlk/)分享

P-脂肪的含量高,于是他们提出了上述假说。此外还发现,早产5-6周的婴儿比足月出生的婴儿视力要差。

如果以下哪一项陈述为真,最能支持上述科学家的假设?

A.胎儿只是在妊娠期的最后4周里加大了从母体中获取的P-脂肪的量。

B.日常饮食中缺乏P-脂肪的成年人比日常饮食中P-脂肪含量高的成年人视力要差。

C.胎儿的视力是在妊娠期的最后3个月中发育形成的

D.母亲的视力差并不会导致婴儿的视力差。

22、文明人与野蛮人或其他动物的重要区别在于通过深谋远虑来抑制本能的冲动。唯有当一个人去做某一件事并不是受本能冲动的驱使,而是因为他的理智告诉他,到了未来某个

时期他会因此而受益,这时才出现了真正的深谋远虑,耕种土地就是一种深谋远虑的行动,人们为了冬天吃粮食而在春天工作。

以下哪一项陈述是上面论证所依赖的假设?

A.能否通过深谋远虑来抑制本能的冲动,这是文明人与野蛮人或其他动物的唯一区别。

B.松鼠埋栗子、北极狐埋鸟蛋等行动纯属受本能驱使的行动。

C.人对自己本能冲动的抑制力越强,对目前痛苦的忍受力就越大,因而其文明程度就越高。

新东方在线在职硕士频道(http://shuoshi.koolearn.com/zzlk/)分享

D.人不仅通过自己的深谋远虑来抑制本能的冲动,还通过外在的法律、习惯于宗教等抑制本能的冲动。

23、营养学家:迄今为止的所有医学研究都表明,每天饮用3杯或更少的咖啡不会对心脏造成伤害。因此,如果你是一个节制的咖啡饮用者,那么,你完全可以放心地享用,不必担心咖啡损害你的健康。

以下哪一项陈述最准确地指出了以上论证的缺陷?

A.咖啡饮用者在饮用咖啡的同时可能食用其他对心脏有害的食物。

B.该营养学家的结论只依据了相关的研究成果,缺乏临床数据的支持。

C.喝咖啡对心脏无害不意味着对身体无害。

D.常喝咖啡的人往往有较大的心理压力,而较大的心理压力本身就对心脏有害。

24.托马斯·潘恩在《常识》一书中讨论了君主制和世袭制是否合理的问题。对于那些相信

世袭制度合理的人,潘恩迫问并回答道,最初的国王是如何产生的呢?只有3种可能——

或者通过抽签,或者通过选举,或者通过篡权。如果第一位国王是通过抽签或选举产生的,这就为以后的国王们奠定了先例,从而否定了世袭的做法。如果第一位国王的王位是篡权得到的,那谁也不会如此大胆,竟敢为王位的世袭加以辩护。

以下哪一项最好地描述了潘恩的论证所使用的技巧?

新东方在线在职硕士频道(http://shuoshi.koolearn.com/zzlk/)分享

A.通过表明一个命题与某个已确立为真的命题矛盾,来论证前者不成立。

B.通过一个命题能推出假的结论,来论证这个命题不成立。

C.通过表明所有可能的解释都推出同一个命题,来论证这个命题成立。

D.通过排除所有其他可能的解释,来论证剩余的那个解释是成立的。

25、有时候,一个人不能精确地解释一个抽象语词的含义,却能十分恰当地使用这个语词进

行语言表达。可见,理解一个语词并非一定依赖于对这个语词的含义做出相确的解释。

以下哪一项陈述能为上面的结论提供最好的支持?

A.抽象语词的含义是不容易得到精确解释的。

B.如果一个人能精确地解释一个语词的含义,那他就理解这个语词。

C.一个人不能精确地解释一个语词的含义,不意味着其他人也不能精确地解释这个语词的含义。

D.如果一个人能十分恰当地使用一个语词进行语言表达,那他就理解这个语词。

26、有网络媒体报道称,让水稻听感恩歌大悲咒能增产l5%.福建省良山村连续3季的水稻种植结果证实,听大悲咒不仅增产了15%,水稻颗粒也更加饱满。有农业专家表示,音乐不仅有助于植物对营养物质的吸收、传输和转化,还能达到驱虫的效果。

新东方在线在职硕士频道(http://shuoshi.koolearn.com/zzlk/)分享

以下哪一个问题的回答对评估上述报道的真实性最不相关?

A.听大悲咒的水稻与不听大悲咒的水稻的其他生长条件是否完全相同?

B.该方法是否具有大面积推广的可行性?

C.专家能否解释为什么大悲咒对水稻的生长有益而对害虫的生长无益?

D.专家的解释是否具有可靠的理论支持?

27、关于如何界定“裸官”,2010年发布的相关《暂行规定》明确了以下3类国家工作人员为“裸官”:配偶、子女均已移居国(境)外的;没有子女,配偶已移居国(境)外的;没有配偶,子女均已移居国(境)外的。2014年中组部下发的相关《管理办法》规定:配偶已移居国(境)外的,或者没有配偶,子女均已移居国(境)外的国家工作人员均为“裸官”。

以下哪一项陈述与上述两个文件的规定是相符的?

A.根据《管理办法》,只有子女均已移居国(境)外的国家工作人员才是“裸官”。

B.对于既有配偶也有子女的国家工作人员来说,两个文件的规定是相同的。

C.根据《暂行规定》,只要某国家工作人员的配偶已移居国(境)外.他(她)就是“裸官”。

D.对于只有配偶没有子女的国家工作人员来说,两个文件的规定是相同的。

28、政府与民营企业合作完成某个项目的模式(简称PPP)能够使政府获得资佥,也可以让社

新东方在线在职硕士频道(http://shuoshi.koolearn.com/zzlk/)分享

会资本进入电力、铁路等公用事业领域。这种模式中存在的问题是政府违约或投资人违约而给对方造成经济损失。在以往的PPP项目中,政府违约不是小概率事件。尽管地方政府违约的现象屡见不鲜,但投资人还是一如既往积极地投资于PPP项目。

如果以下哪一项陈述为真,能够最好地解释上述看似矛盾的现象?

A.随着经济体制的改革和新城镇化建设的推力,PPP模式被社会各界寄予厚望。

B.PPP模式比较复杂,地方政府的谈判能力和PPP专业能力都不如投资人。

C.今年国家发改委发布了80个重要的项目,鼓励社会资本以PPP等方式参与建设和运营。

D.为防止地方政府违约带来的经济损失,投资人设法将其他方面的利益转移到自己的企业。

29、李明极有可能是一位资深的逻辑学教师。李明像绝大多数资深的逻辑学教师一样,熟悉哥德尔的完全性定理和不完全性定理,而绝大多数不是资深的逻辑学教师的人并不熟悉这些定理。实际上,许多不是资深的逻辑学教师的人甚至没有听说过哥德尔。

以下哪一项陈述准确地指出了上述推理的缺陷?

A.忽视了这种可能性:大多数熟悉哥德尔这些定理的人不是资深的逻辑学教师。

B.忽视了这种可能性;有些资深的逻辑学教师不熟悉哥德尔的这些定理。

C.推理中“资深的”这一概念是模糊的概念。

新东方在线在职硕士频道(http://shuoshi.koolearn.com/zzlk/)分享

D.不加证明就断定不熟悉哥德尔完全性定理和不完全性定理的人也没有听说过哥德尔。

30.由于人口老化,德国政府面临困境:如果不改革养老体系,将出现养老鱼不可持续的现

象.解决这一难题的政策包括提高养老金缴费比例、降低养老金支付水平,提高退休年

龄。其中提高退休年龄所受阻力最大,实行这一政策的政府可能会在下次选举时丢失大量选票。但德国政府于2007年完成法定程序,讲退休年龄从65岁提高到67岁。

如果以下哪一项陈述为真,能够最好地解释德国政府为什么冒险采用了这一政策?

A.2001年德国以法律形式确定了养老金缴费上限,2004年确定了养老金支付下限,两项政策已经用到极致。

B.为减轻压力,德国政府规定从2012年起用20年的过渡期来实现退休年龄从65岁提

高到67岁。

C.延迟一年退休,所削减的养老金可达GDP的近l%。

D.现在德国人的平均寿命大大提高,退休者领取养老金的年限越来越长。

31.为什么古希腊会产生城邦制,东方国家却长期存在君主专制?亚里士多德认为,君主专制在野蛮人中间常常可以见到,同僭主或暴君制很接近。因为野蛮民族的性情天生就比希

新东方在线在职硕士频道(http://shuoshi.koolearn.com/zzlk/)分享

腊各民族更具有奴性,其中亚细亚蛮族的奴性更甚于欧罗巴蛮族,所以他们甘受独裁统治而不起来叛乱。

如果以下各项陈述为真,除哪一项外,都能削弱亚里士多德的解释?

A.城邦制造就了公民的自主性,君主专制造就了顺民的奴性。

B.地理环境的差别造就了城邦制和君主专制的区别。

C.亚里士多德的解释在感情上令绝大多数东方人难以接受。

D.文明人与野蛮人的区别是文化和社会组织不同造成的。

32、美国电动汽车Tesla使用的电池是由近7000块松下18650型电池通过串联、并联结合在一起的大电池包。Tesla电池动力系统的安全性一直受到汽车界的质疑。一位电池专家说,18650型电池在美国的起火概率是百万分之0.2,那么,70000块小电池组成的电池包的起火概率就是百分之0.14,以Tesla目前的销量看,这将导致它几乎每个月发生一次电池起火事故。

如果以下陈述为真,哪一项最有力地削弱了专家的判断?

A.18650型电池具有能量密度大、稳定、一致性好的特点。

B.全球每年生产数十亿块18650型电池,其安全级别不断提高。

C.Tesla有非常先进的电池管理系统,会自动断开工作异常的电池单元的输出。

D.18650型电池可循环充电次数多,因此大大延长了电池的使用寿命。

新东方在线在职硕士频道(http://shuoshi.koolearn.com/zzlk/)分享

33、北极地区蕴藏着丰富的石油、天然气、矿物和渔业资源,其油气储量占世界未开发油气资源的1/4。全球受暖使北极地区冰面以每10年9%的速度融化,穿过北冰洋沿俄罗斯北部海岸线连通大西洋和太平洋的航线可以使从亚洲到欧洲比走巴拿马运河近上万公里。

因此,北极的开发和利用将为人类带来巨大的好处。

如果以下陈述为真,除哪一项外,都能削弱上述论证?

A.穿越北极的航船会带来入侵生物,破坏北极的生态系统。

B.国际社会因北极开发问题发生过许多严重冲突,但当事国做了冷静搁置或低调处理。

C.开发北极会使永久冻土融化,释放温室气体甲烷,导致极端天气增多。

D.开发北极会加速冰雪融化,使海平面上升,淹没沿海低地。

34.“倾销”被定义为以低于商品生产成本的价格在另一国销售这种商品的行为。H国的河虾生产者正在以低于M国河虾生产成本的价格,在M国销售河虾。因此H国的河虾生产者正在M国倾销河虾。

以下哪一项对评估上文提到的倾销行为是必要的?

A.倾销定义中的“生产成本”指的是商品原产地的生产成本,还是销售地同类商品的生产成本。

新东方在线在职硕士频道(http://shuoshi.koolearn.com/zzlk/)分享

B.如果H国一直以低于M国的河虾生产成本的价格在M国销售河虾,M国的河虾业就会破产。

C.专家们在倾销行为对两国的经济都有害或都有利,还是只对其中的一方有害或有利的问题上达成了共识。

D.由于计算商品生产成本的方法不同,很难得出同一种两品在不同国家的生产成本的精

确比较数值。

35、由于量子理论的结论违反直观,有些科学家对这一理论持不同看法,尽管他们试图严格地表明量子理论的断言是不精确的(即试图严格地证伪它),但是发现,其误差在通常可接受的统计范围之内。量子理论的这些结果不同于与它相竞争的理论的结果,这表明接受量子理论是合理的。

以下哪一项原则最有助于表明上述推理的合理性?

A.一个理论在被试图严格地证伪之前不应当被认为是合理的。

B.只有一个理论的断言没有被实验所证伪,才可以接受这个理论。

C.如果一个科学理论中违反直观的结论比与它相竞争的理论少,那么应该接受这个理论。

D.如果试图严格地证伪一个理论,但该理论经受住了所有的考验,那么应该接受它。

新东方在线在职硕士频道(http://shuoshi.koolearn.com/zzlk/)分享

36、直立人大约于200万年前起源于非洲,并且扩散到了欧亚大陆:现代人约在20万年前出

现。这两种人类的化石在中国均有分布。比如北京周口店古老地层出土的“北京人”属于直立人;年轻地层中的“山顶洞人”属于现代人。对中国当代人群的研究发现,父系遗传的Y染色体均源自非洲,起源时间在8.9万年至3.5万年前;母系遗传的线粒体DNA均源自非洲,起源时间在10万年以内;没有检测到直立人的遗传组分。

如果以上陈述为真,最能支持以下哪个假说?

A.“北京人”的后代可能灭绝了,中国当代人的祖先是大约10万年前从非洲来到亚洲的。

B.中国的直立人和现代人分别来自非洲大陆,他们杂交的后代是中国当代人的祖先。

C.北京周口店的“山顶洞人”是从“北京人”进化而来的.D.中国当代人是200万年前从非洲扩散到欧亚大陆的直立人的后代。

37、桓公:“为何说寡人读的是古人的糟粕?”轮扁:“依我的经验看,斫车轮,轮孔做得稍大就松滑而不坚固,做得稍小就滞涩难入。要想做得不大不小,不松不紧,必须得之于心而应之于手,有高超的技术存在其中,却无法用语言表达,我无法教给我儿子,所以,我都七十岁了还得斫轮。古人已经死了,他们所不能的精华也跟着消失了,那么您所读的就是古人的糟粕了。”

以下哪一项陈述是轮扁的议论所依赖的假设?

新东方在线在职硕士频道(http://shuoshi.koolearn.com/zzlk/)分享

A.除了精华和糟粕外,还有其他值得阅读的内容。

B.如果精华不能言传,读书不但无用反而会有害。

C.高超的技术是无祛通过语言传授给别人的。

D.除了高超的技术外,其他精华也是不能言传的。

38、某厨艺大赛,要求厨师制作热菜、凉菜各一项,结果评定为“上品”、“中品”、“下品”3种。如果甲厨师每项结果不低于乙厨师,且至少有一项比乙厨师高,则称“甲厨师比乙厨师技艺高”。现有厨师若干,他们之中没有一个比另一个技艺高,并且没有任意两人热菜结果一样,凉菜结果也是一样的。

满足上述条件的厨师最多能有几人?

A.无法确定

B.9人

C.6人

D.3人

39.某品牌的一款节油型车售价27万元,而另一款普通车售价17万元。根据目前的汽油价格以及这两款车百公里油耗的测试数据,购买这款节油型车的人需开满30万公里才能补足比购买普通车高出的价差。如果将来油价上涨,那么,为补足购车价差所需的里程数还要相应增加。以下哪一项陈述最准确地指出了以上论证的缺陷?

新东方在线在职硕士频道(http://shuoshi.koolearn.com/zzlk/)分享

A.论据与结论是互相矛盾的。

B.论据不能充分地支持其结论。

C.论证没有考虑将来油价下调这种可能性。

D.论证使用了未经证实的假设作论据。

40.在信息纷繁复杂的互联网时代,每个人都时刻面临着被别人的观点欺骗、裹挟、操纵的风险。如果你不想总是受他人摆布,如果你不想混混沌沌地度过一生,如果你想学会独立思考、理性决策,那你就必须用批判性思维来武装你的头脑。

如果以上陈述为真,以下哪一项陈述不必然为真?

A.不能用批判性思维武装头脑的人,就不可能学会独立思考、理性决策。

B.你或者选择用批判性思维来武装你的头脑,或者选择混混沌沌地度过一生。

C.不想学会独立思考、理性决策的人,就不必用批判性思维来武装头脑。

D.只有用批判性思维武装头脑的人,才能摆脱被他人摆布的命运。

41-45基于以下共同题干:

某单位在大年初

一、初二初三安排6个人值班,他们是GHKLPS,每天需要2人值班。人员安排要满足以下条件:

(1)L与P必须在同一天值班。

新东方在线在职硕士频道(http://shuoshi.koolearn.com/zzlk/)分享

(2)G与H不能在同一天值班。

(3)如果K在初一值班,那么G在初二值班。

(4)如果S在初三值班,那么H在初二值班。

41.以下哪一项可以是 这些人值班日期的一个完整且准确的安排?

A、初一:L和P;初二:G和K;初三: H和S

B、初一:L和P;初二:H和K;初三: G和S

C、初一:G和K;初二:L和P;初三: H和S

D、初一:K和S;初二:G和H;初三: L和P

42.以下哪一项必然为真?

A、G与S在同一天值班

B、S与H不在同一天值班

C、K与S不在同一天值班

D、K与G不在同一天值班

43.如果P在初二值班,以下哪一项可以为真?

A.S在初三值班 B.H在初二值班

新东方在线在职硕士频道(http://shuoshi.koolearn.com/zzlk/)分享

C.K在初一值班 D.G在初一值班

44.如果G和K在同一天值班,以下哪一项必然为真?

A.S不在初三值班 B.K在初二值班

C.L在初一值班 D.H在初一值班

45.如果H在S的前一天值班,则以下哪一项不能为真?

A.G在初二值班 B.P在初二值班

C.K在初一值班 D.H在初一值班

46-50题基于以下共同题干:

一桌宴席的所有凉菜上齐后,热菜共有7个,其中,3个川菜:K、L、M;3个粤菜:Q、N、P;一个鲁菜:X。每次只上一个热菜,上菜的顺序必须符合下列条件:

(1)不能连续上川菜,也不能连续上粤菜;

(2)除非第三个上Q,否则P不能在Q之前上。

(3)P必须在X之前上。

(4)M必须在K之前上,K必须在N之前上。

46.以下列出的是从第一到第七的上菜顺序,哪一项符合条件?

新东方在线在职硕士频道(http://shuoshi.koolearn.com/zzlk/)分享

A、Q、M、P、L、X、K、N

B、Q、M、X、P、K、N、L

C、L、P、M、Q、X、K、N

D、M、Q、K、L、P、X、N

47.如果第四个上X,以下哪一项陈述必然真?

A、第一个上Q

B、第三个上Q

C、第二个上M

D、第三个上M

48.如果第四个上K,以下哪一项陈述可能真?

A、第五个上P

B、第六个上X

C、第五个上L

D、第一个上M

49.以下哪一项陈述可能真?

新东方在线在职硕士频道(http://shuoshi.koolearn.com/zzlk/)分享

A、第二个上P

B、第三个上N

C、第二个上K

D、第二个上L

50.如果第三个上M,以下哪一项陈述可能真?

A、第五个上X

B、第一个上Q

C、第六个上L

D、第四个上K 更多在职硕士考试免费资料请访问“新东方在线在职硕士频道”

第五篇:2011年GCT逻辑真题及答案

2011年GCT逻辑真题

1、有人说道:读万卷书不如行万里路,行万里路不如阅人无数,阅人无数不如名师指路,名师指路不如自己领悟。

以下各项都构成对上述观点的质疑,除了: A、阅人无数自会遇到名师指路。

B、书中自有乾坤,读万卷书如同行万里路。

C、若无名师指路,仅凭万卷书和行万里路,自己仍无法领悟。D、行万里路,游历大千世界,必定阅人无数。

2、爱因斯坦发表狭义相对论时,有人问他:预计公众会有什么反应?他答道:很简单,如果我的理论是正确的,那么,德国人会说我是德国人,法国人会说我是欧洲人,美国人会说我是世界公民;如果我的理论不正确,那么,美国人会说我是欧洲人,法国人会说我是德国人,德国人会说我是犹太人。

如果爱因斯坦的话是真的,以下哪项陈述一定为真? A、有人会说爱因斯坦是德国人。B、有人会说爱因斯坦是世界公民。C、有人会说爱因斯坦是犹太人。D、德国人会说爱因斯坦是欧洲人。

3、世界卫生组织报告说,全球每年有数百万人死于各种医疗事故。在任何一个国家的医院,医疗事故致死的概率不低于0.3%。因此,即使是癌症患者也不应当去医院治疗,因为去医院治疗会增加死亡的风险。为了评估上述论证,对以下哪个问题的回答最为重要?

A、在因医疗事故死亡的癌症患者中,即使不遭遇医疗事故最终也会死于癌症的人占多大比例?

B、去医院治疗的癌症患者和不去医院治疗的癌症患者的死亡率分别是对少? C、医疗事故致死的概率是否因医院管理水平的提高而正子啊下降? D、患者能否通过自身的努力来减少医疗事故的发生?

4、因对微博的无知,某局长和某主任在微博上泄露个人隐私,暴露其不道德行为,受到有关部门的查处。有网友对他们的行为冷嘲热讽,感慨到:知识改变命运,没有知识也改变命运。以下哪项陈述最接近该网友所表达的意思? A、无论是否有知识,都会改变命运。

B、“知识就是力量”这一说法过于夸张,实际上,权利和金钱才是力量。

C、有知识导致命运由不好向好的方面改变,没有知识导致命运由好向坏的方向改变。D、“命运”的本义就是先天注定,它不会因有无知识而改变。

5、在大学里,许多温和宽厚的教师是好教师,但有些严肃且不讲情面的教师也是好教师,而所有好教师都有一个共同点:他们都是学识渊博的人。如果以上陈述为真,以下哪项陈述一定为真? A、许多学识渊博的教师是温和宽厚的。

B、有些学识渊博的教师是严肃且不讲情面的。C、所有学识渊博的教师都是好教师。D、有些学识渊博的教师不是好教师。

6、《与贸易有关的知识产权协定》规定,不得仅仅因为成员国本国法律禁止某些发明的商业性实施就不授予那些发明专利权。已知A国是《与贸易有关的知识产权协定》的成员国,以下哪一项陈述与上述规定不一致?

A、从A国法律禁止一项发明的商业性实施推不出不能授予该项发明专利权。B、从A国法律允许授予一项发明专利权推不出允许该项发明的商业性实施。C、在A国,一种窃听装置的商业性实施是被法律禁止的,因此不允许授予其专利权。D、在A国,一种改进枪支瞄准的发明被授予了专利权,但该发明的商业性实施被禁止。

7、每一个政客都不得不取悦他的选民。尽管马英九是一位诚实的人,但他也是一位政客。如果不偶尔说出一些含糊其辞的话,任何人都不能取悦他的选民。如果以上陈述为真,以下哪项陈述一定为真?

A、马英九不会说出含糊其辞的话。B、马英九会说出一些含糊其辞的话。

C、说含糊其辞话的政客不是诚实的人。

D、有的诚实的人不是政客。

8、具有能够让一个乐队特别是一流乐队反复进行排练的权威,这是一个优秀指挥家的标志。

这种威望不是轻而易得的,一个指挥家必须通过赢得乐队对他所追求的艺术见解的尊重才能获得这种威望。在上文的论述过程中,作者预先假设了以下哪项陈述?

A、优秀的指挥家在与不同的乐队合作时,对同一首作品会有不同的艺术见解。B、优秀的指挥家都是完美主义者,即使对一流乐队的表演,他们也从不满意。C、如果优秀的指挥家认为附加的排练是必须的,一流乐队总是时刻准备加班排练。

D、即使一种艺术见解还没有被充分表现出来,一流乐队也能够领悟这种艺术见解的优点。9、2010年,卫生部推出新的乳业国家标准,将原奶蛋白质含量由原来的2.95%降至2.8%,新标准不升反降,引发了一片质疑。某业内人士解释说,如果我们的牛奶检测标准把蛋白质含量定的太高,奶农为了达标就会往奶里添加提高蛋白质检测含量的东西,如三聚氰胺;以下哪项推理含有与该业内人士的推理相同的逻辑错误?

A、真正的强者都不惧怕困难或挑战,赵涛害怕困难或挑战,说明赵涛不是真正的强者。B、如果台风在海口登陆,飞往海口的航班就会被取消。现在飞海口的航班没有被取消,说明台风没有在海口登陆。

C、要是铁路部门的管理存在漏洞,铁路运输就会出事故。7.23温州动车事故说明铁路部门的管理存在漏洞。

D、仅当人们信任一个慈善机构时才会向该机构捐款,所以,得不到捐款的慈善机构一定是丧失了公众的信任。

10、现代社会中有很多人发胖,长有啤酒肚,体重严重超标,因为他们常常喝啤酒。对以下各项问题的回答都可能质疑上述论证,除了:

A、如果人们每天只喝啤酒,吃很少的其他食物,特别是肉食品,他们还会发胖吗? B、为什么美国有很多女人和孩子常喝可乐、吃炸鸡和披萨饼,其体重也严重超标? C、发胖的人除常喝啤酒外,是否经常进行体育锻炼?

D、很多发胖的人也同时抽烟,能够说“抽烟导致发胖”吗?

11、经济学家:中国外汇储备在过去10年的快速增长是中国经济成功的标志之一。没有外汇储备的增长,就没有中国目前的国际影响力。但是,不进行外汇储备投资,就不会有外汇储备的增长。外汇储备投资面临风险是正常的,只要投资寻求收益,就要承担风险。以下哪项陈述能从这位经济学家的论述中合逻辑地推出? A、如果能够承担风险,就会有外汇储备的增长。B、如果不进行外汇储备投资,就不用承担风险。

C、只要进行外汇储备投资,中国就能具有国际影响力。

D、中国具有目前的国际影响力,是因为中国承担了投资风险。

12、山西醋产业协会某前副会长称,在市面上销售的山西老陈醋中,只有5%是不加添加剂的真正意义上的山西老陈醋,中国调味品协会某副会长就此事件接受记者采访时说:“只要是按国家标准添加添加剂,都没有安全问题。有些企业强调自己未加添加剂,这对按正常标准加添加剂的企业来说是不公平的。”以下哪项陈述能够从该调味品协会副会长的话中逻辑地推出? A、为了保证公平性,企业或者不应该生产高于国家标准的产品,或者要对产品质量高于国家标准的事实秘而不宣。

B、要想促进行业的技术创新,就应当提高行业的国家标准。C、某个行业的国家标准定的太高,不利于该行业的良性发展。D、如果不按国家标准加添加剂,就会有安全问题。

13、午夜时分,小约翰安静地坐着。他非常希望此时是早晨,这样他就可以出去踢足球了。他平心静气,祈祷太阳早点升起来。在他祈祷的时候,天慢慢变亮了。他继续祈祷。太阳逐渐冒出地平线,升上天空。小约翰想了想所发生的事情,得出这样的结论:如果他祈祷的话,他就能够把寒冷而孤寂的夜晚变成温暖而明朗的白天。他为自己感到自豪。下面哪项陈述最恰当的指明了小约翰推理中的缺陷?

A、小约翰只是个孩子,他懂的很少很少。

B、太阳环绕地球运转,不管他祈祷还是不祈祷。

C、一件事情在他祈祷之后发生,并不意味着因为他祈祷而发生。D、他有什么证据表明:如果他不祈祷,该事情就不会发生?

14、借债者的信用等级越低,其借债的成本越高,债主的收益率也越高。2011年8月5日,标普将美国主权信用评级从AAA降为AA+。按照常理,投资者出于避险需求应当减持美国国债,从而推动其价格走低,收益率上升。然而,8月8日美国国债价格上涨,收益率降低,最新10年期美国国债收益率降至2.34%,创今年以来新低。

如果以下哪项陈述为真,能够最好地解释上述违反常理的现象?

A、8月5日晚间,美国财政部回应称,标普进行评级计算时有2万亿美元的误差。B、与欧盟、日本以及其他国家的债权相比,美国国债依然是相对安全和流动性最高的投资品。C、美国主权信用评级下调后,俄罗斯开始减持美国国债,而中国仍增持美国国债。

D、欧盟为打破标普等三家美国评级机构的垄断地位,计划建立一家欧洲的信用评级机构。

15、在《反省的生命》艺术中,诺齐克写到:“我不会像苏格拉底一样,说未经反省的生命是不值得的——那是过分严苛了。但是,如果我们的人生是由深思熟虑的反省所引导,那么,它就是我们为自己活的生命,而不是别人的。藏歌意义上说,未经反省的生命是不完整的生命。” 以下各项都能从诺齐克的陈述中推出,除了:

A、诺齐克认为,值得过的生命都是经过反省的生命。B、诺齐克认为,只有为自己活的生命才是完整的生命。C、诺齐克认为,完整的生命都是经过反省的生命。D、诺齐克认为,未经反省的生命不是完整的生命。

16、中国人仇富,居然有那么多人为骗子说话,只因为他们骗的是富人,我敢断定,那些骂富人的人,每天都在梦想成为富人。如果他们有机会成为富人,未必就比他们所骂的人干净。况且,并非所有的富人都为富不仁,至少我周围有的富人不是,我看到他们辛勤工作且有慈悲心怀。——有网友对达芬奇家具造假事件的网上评论如是说 根据该网友的说法,不能合逻辑地确定以下哪项申述的真假? A、有的仇富者是中国人。B、有的富人并非为富不仁。

C、那些每天都在梦想成为富人的人却在骂富人。D、有的辛勤工作且有慈悲心怀的人是富人。

17、目前我国有3种转基因水稻正等待商业化种植审批,每种至少涉及5~12项国外专利;有5种转基因水稻正处于研发过程中,每种至少涉及10项国外专利。有专家认为,水稻是我国的主要粮食作物,如果我国允许转基因水稻商业化种植,国家对主要粮食作物的控制权就可能受到威胁。

如果以下哪项陈述为真,将最有力地支持该专家的观点?

A、转基因水稻的优势在于抵抗特定的害虫,但我国水稻很少有这些害虫。

B、目前还没有任何一种转基因水稻能超出我国超级稻、杂交稻等品种的产量和品质。C、美国引入转基因种子后,玉米、棉花、大豆等种子的价格大幅上涨。

D、如果我国商业化种植转基因水稻,国外专利持有人就会禁止我国农民保留种子,迫使他们每个播种季都高价购买种子。

18、孔子非常懂得饮食和养生的道理,《论语·乡党》就列出了很多“食”和“不食”的主张,比如“不时不食”,意思是说不要吃反季节蔬菜。以下哪项陈述是上述解释所必须依赖的假设? A、孔子在饮食方面的要求很高。

B、孔子生活的时代既有当季蔬菜,也有反季节蔬菜。C、我们可以选择吃当季蔬菜,还是吃反季节蔬菜。D、饮食不仅滋养人的身体,还塑造人的心灵。

19、在众声喧哗中,尽可能打捞那些沉没的声音,是社会管理者应尽之责。以政府之力,维护弱势人群的表达权,使他们的利益能够通过制度化规范化渠道正常表达,这是构建和谐社会的关键所在。只有这样,才能让“说话”、“发声”不仅是表达诉求的基本手段,更成为培育健康社会心态的重要环节,成为社会长治久安的坚实基础。如果以下哪项陈述为真,将最有力地支持上述论证? A、有些弱势者缺医少药,甚至得不到温饱 B、弱势者是社会中“沉默的大多数”,一旦真正发怒,其力量足以颠覆整个国家机器。C、有些弱势者的诉求长期得不到表达和满足,容易患各种心理疾病。D、甚至像美国这样的国家,也有很多食不果腹、居无定所的弱势者。

20、天津许云鹤一案:许云鹤在驾车行驶途中,遇王老太翻越马路中心护栏,后王老太倒地受伤,许云鹤把她扶起。王老太坚称是许将她撞伤。经司法鉴定,许的车没有碰到她。法院一审认为,许见到王老太时离她仅四、五米远,她摔倒定然是由于受到许的车惊吓,判决许承担40%的责任,赔偿王老太108606元。

以下哪项陈述最好地指出了该案一审中的纰漏之处?

A、法院推理悖于常理:王老太违章翻越马路护栏,应该预期机动车道上有车,而受惊吓通常是看到未预期的事物。

B、该判决违背传统道德:像南京彭宇案一样,会使大众不敢向受伤者伸出援助之手。C、该判决没有弄清事实,证据严重不足,有可能王老太先摔倒,许才开车过来。D、该判决颠倒了原因和结果。

21、英国医生约翰·斯诺的“污水理论”开启了流行病学研究的历史。1854年,伦敦爆发了大规模的霍乱,约翰·斯诺发现,大多数死亡病例都曾经引用同一个水泵汲取的水,而使用其他水泵或水井的人最初都没有感染霍乱。后经调查,下水道的废水污染了那个水泵,从而引发了霍乱。

以下哪一选项是约翰·斯诺的推理没有运用的方法或原则?

A、在被研究现象出现的各个场合都存在的因素很可能是该现象的原因。B、在被研究现象不出现的各个场合都不出现的因素很可能不是该现象的原因。C、当被研究现象变化时,同步发生量变的那个因素很可能是该现象的原因。

D、在被研究对象出现的场合与该现象不出现的场合之家的差异很可能是该现象的原因。

22、尼禄是公元一世纪的罗马皇帝,每一位罗马皇帝都喝葡萄酒,且只用锡壶和锡高脚酒杯喝酒。无论是谁,只要使用锡器皿去饮酒,哪怕只用过一次,也会导致中毒,而中毒总是导致精神错乱。

如果以上陈述都是真的,以下哪项陈述一定为真?

A、那些精神错乱的人至少用过一次锡器皿去饮葡萄酒。B、不管他别的方面怎么样,皇帝肯定是精神错乱的。C、使用锡器皿是罗马皇帝特权。

D、在罗马王朝的臣民中,中毒是一种常见现象。

23、巴勒斯坦准备在2011年9月申请加入联合国,已经争取到140个国家的支持。如果美国在安理会动用否决权,阻止巴勒斯坦进入联合国,会在整个阿拉伯世界引燃反美情绪;则如果美国不动用否决权,则会得罪以色列并使奥巴马失去一部分支持以色列的选民。如果以上陈述为真,以下哪项陈述一定为真?

A、美国在安理会动用否决权,阻止巴勒斯坦进入联合国。

B、美国不会得罪以色列,却会在整个阿拉伯世界引燃反美情绪。

C、美国会在阿拉伯世界引燃反美情绪,或者奥巴马失去一部分支持以色列的选民。

D、即使美国动用否决权,联合国大会仍打算投票表决,让巴勒斯坦成为具有国家地位的观察员。

24、通过风险投资方式融资创建的公司比通过其他渠道融资创建的公司失败率要低。可见,与企业家个人素质、公司战略规划或公司管理结构等因素对比,融资渠道对于一个新公司的成功发展是更为重要的因素。

如果以下哪项陈述为真,将最有力地削弱上述论证? A、有大约一半的新公司在创立后5年内就倒闭了。

B、初创公司的管理结构通常不如发展中公司的管理结构那样合理。

C、在与其他投资人相比,风险投资人对初创公司在财务需求方面的变化更为敏感。D、在决定是否为初创公司提供资金时,风险投资人必须考虑企业家的个人素质、公司的战略规划等因素。

25、未来的中国,将是一个更加开放包容、文明和谐的国家。一个国家、一个民族,只有开放包容,才能发展进步。唯有开放,先进和有用的东西才能进的来;唯有包容,吸收借鉴优秀文化,才能使自己充实和强大起来。

如果以上说法为真,以下哪项陈述一定为假?

A、一个国家或民族,即使不开放包容,也能发展进步。B、一个国家或民族,如果不开放包容,它就不能发展进步。C、一个国家或民族,如果要发展进步,它就必须开放包容。D、一个国家或民族,即使开放包容,也可能不会发展进步。

26、大约在12000年前,当气候变暖时,人类开始陆续来到北美洲各地。在同一时期,大型哺乳动物,如乳齿象、猛犸和剑齿虎等,却从它们曾经广泛分布的北美洲土地上灭绝了。所以,与人类曾经与自然界其他生物和平相处的神话相反,早在12000年前,人类的活动便导致了这些动物的灭绝。

以上论证最容易受到以下哪些项陈述的质疑? A.该论证未经反思地把人类排除在自然界之外。B.人类来到北美洲可能还会导致乳齿象、猛犸和剑齿虎之外的其它动物灭绝。

C.乳齿象、猛犸和剑齿虎等大型哺乳动物的灭绝,对于早期北美的原始人来说,具有非同寻常的意义。

D.所提出的证词同样适用于两钟可选择的假说:气候的变化导致大型哺乳动物灭绝,但同样的原因使得人类来到北美各地。

27、平均而言,今天受过教育的人的读书时间明显少于50年前受过教育的人的读书时间。但是,现在每年的销售书册数却比50年前增加了很多。以下各项陈述都有助于解释上述现象,除了

A.今天受过教育的人比50年前受过教育的人的数量大大增加。B.与现在相比,50年前的人们更喜欢从图书馆借阅图书。

C.与现在相比,50年前的人更喜欢通过大量藏书来显示其良好的教育和品位。D.现在的书往往比50年前的书更薄,也更容易读。

28、思想观念的价值,在竞争中才会彰显,在实践中才能检验。“我不同意你的看法,但我誓死捍卫你说话的权利”,这是一种胸怀,更是一种自信。那种扣帽子、抓辫子的辩论方式,“不同即敌对”的思维模式,本质上都是狭隘虚弱的表现,无助于和谐社会的构建。全社会都应该以包容的心态对待“异质思维”,尤其是执掌权柄者。以下各项都支持上述观点和论证,除了:

A、让人说话天不会塌下来,科学是在不断颠覆或改进“正统”观念中前进的。B、个别的异质思维者是偏执狂,其言行近乎疯子,理应受到严格管制。

C、每个人,甚至是当今的执掌权柄者,都有可能在某一天成为异质思维者,成为少数派。D、如果社会成员的思想和情绪得不到适当的表达和宣泄,容易导致各种极端行为。

29、一家商场按下述方式促销商品:一年中任何时候,或者有季节性促销,或者有节日促销,或两者兼而有之。每一种促销都会持续一个月。在任何一个月,如果商场想要把某一类商品清仓,就宣布季节性促销;如果某个月份有节日并且仓库中仍有剩余商品,就宣布节日促销。不过,11月没有节日而且这个月份仓库中也没有剩余商品。以下哪项陈述能从上文中合逻辑地推出?

A、如果某个月没有季节性促销,这个月一定有节日促销。B、如果节日促销没有进行,那一定是在底11月份。

C、如果季节性促销在某个月进行,这个月仓库中一定有剩余商品。D、如果在某个月中有节日,但仓库中没有剩余商品,则宣布节日促销。

30、在对6岁儿童所做的小学入学前综合能力检测中,全天上甲学前班达9个月的儿童平均得分58,只在上午上甲学前班达9个月的平均得分52,只在下午上甲学前班达9个月的平均得分51,全天上乙学前班达9 个月的平均得分54分,而那些来自低收入家庭且没有上过学前班的6岁儿童,在同样的小学入学前综合水平测试中平均得32分。在统计学上,32分与上述其他分数之间的差距有重要意义。

从上面给定的数据,可以最合理地得出下面哪个假设性结论? A、得50分以上的学生可以上小学

B、要做出一个合情理的假设,还需要做更多的测试。

C、是否上过学前班与小学入学前的综合能力之间有相关性。D、应该给6岁以下儿童上学前班提供更多的经费支持。

31、稀土是储量较少的一类金属的统称,广泛应用于尖端科技和军工领域。世界绝大部分稀土产自中国。1998年以来,中国开始减少稀土开采量,控制稀土出口配额,加强稀土行业的集中度。对此,一些国家职责中国垄断资源,对世界其他国家“有极大地破坏性”,要求中国放宽对稀土产业的控制。

如果以下哪项陈述为真,最适合用来反驳这些国家对中国的指责?

A、稀土是不可再生的重要战略资源

B、目前按人均计算,中国已经属于稀土资源相对稀缺的国家。C、从1980年代起,中国一些地方队稀土滥采滥挖,造成资源的严重浪费和对环境的极大破坏。

D、2009年,中国稀土储量占全球储量的36%,美国占13%,俄罗斯占19%,澳大利亚占5.4%;中国产量占世界产量的97%,其他国家均为零。

32、政治家:大约4年前,我们党一开始执政就致力于治理通货膨胀,并成功地将消费者物价指数(CPI)的涨幅从当时的4%降到现在的1.5%。反对党在前一个4年的执政期间,CPI涨幅都超过了4%。因此,在我们党的领导下,商品的价格越来越低。这个政治家的论证最容易受到以下哪项批评?

A、没有详细的说明反对党的主张,而是简单地将它忽略了。

B、用来支持这一结论的证据是事实上为否定这一结论提供了更多的支持。

C、没有提到反对党执政期间是国内经济过热的时期,而现在是欧美出现了经济危机。D、没有提到这样一种可能性:反对党执行了治理通货膨胀的政策,但该政策的效果要经过一段时间才能显现。

33、一般情况下,上市公司的业绩与其股票价格是正相关的:一个公司公布年报时,如果其利润高于市场预期,则该公司的股票价格会上涨。然而,2011年3月31日,我国从事核电等电力机组生产的东方电气公司公布年报,67%的净利润增速超过了市场预期,但收盘时其股票价格却下挫1.59%,跑输大盘0.73个百分点。

如果以下哪项陈述为真,能够最好地解释上述反常现象?

A、2011年3月11日,日本发生地震和海啸,福岛第一核电站发生严重核泄漏事故,我国有人抢购碘盐。

B、2011年3月16日,中国国务院决定对核设施进行全面检查并暂停审批核电项目。

C、2011年3月底至4月26日,世界各地举办纪念切尔诺贝利核电站爆炸25周年的活动。D、经过长期辩论,2011年5月德国执政联盟决定,将于2022年前关闭德国所有核电站。

34、在某公司内部,新近设立了许多非常专门化的部门。这表明,该公司对下述问题非常感兴趣:如何以更精确的方式抓住其消费群体? 上述推理缺少下面哪一个选项?

A、前提:这些新部门在以更精确地方式去抓住其消费群体。B、结论:管理部门要求采取新的措施去抓住其消费群体。

C、前提:在设立这些新部门之前,该公司未能抓住其消费群体。D、结论:该公司将很快做更多的努力去抓住其消费群体。

35、我国农村的宅基地属集体所有,农民只能使用,不能买卖、出租和继承。宅基地制度保证了农民的生存权益。农民在宅基地上建造的房屋时农民的资产。如果允许农民出卖自己的房屋,则实际上允许出让宅基地的使用权。如果宅基地的使用权被别人买走,则会损害农民的生存权益。但如果不允许农民出卖自己的房屋,则侵害了农民的资产权益。如果以上陈述为真,以下哪项陈述一定为真? A、农民在宅基地上建造的房屋没有产权证。B、如果农民工在城市里购买了住房,则不应当再在农村老家占有宅基地。C、如果不损害农民的生存权益,则会侵害农民的资产权益。

D、对绝大多数农民工而言,农村老家的宅基地和责任田是他们最后的生存保障。

36、自从1989年阿拉斯加埃克森油轮灾难和1991年中东战争以来,航空油料的价格已经巨幅上涨。在同一时期内,几种石油衍生品的价格也大幅上扬,这两个事实表明:航空油料是石油衍生品。以下哪项陈述最好地评价了上述论证? A、好的思维,因为航空油料是石油衍生品。B、坏的思维,没有精确地陈述所有的事实。

C、坏的思维,同一时期内食品价格也上涨了,但这不能证明航空油料是食品。D、坏的思维,给定关于石油衍生品的事实,不能得出关于航空油料的任何结论。37、2011年8月,中国第一艘航空母舰 “瓦良格”号出海试航。中国拥有航母会对美国海军在西太平洋的制海权构成潜在的挑战,美军对此感到担忧并持防范态度。然而,美国一位富有实战经验的海军专家却希望中国建造更多的航母。

如果以下哪项陈述为真,能够最好地解释这位专家看似矛盾的态度? A、2011年8月,越南与美国在南海进行了联合军事演习。

B、美国、日本、印度等国借中国航母试航之机,掀起新的“中国威胁论”的鼓噪。C、中国海军并未充分部署支持航母的运输船和加油船,护卫航母的驱逐舰和潜艇也十分脆弱。D、航母需要支持和护卫,建造航母越多,用来制造支持航母的其他舰艇和飞机的资源越少。

38、哲学家:在18世纪,某篇关于运动是绝对的论文断言,一个物体在一段时间内的位置变化可以不参照任何其他物体的位置而测得。不过,一位颇受尊敬的物理学家声称,这篇论文是不连贯的。既然一篇不连贯的论文不能认为是对实在的描述,故运动不可能是 绝对的。

这个哲学家的论证使用了下面哪一种论证方法或技巧? A、使用专业术语来说服别人。

B、依赖某个专家的权威来支持一个前提。

C、使用实验结果来说明所提到的位置变化是合理的。

D、观察到某物在实验条件下是某种情况,推出该物在任何情况下也是这种情况。

39、国内油价与国际油价相比一向是“涨快跌慢”,原因之一是我国成品油调价一句三地原油价格变动超过4%的边界条件。比如,从100美元一桶涨到104美元时,我们就跟着涨价;但是,从104美元回调4%再降价,则要等价格回落到99.84美元。两个条件实际上是不同的,降价要求的条件更高,更不容易达到。

以下哪项陈述最好地指出了上述论证的缺陷?

A、上述论证没能说明,国内油价不合理是由多种原因造成的。B、上述论证没能说明,国内油价不合理的根本原因在于行业垄断。

C、上述论证没能说明,以4%为边界条件,则油价越高,国内反应越迟钝。

D、上述论证没有从相同的价格基点出发进行比较,如果涨价的基点是100美元,对降

价的条件分析也应该是100美元为基点。

40、在某个班级中,L同学比X同学个子矮,Y同学比L同学个子矮,但M同学比Y同学个子矮,所以,Y同学比J同学个子矮。

必须增加以下哪一项陈述做前提,才能合逻辑地推出上述结论? A、J同学比L同学个子高 B、X同学比J同学个子高 C、L同学比J同学个子高 D、J同学比M同学个子高 41-45题基于以下共同的题干: 一家剧院计划在秋季的7周上演7个剧目,他们是F、G、J、K、O、R、S。每周上演一个剧目,每个剧目恰好演出一周,剧目的安排必须满足以下条件:(1)G必须在第三周上演(2)O和S不能连续演出

(3)K必须安排在J和S之前上演

(4)F和J必须安排在连续的两周中演出

41、如果把F安排在第五周上演,以下哪项准确地流出了所有可以安排在第七周上演的剧目? A、J、S

B、O、R C、O、R、S

D、K、O、R

42、对于任何一种可接受的安排,以下哪项一定为真? A、F被安排在K之后的某一周。

B、G恰好被安排在O之前的那一周。C、J被安排在第一周。

D、R被安排在第二周或第七周

43、如果把R安排在第五周,把O安排在第二周,则可供选择的安排方式有:

A、2种 B、3种 C、5种 D、6种

44、如果把S安排在第六周,那么必须把R安排在哪一周?

A、第二周 B、第四周 C、第五周 D、第七周

45、如果O恰好被安排在J之前的那一周,以下哪项一定为真?

A、把F安排在O之前。

B、K被安排在G之前的某一周。C、R被安排在第一周或第二周。D、S恰好安排在K之后的那一周。

46-50题基于以下共同的题干:

一个委员会工作两年,每年都由4人组成,其中2名成员来自下面4位法官:F、G、H、I,另外2名成员来自下面3位科学家:V、Y、Z。每一年,该委员会有1名成员做主席。在第一年做主席的成员在第二年必须退出该委员会。在第二年做主席的人在第一年必须是该委员会的成员。该委员会成员必须满足下面的条件:

G和V不能再同一年成为该委员会的成员。H和Y不能再同一年成为该委员会的成员。

每一年,I和V中有且只有一位做该委员会的成员。

46、下面哪项列出了能够在第一年成为该委员会成员的名单?

A、F,G,V,Z B、F,H,V,Z C、H,I,Y,Z D、G,H,I,Z

47、如果V在第一年做该委员会主席,下面哪一项列出了在第二年必须做该委员会成员的两个人?

A、G和 Y B、V和Y C、H和 I D、I和Y

48、如果H在第一年做主席,下面哪一位能够在第二年做主席?

A、F B、G C、Y D、I

49、如果F在某一年是该委员会成员,下面任何一位都可以再那一年是该委员会成员,除了:

A、I B、H C、G D、V 50、下面哪项一定为真?

A、H在第一年是该委员会成员。B、F在第二年是该委员会成员。C、I在两年之内都是该委员会成员。D、Z在第二年是该委员会成员。

2011年GCT入学资格考试逻辑推理能力测试试题

参考答案与解析

1.【答案】C

【解析】选项A、B和D指出了名师指路、读万卷书、行万里路、阅人无数是并没有等级之分,与题干不符。2.【答案】A

【解析】如果爱因斯坦的话是真的,那么若他的理论正确,则德国人会说他是德国人,若他的理论不正确,则法国人会说他是德国人。即不管正确与否,都会有人说他是德国人。

3.【答案】B

【解析】题设结论是,癌症患者不应该去医院治疗,医疗会增加死亡风险。要确定癌症患者去医院治疗是否会增加死亡的风险,需要指出的是去医院治疗的癌症患者的死亡率是否不去医院治疗的癌症患者的死亡率要高。即正确答案为B。对于选项A,因为在因医疗事故死亡的癌症患者中,即使不遭遇医院事故最终也会死于癌症的人只占很小的比率,也只能说明医疗事故是造成癌症患者死亡的原因之一,不足以说明癌症患者去医院治疗会增加死亡的风险。

4.【答案】C

【解析】根据题干信息,“知识改变命运,没有知识也改变命运”是对某局长和某主任整个经历的描述。前半句“知识改变命运”指的是他们由普通人成为领导人的过程,后半句“没有知识也改变命运”指的是他们对微博无知,因暴露隐私而遭到查处。与选项C的意思相近,故正确答案为C。5.【答案】B

【解析】“有些严肃且不讲情面的教师也是好教师”,而“好教师都是学识渊博的人”,故可推出有些严肃且不讲情面的教师是学识渊博的人,这与“有些学识渊博的教师是严肃且不讲情面的”是同真的,故正确答案为B。

6.【答案】C

【解析】根据题意可知,虽然本国法律禁止某些发明的商业性实施,但还是可以授予其发明专利权的。选项C与其是不一致的,故正确答案为C。

7.【答案】B

【解析】根据题意可得出命题:政客→要取悦他的选民;不说一些含糊其辞的话→不能取悦他的选民,即取悦了他的选民→说了一些含糊其辞的话。则马英九是位政客→要取悦他的选民→说了一些含糊其辞的话,即马英九说出了一些含糊其辞的话。故正确答案为B。

8.【答案】D

【解析】由题干知,一个指挥家必须通过赢得乐队对他所追求的艺术见解的尊重,才能获得指挥一流乐队反复进行排练的权威。这里产生了一个矛盾,如果指挥家不指挥乐队反复排练,怎么才能让乐队了解并尊重其艺术见解呢?为了排除该矛盾,显然必须假设:即使一种艺术见解还没有被充分表现出来,一流乐队也能够领悟这种艺术见解的优点。即正确答案为D。

9.【答案】C

【解析】题干的推理可总结为:蛋白质含量定得太高,奶农会往奶里添加提高蛋白质检测含量的东西。2008年的三聚氰胺事件就说明原来的标准太高了。从形式上讲,选项C与其推理方式相同,都犯了倒置因果的逻辑错误。

10.【答案】C

【解析】题设的论证指出,常常喝啤酒与发胖有因果关系。选项A和B指出发胖是否与其他饮食有关,选项D指出同时发生不能说明具有因果关系,都对题设论证提出了质疑。选项C的提问与题设没有必然的联系。故正确答案为C。11.【答案】D

【解析】由题干信息可知,没有外汇储备的增长→没有中国目前的国际影响力,不进行外汇储备投资→不会有外汇储备的增长,外汇储备投资→面临投资风险。一个命题与其逆否命题等价,故而可推出中国目前有国际影响力→外汇储备的增长→外汇储备投资→面临投资风险,即中国目前有国际影响力,肯定承担了投资风险。故正确答案为D。

12.【答案】A

【解析】依题设知:按照国家标准添加→没有安全问题,高于国家标准时强调自己未添加→不公平,则可推出,公平→高于国家标准时不应强调自己未添加。即为了保证公平性,企业或者不应该生产高于国家标准的产品,或者产品高于国家标准时不应强调自己未添加。正确答案为A。

13.【答案】C

【解析】太阳升起和小约翰祈祷本身显然没有相关性,不能因为事件先后发生而认定其具有必然的因果联系。故正确答案为C。

14.【答案】B

【解析】选项A指出标普的评级有误,美国信用应该不变,选项B指出美国国债可能会增持,收益率会降低,都一定程度上解释了题设中反常理的现象;选项D属于无关选项;选项B指出尽管美国信用等级降低,但美国国债还是相对安全和流动性高的投资品,仍然会有客户增持美国国债,从而导致收益率降低,从根本上解释了反常理的现象,是最好的解释,故正确答案为B。

15.【答案】A

【解析】依据题设陈述可知,未经反省的生命→不完整的生命,经反省的生命→为自己的生命。由此可推出,完整的生命→经反省的生命→为自己的生命,即选项B、C、D均为真,选项A不能由题设推出。正确答案为A。

16.【答案】C

【解析】由“中国人仇富”可推出“有的仇富者是中国人”,选项A为真;由“并非所有的富人都为富不仁”可推出“有的富人并非为富不仁”,选项B也为真;根据“至少我周围有的富人不是,我看到他们辛勤工作且有慈悲心怀”可推出“有的辛勤工作且有慈悲心怀的人是富人”。只有选项C是不能确定真假的,即为正确答案。

17.【答案】D

【解析】专家的观点是“如果我国允许转基因水稻商业化种植,国家对主要粮食作物的控制权就可能受到威胁”。选项D指出受国外专利影响,转基因的种子受到国外高价控制,从而国家对主要粮食作物的控制权就可能受到威胁,即支持了专家的观点。其余选项与专家提到的控制权无关。故正确答案为D。

18.【答案】B

【解析】“意思是说不要吃反季节蔬菜”,显然前提是要有反季节蔬菜,否则结论不成立。故正确答案为B。

19.【答案】B

【解析】题干的结论是社会管理者应该尽可能地关注来自弱势者的声音。原因是维护弱势人群的表达权,关系到社会的长治久安。显然选项B是最有力的补充,加强了题设论证。正确答案为B。

20.【答案】A

【解析】一审认为“许见到王老太时离她仅四、五米远,她摔倒定然是由于许的车惊吓”。选项A与一审并不冲突,不管许是王老太摔倒前或后开车过来,她都可能是受车的惊吓而摔倒;选项B提到的道德与依法审案无关;选项A指出,王老太应该对许的车有预期,不会是受惊吓摔倒的,一审判决有误,选项D表述不充分。综上知,正确答案为A。

21.【答案】C

【解析】题设中没有提出关于量变的信息,其他选项均有涉及,故正确答案为C。22.【答案】B

【解析】由题设可得出命题:罗马皇帝→都喝葡萄酒,且只用锡壶和锡高脚杯喝酒;使用锡器皿去饮酒→导致中毒;中毒→导致精神错乱。因为尼禄是罗马皇帝,所以根据关系的传递性可知,尼禄皇帝肯定是精神错乱的。故正确答案为B。

23.【答案】C

【解析】根据题意,美国动用否决权→在整个阿拉伯世界引燃反美情绪;美国不动用否决权→奥巴马失去一部分支持以色列的选民。即选项C中给出的选言命题为真,两个选言支中至少有一个为真,正确答案为C。

24.【答案】D

【解析】题设的论证指出,融资渠道是相对于企业家个人素质、公司战略规划等因素对一个新公司的成功发展更为重要的因素。选项D指出风险投资人在选择投资时,是考虑了企业家的个人素质、公司的战略规划等因素的,直接削弱了题设论证的结论,即企业家的个人素质、公司的战略规划等因素也是一个新公司的成功发展的重要因素。选项A和B是无关选项,选项C有一定的支持作用。故正确答案为D。

25.【答案】A

【解析】总结题设的命题“只有…才…”可得出:发展进步→开放包容。选项A可简化为:不开放包容→发展进步,结合题设条件和关系的传递性可得,不开放包容→开放包容,自相矛盾,故知选项A的命题为假;选项B可简化为:不开放包容→不能发展进步,是“发展进步→开放包容”的逆否命题,为真;选项C可简化为:发展进步→开放包容,为真;选项D可简化为:开放包容→可能不会发展进步,可真可假。故正确答案为A。

26.【答案】D

【解析】选项A表明,大型哺乳动物灭绝可能是气候的变化导致,而与人类的活动无关,直接颠覆了题设的陈述。故正确答案为D。

27.【答案】C

【解析】选项A说明50年前的人更喜欢买书,现在每年的售书册应该比50年前少,与题干的现象冲突,故正确答案为C。

28.【答案】B

【解析】题设的观点是“全社会都应该以包容的心态对待„异质思维‟”,没有提到要严格管制,即选项A不支持题设观点,故正确答案为B。

29.【答案】A

【解析】根据题干,一年中任何时候,或者有季节性促销,或者有节日促销,或者两者兼而有之。由此可推出,如果某个月没有季节性促销,这个月一定有节日促销。正确答案为A。

30.【答案】C

【解析】“在统计学上,32分与上述其他人数之间的差距有重要的意义”,这说明分数之间代表的内容之间是有相关性的。即是否上过学前班与小学入学前的综合能力之间有相关性,正确答案为C。

31.【答案】D

【解析】选项D指出,中国稀土储量只占全球储量的36%,不及美国,俄罗斯和澳大利亚占有量之和,但产量占到了世界产量的97%,其他3个国家均为零,显然指责中国垄断资源是毫无根据的,相反中国对世界稀土产业的贡献是极大的,即选项A是合适的反驳。其余选项只有较弱的反驳作用,故正确答案为D。

32.【答案】B

【解析】反对党执政的前一个4年,CPI涨幅都超过了4%,我们党执政的接下来4年,CPI涨幅降到1.5%,这说明CPI总是在涨的,商品的价格也在增长。这与商品的价格越来越低的结论产生了矛盾。即实际上,用来支持这一结论的证据实际上为否定这一结论提供了更多的支持,正确答案为B。

33.【答案】B

【解析】引入其他导致股票价格下跌的原因,正确答案为B。

34.【答案】A

【解析】题干说明建立了专门化部门,结论是为了更准确的抓住群体。根据逻辑的严密性可知,需要的假设是,这些部门是在以更精准的方式抓住消费群体,正确答案为A。这里选项C的假设“之前完全没有抓住消费群体”是无关紧要的。

35.【答案】C

【解析】题干运用了两难推理,即允许农民出卖自己的房屋→损害农民生存权益,不允许出卖房屋→侵害农民资产权益,故而或者损害农民生存权益,或者侵害农民资产权益。正确答案为C。

36.【答案】C

【解析】题干的谬误在于,根据两个对象有某一共同属性,推断出它们是同一类对象。选项C的论证是最好的评价,即为正确答案。

37.【答案】D

【解析】选项D指出,航母多也有弊端,即航母多导致相应配套的航舰和飞机减少,从而得不到有效的支持和护卫,这很好地解释了专家的态度。选项A属于无关选项;选项B不能解释支持航母多的态度,因为有航母,即可捏造“中国威胁论”;选项C提到的只是暂时现象,一旦该现在得以缓解,这个解释就不成立了。故正确答案为D。

38.【答案】B

【解析】题干得出结论所采用的论据是一个物理学家的观点,显然这是依赖某个专家的权威来支持一个前提。故正确答案为B。

39.【答案】C

【解析】题设对数字的分析表明,国内油价容易涨,不容易跌,但没能说明上涨后的价格不会明显跌下来,选项C的陈述很好地弥补了这一点,即随着价格的增加,国内的反应变得迟钝,总体而言价格就出现了“涨快跌慢”的现象。故正确答案为C 40.【答案】A

【解析】如果用“<”替代“个子矮”,那么题设的条件可写为:M

41.【答案】C

【解析】根据条件(3)和条件(4)及题设条件“F安排在第五周上演”可知,K不能被安排在最后一周,J只能安排在第四或六周,即K和J不能安排在第七周。O,R和S安排在第七周与题设条件不冲突,故正确答案为C。

42.【答案】A

【解析】根据条件(3)知,K安排在J之前,又根据条件(4)知,F和J是连续的,故而K肯定安排在F之前,即正确答案为A。

43.【答案】A

【解析】已知O安排在第二周,G安排在第三周,R安排在第五周。则根据条件(4)可知,F和J只能安排在连续的两周中,即第六、七周,位置可以互换。又根据条件(3)可知,K要在S之前,即K肯定安排在第一周,S肯定安排在第四位。即可供选择的安排方式有两种:KOGSRFJ和KOGSRJF。正确答案为A。

44.【答案】D

【解析】已知G安排在第三周,S安排在第六周。则根据条件(4)可知,F和J只能安排在连续的两周中,即第一、二周或者第四、五周。又根据条件(3)可知,K要在J之前,即F和J只能安排在第四、五周,位置可以互换,K安排在第一或二周。又根据条件(2)只,O和S不能连续演出,即O不能安排在第七周,只能安排在第一或二周。综上知,R只能安排在剩余的第七周。即正确答案为D。

45.【答案】B

【解析】如果O恰好被安排在J之前的那一周,则根据条件(4)可知,OJF要连续上演。又根据条件(1)可知,G安排在第三周,OJF要安排在最后四周中的三周。根据条件(3),K在S之前上演,而G之后只有一周剩余,故而K必然在G之前上演。即正确答案为B。

46.【答案】B

【解析】选项A中的名单违背了条件“G和V不能在同一年成为该委员会的成员”;选项D中的名单违背了条件“2名成员来自下面4位法官:F,G,H和I”;选项C中的名单违背了条件“H和Y不能在同一年成为该委员会的成员”。故而采用排除法知,正确答案为B。

47.【答案】D

【解析】由“V在第一年做该委员会主席”、“在第一年做主席的成员在第二年必须退出该委员会”和“2名成员来自下面3位科学家:V,Y和Z”知,第二年的成员有Y和Z。又“每一年,I和V中有且只有一位做该委员会的成员”可知,第二年的成员还有I。综上知,第二年的成员有I,Y和Z,另外一名不确定。故正确答案为D。

48.【答案】A

【解析】根据“H在第一年做主席”和“H和Y不能在同一年成为该委员会的成员”可知,Y不会是第一年的成员。又“2名成员来自下面3位科学家:V,Y和Z”可知,第一年的成员有“V和Z”。再根据“G和V不能在同一年成为该委员会的成员”和“每一年,I和V中有且只有一位做该委员会的成员”可知,G和I也不会出现在第一年。又“第二年做主席的人在第一年必须是该委员会的成员”,故而Y,G和I都不会是第二年的主席。根据排除法知,正确答案为A。

49.【答案】C

【解析】如果F和G是该委员会的成员,则根据“G和V不能在同一年成为该委员会的成员”知,V不能是该年的成员。又“每一年,I和V中有且只有一位做该委员会的成员”和“2名成员来自下面3位科学家:V,Y和Z”,故I,Y和Z是该年的成员。即该年有F,G,I,Y和Z共5名委员组成,这与题设的条件“每年都由4人组成”矛盾。故而G不能与F是同一年的成员,正确答案为C。

50.【答案】D

【解析】如果Z不是第二年的成员,则根据“2名成员来自下面3位科学家:V,Y和Z”可知,第二年的成员中友V和Y。又根据“G和V不能在同一年成为该委员会的成员”,“ 每一年,I和V中有且只有一位做该委员会的成员”和“H和Y不能在同一年成为该委员会的成员”可知,4位法官中只有F可以是该年的成员,这与题设是相矛盾的。故Z是第二年的成员,即正确答案为D。

下载2005工程硕士GCT考试真题(英语、语文、逻辑)word格式文档
下载2005工程硕士GCT考试真题(英语、语文、逻辑).doc
将本文档下载到自己电脑,方便修改和收藏,请勿使用迅雷等下载。
点此处下载文档

文档为doc格式


声明:本文内容由互联网用户自发贡献自行上传,本网站不拥有所有权,未作人工编辑处理,也不承担相关法律责任。如果您发现有涉嫌版权的内容,欢迎发送邮件至:645879355@qq.com 进行举报,并提供相关证据,工作人员会在5个工作日内联系你,一经查实,本站将立刻删除涉嫌侵权内容。

相关范文推荐

    2009年GCT考试(英语)真题及答案

    第四部分 外语运用能力测试(英语) (50题,每题2分,满分100分) Part One Vocabulary and Structure Directions: There are ten incomplete sentences in this part. For each......

    历年GCT逻辑考试真题试卷及答案4

    2011GCT考试真题试卷及答案-逻辑 第三部分逻辑推理能力测试 (50题,每题2分,满分100分) 1.1993年以来,我国内蒙古地区经常出现沙尘暴,造成重大经济损失。有人认为,沙尘暴是由于气候干......

    2013年在职硕士GCT逻辑考试真题及答案

    新东方在线在职硕士频道(http://shuoshi.koolearn.com/zzlk/)分享 2013年在职硕士GCT逻辑考试真题及答案 第三部分 逻辑推理能力测试 (50题,每小题2分,满分100分) 1.人或许可......

    2006年GCT入学考试逻辑真题及答案

    2006年GCT入学考试逻辑真题及答案 http://www.xiexiebang.com 北京外国语大学外研培训中心 2009-3-28 22:55:30 2006GCT考试真题试卷及答案-逻辑 第三部分逻辑推理能力测试......

    2010年GCT逻辑真题及答案5篇

    2010 年GCT 逻辑推理能力测试试题 (50 题,每题2 分,满分100 分) 1. 去年,有6000 人死于醉酒,有4000 人死于开车,但只有500 人死于醉酒开车。因此,醉酒开车比单纯的醉酒或者单纯的开......

    2011年在职GCT逻辑真题及答案(A)

    http://www.xiexiebang.com 2011-10-31 15:16:13 考易网 2011年在职GCT逻辑真题及答案(A卷) http://www.xiexiebang.com 2011-10-31 15:16:13 考易网 逻辑A卷参考答案 1......

    2010年海南GCT考试语文真题

    2010年GCT真题及答案分析 第一部分 语言表达能力测试 (50题,每题2分,满分100分) 一、选择题 1.下面没有错别字的一句是 A.光阴荏苒,岁月流逝,大学毕业到现在,转眼40多年过去了。......

    2003年GCT逻辑真题及答案(共5篇)

    2003 年GCT 入学资格考试逻辑推理能力试题 (50 题,每题2 分,满分100 分,考试时间45 分钟) 1. 古希腊哲人说,未经反省的人生是没有价值的.下面哪一个选项与这句格言的意思最不接近......